1

Post on 16-Jan-2023

5 views 0 download

Transcript of 1

Solutions Manual

FINANCIALMANAGEMENTPrinciples and Practice

Fourth Edition

Timothy J. GallagherColorado State University

Joseph D. Andrew, Jr.Webster University

2006 Freeload Press, Madison Wisconsin

(Insert publication data on this page)

i

Solutions Manualto accompany

Financial Management: Principles and Practice

4rd Edition

by Timothy J. Gallagher and Joseph D. Andrew, Jr.

This solutions manual provides the answers to all the review questions and end-of-chapter problems in Financial Management: Principles and Practice, by Gallagher and Andrew. The answers and the steps taken to obtain the answers are shown.

We remind our readers that in finance there is often more than one answer to a question or to a problem, depending on one’s viewpoint and assumptions. We provide one answer to each question and show one approach to solving each problem. Other answers and approaches may be equally valid, or judged even better according to each individual’s preference.

ii

TABLE OF CONTENTS

Chapter 1 Solutions............................................5

Chapter 2 Solutions............................................9

Chapter 3 Solutions...........................................13

Chapter 4 Solutions...........................................16

Chapter 5 Solutions...........................................24

Chapter 6 Solutions...........................................34

Chapter 7 Solutions...........................................41

Chapter 8 Solutions...........................................53

Chapter 9 Solutions...........................................61

Chapter 10 Solutions..........................................67

Chapter 11 Solutions..........................................79

Chapter 12 Solutions..........................................93

Chapter 13 Solutions.........................................103

Chapter 14 Solutions.........................................113

Chapter 15 Solutions.........................................120

Chapter 16 Solutions.........................................124

Chapter 17 Solutions.........................................131

Chapter 18 Solutions.........................................138

Chapter 19 Solutions.........................................147

iii

Chapter 20 Solutions.........................................163

Chapter 21 Solutions.........................................167

iv

Chapter 1 Solutions

Answers to Review Questions

1. How is finance related to the disciplines of accounting and economics?

Financial management is essentially a combination of accounting andeconomics. First, financial managers use accounting information—balance sheets, income statements, and so on—to analyze, plan, and allocate financial resources for business firms. Second, financial managers use economic principles to guide them in making financial decisions that are in the best interest of the firm. In other words, finance is an applied area of economics that relies on accounting for input.

2. List and describe the three career opportunities in the field of finance.

Finance has three main career paths: financial management, financial markets and institutions, and investments.

Financial management involves managing the finances of a business. Financial managers—people who manage a business firm's finances—perform a number of tasks. They analyze and forecast a firm's finances; assess risk, evaluate investment opportunities, decide when and where to find money sources and how much money to raise, and decide how much money to return to the firm's investors.

Bankers, stockbrokers, and others who work in financial markets andinstitutions focus on the flow of money through financial institutions and the markets in which financial assets are exchanged. They track the impact of interest rates on the flow of that money.

People who work in the field of investments locate, select, and manage income-producing assets. For instance, security analysts andmutual fund managers both operate in the investment field.

5

3. Describe the duties of the financial manager in a business firm.

Financial managers measure the firm's performance, determine what the financial consequences will be if the firm maintains its present course or changes it, and recommend how the firm should useits assets. Financial managers also locate external financing sources and recommend the most beneficial mix of financing sources,and they determine the financial expectations of the firm's owners.

All financial managers must be able to communicate, analyze, and make decisions based on information from many sources. To do this,they need to be able to analyze financial statements, forecast and plan, and determine the effect of size, risk, and timing of cash flows.

4. What is the basic goal of a business?

The primary financial goal of the business firm is to maximize the wealth of the firm's owners. Wealth, in turn, refers to value. Ifa group of people owns a business firm, the contribution that firm makes to that group's wealth is determined by the market value of that firm.

5. List and explain the three financial factors that influence the value of a business.

The three factors that affect the value of a firm's stock price arecash flow, timing, and risk.

The Importance of Cash Flow: In business, cash is what pays the bills. It is also what the firm receives in exchange for its products and services. Cash is therefore of ultimate importance, and the expectation that the firm will generate cash in the future is one of the factors that gives the firm its value.

The Effect of Timing on Cash Flows: Owners and potential investorslook at when firms can expect to receive cash and when they can expect to pay out cash. All other factors being equal, the sooner companies expect to receive cash and the later they expect to pay

6

out cash, the more valuable the firm and the higher its stock pricewill be.

The Influence of Risk: Risk affects value because the less certainowners and investors are about a firm's expected future cash flows,the lower they will value the company. The more certain owners and investors are about a firm's expected future cash flows, the higherthey will value the company. In short, companies whose expected future cash flows are doubtful will have lower values than companies whose expected future cash flows are virtually certain.

6. Explain why accounting profits and cash flows are not the same thing.

Stock value depends on future cash flows, their timing, and their riskiness. Profit calculations do not consider these three factors. Profit, as defined in accounting, is simply the difference between sales revenue and expenses. It is true that more profits are generally better than less profits, but when the pursuit of short-term profits adversely affects the size of future cash flows, their timing, or their riskiness, then these profit maximization efforts are detrimental to the firm.

7. What is an agent? What are the responsibilities of an agent?

An agent is a person who has the implied or actual authority to acton behalf of another. The owners whom the agents represent are theprincipals. Agents have a legal and ethical responsibility to makedecisions that further the interests of the principals.

8. Describe how society's interests can influence financial managers.

Sometimes the interests of a business firm's owners are not the same as the interests of society. For instance, the cost of properly disposing of toxic waste can be so high that companies maybe tempted to simply dump their waste in nearby rivers. In so doing, the companies can keep costs low and profits high, and drivetheir stock prices higher (if they are not caught). However, many people suffer from the polluted environment. This is why we have

7

environmental and other similar laws: So that society's best interests take precedence over the interests of individual company owners.

When businesses take a long-term view, the interests of the owners and society often (but not always) coincide. When companies encourage recycling, sponsor programs for disadvantaged young people, run media campaigns promoting the responsible use of alcohol, and contribute money to worthwhile civic causes, the goodwill generated as a result of these activities causes long-termincreases in the firm's sales and cash flows, which translate into additional wealth for the firm's owners.

9. Briefly define the terms proprietorship, partnership, and corporation.

A proprietorship is a business owned by one person.

Two or more people who join together to form a business make up a partnership. This can be done on an informal basis without a written partnership agreement, or a contract can spell out the rights and responsibilities of each partner.

A limited liability company is a hybrid between a partnership and acorporation. Profits and losses pass through to the members. Members generally enjoy limited liability.

Corporations are legal entities separate from their owners. To forma corporation, the owners specify the governing rules for the running of the business in a contract known as the articles of incorporation. They submit the articles to the government of the state in which the corporation is formed, and the state issues a charter that creates the separate legal entity.

10. Compare and contrast the potential liability of owners of proprietorships, partnerships (general partners), and corporations.

The sole proprietor has unlimited liability for matters relating tothe business. This means that the sole proprietor is responsible for all the obligations of the business, even if those obligations exceed the amount the proprietor has invested in the business.

8

Each partner in a partnership is usually liable for the activities of the partnership as a whole. Even if there are a hundred partners, each one is technically responsible for all the debts of the partnership. If ninety-nine partners declare personal bankruptcy, the hundredth partner still is responsible for all the partnership's debts.

A corporation is a legal entity that is liable for its own activities. Stockholders, the corporation's owners, have limited liability for the corporation's activities. They cannot lose more than the amount they paid to buy the corporation’s stock.

Answers to End-of-Chapter Problems

1. An accountant prepares financial statements while a financial analyst interprets them.

2. A financial manager’s role in a publicly traded company is to make financial decisions so as to best serve the principal stockholders.

3. a. The value of the firm would go down due to the increase in theamount of time it takes to receive the cash inflows.

b. The value of the firm would go up due to the increase in expected cash inflows.

c. If expected future cash flows do not change the value of the firm would go down due to the increased riskiness of the firm.

4. This practice obviously takes advantage of people who are in a difficult financial situation. This transaction is voluntary, however, and high risk loans have high interest rates.

9

5. LLCs have a small number of members like partnerships and each ofthese members is likely to have an active voice in the company like a partnership. The LLC is taxed like a partnership. Unlikea partnership, and more like a corporation, the owners generally enjoy limited liability.

10

Chapter 2 Solutions

Answers to Review Questions

1. What are financial markets? Why do they exist?

Financial markets are where financial securities are bought and sold. They exist primarily to bring deficit economic units (those needing money) and surplus economic units (those having extra money) together.

2. What is a security?

Securities are claims on financial assets. They can be described as “claim checks” that give their owners the right to receive fundsin the future. Securities are traded in both the money and capitalmarkets. Money market securities include Treasury bills, negotiablecertificates of deposit, commercial paper, and banker’s acceptances. Capital market securities include bonds and stock.

3. What are the characteristics of an efficient market?

The term market efficiency refers to the ease, speed, and cost of trading securities. In an efficient market, securities can be traded easily, quickly, and at low cost. Markets lacking these qualities are considered to be inefficient.

4. How are financial trades made on an organized exchange?

Each exchange-listed security is traded at a specified location on the trading floor called the post. The trading is supervised by specialists who act either as brokers (bringing together buyers andsellers) or as dealers (buying or selling the stock themselves). Prominent international securities exchanges include the New York Stock Exchange (NYSE) and major exchanges in Tokyo, London, Amsterdam, Frankfurt, Paris, Hong Kong, and Mexico.

11

5. How are financial trades made in an over-the-counter market? Discuss the role of a dealer in the OTC market.

In contrast to the organized exchanges, which have physical locations, the over-the-counter market has no fixed location,or more correctly, it is everywhere. The over-the-counter market, or OTC, is a network of dealers around the world who maintain inventories of securities for sale. If you wanted to buy a securitythat is traded OTC, you would call your broker, who would then shopamong competing dealers who have the security in their inventory. After locating the dealer with the best price, your broker would buy the security on your behalf.

The role of dealers: Dealers make their living buying securities and reselling them to others. They operate just like car dealers who buy cars from manufacturers for resale to others. Dealers makemoney by buying securities for one price (called the bid price) andselling them for a higher price, (called the ask price). The difference, or spread, between the bid price and the ask price represents the dealer’s fee.

6. What is the role of a broker in security transactions? How are brokers compensated?

Brokers handle orders to buy or sell securities. Brokers are agentswho work on behalf of an investor. When investors call with an order, brokers work on their behalf to find someone to take the other side of the proposed trade. If investors want to buy, brokersfind sellers. If investors want to sell, brokers find buyers. Brokers are compensated for their services when the person whom they represent, the investor, pays them a commission on the sale orpurchase of securities.

7. What is a Treasury bill? How risky is it?

Treasury bills are short-term debt instruments issued by the U.S. Treasury that are sold at a discount and pay face value at maturity. They are very nearly risk-free as they are backed by the

12

U.S. Government which could, if need by, print money to pay their holders at maturity.

8. Would there be positive interest rates on bonds in a world with absolutely no risk (no default risk, maturity risk, and so on)? Whywould a lender demand, and a borrower be willing to pay, a positiveinterest rate in such a no-risk world?

Yes, there would be a positive rate of interest in a risk-free world. This is because regardless of risk, lenders of money must postpone spending during the time the money is loaned. Lenders, then, lose the opportunity to invest their money for that period oftime. To compensate for the cost of losing investment opportunitieswhile they postpone their spending, lenders demand, and borrowers pay, a basic rate of return, the real rate of interest.

Answers to End of Chapter Problems

2-1. a. Surplus economic units have income that exceeds their expenditures. Wealthy families in the household sector and most states (which have balanced budget requirements) are surplus economic units.

b. Deficit economic units have expenditures that exceed their incomes. Home buyers and college students are likely to be deficiteconomic units.

2.2. a. falseb. falsec. falsed. false

2-3. a. 2 3 4 1

b. The money market is dominated by large institutional traders and there is much competition. The New York Stock Exchange tends to have larger more actively traded stocks. The over-the-counter market tends to have smaller less actively traded securities. The

13

real estate market has very high transaction costs and trades take months.

2.4. a. A money market security is short term and actively traded.

b. Treasury bills and commercial paper are both traded in the moneymarket.

2-5. $66.25/$1,000 = 6 5/8 % coupon rate

2-6. The yield on a Bonds-R-Us bond:

Real rate of interest...................... 2%Inflation premium........................ 3%Default risk premium................... 1%Liquidity risk premium................ 1%Maturity risk premium................. 1%

Total yield on Bonds-R-Us Bond: 8%

(reference figure 2-2)

2-7. Treasury Yield Curve:

Given:

Treasury SecurityYields:

Maturity in Years (for Chart)

Three-month T-bills

4.50% 0.25

Six-month T-bills4.75

% 0.5

One-year T-notes5.00

% 1

Two-year T-notes5.25

% 2Three-year T-

bonds5.50

% 3Five-year T-bonds 5.75 5

14

%

Ten-year T-bonds6.00

% 10Thirty-year T-

bonds6.50

% 30

Chart: (see next page)

Implications:

a. For borrowers: Borrowers tend to look for the low point of thecurve, which indicates the least expensive loan maturity. In this case the low point is 3 months, leading the borrower to seek a short-term loan. However, if a firm borrows long-term and obtains the higher interest rate, that rate is locked in for the life of the loan (30 years in this case). If interest rates rise the borrower may be glad he/she locked in the long-term rate.

b. Lenders face the opposite situation. Granting short-term-term loans at relatively low interest rates may look unattractive now; but if short-term rates rise, the lenders will be able to roll overinvestments at higher and higher rates.

15

16

Chapter 3 Solutions

Answers to Review Questions

1. Define intermediation.

The financial system makes it possible for surplus and deficit economic units to come together, exchanging funds for securities, to their mutual benefit. When funds flow from surplus economic units to a financial institution to a deficit economic unit, the process is known as intermediation. The financial institution acts as an intermediary between the two economic units.

2. What can a financial institution often do for a surplus economic unit that it would have difficulty doing for itself if the surplus economic unit (SEU) were to deal directly with a deficit economic unit (DEU)?

Surplus economic units do not usually have the expertise to determine whether deficit economic units can and will make good on their obligations, so it is difficult for them to predict when a would-be deficit economic unit will fail to pay what it owes. Sucha failure is likely to be devastating to a surplus economic unit that has lent a proportionately large amount of money. In contrast, a financial institution is in a better position to predict who will pay and who won't. It is also in a better position, having greater financial resources, to occasionally absorb a loss when someone fails to pay. (This is just one example of the beneficial things financial institutions do for SEUs)

3. What can a financial institution often do for a deficit economic unit (DEU)that it would have difficulty doing for itself if the DEUwere to deal directly with an SEU?

SEUs typically want to supply a small amount of funds, while DEUs typically want to obtain a large amount of funds. Thus it is oftendifficult for surplus and deficit economic units to come together

17

on their own to arrange a mutually beneficial exchange of funds forsecurities. A financial institution can step in and save the day. A bank, savings and loan, or insurance company can take in small amounts of funds from many individuals, form a large pool of funds,and then use that large pool to purchase securities from individualbusinesses and governments. (This is just one example of the beneficial things financial institutions do for DEUs)

4. What are a bank's primary reserves? When the Fed sets reserve requirements, what is its primary goal?

Vault cash and deposits in the bank's account at the Fed are used to satisfy these reserve requirements; they are called primary reserves. These primary reserves are non-interest-earning assets held by financial institutions.

The Federal Reserve requires all commercial banks to keep a minimumamount of reserves on hand to meet the withdrawal demands of its depositors and to pay other obligations as they come due. Many would argue, however, that the reserve requirement is set more withmonetary policy in mind than to ensure that banks meet their depositors' withdrawal requests.

5. Compare and contrast mutual and stockholder-owned savings and loan associations.

Some savings and loan associations are owned by stockholders, just as commercial banks and other corporations are owned by their stockholders. Other S&Ls, called mutuals, are owned by their depositors. When a person deposits money in an account at a mutualS&L, that person becomes a part owner of the firm. The mutual S&L's profits (if any) are put into a special reserve account from which dividends are paid from time to time to the owner/depositors.

6. Who owns a credit union? Explain.

Credit unions are owned by their members. When credit union members put money in their credit union, they are not technically "depositing" the money. Instead, they are purchasing shares of the

18

credit union. In general, credit unions exist to pay interest on shares bought by, and collect interest on loans made to, the members.

7. Which type of insurance company generally takes on the greater risks: a life insurance company or a property and casualty insurance company?

The risks protected against by property and casualty companies are much less predictable than are the risks insured by life insurance companies. Hurricanes, fires, floods, and trial judgments are all much more difficult to predict than the number of sixty-year-old females who will die this year among a large number in this risk class. This means that property and casualty insurance companies must keep more liquid assets than do life insurance companies.

8. Compare and contrast a defined benefit and a defined contribution pension plan.

In a defined benefit plan, retirement benefits are determined by a formula that usually considers the worker's age, salary, and years of service. The employee and/or the firm contribute the amounts necessary to reach the goal. In a defined contribution plan, the contributions to be made by the employee and/or employer are spelled out, but retirement benefits depend on the total accumulation in the individual's account at the retirement date.

9. Special security software is used such that customers who enter their identification and password information can keep sensitive information out of the hands of hackers.

19

Answers to End-of-Chapter Problems

3-1. a) If there were no financial institutions the SEUs and the DEUs would find that the amount of money needed by a given DEU did not match the amount of money available by a given SEU. The money available would not be put to work and the economic activity that would have otherwise taken place would not.

b) If financial institutions were available in this society they could position themselves between the SEUs and DEUs. The financialinstitution could pool the $1,000 available (100 SEUs times $10 each) and pass that money along in $100 increments to the DEUs. This could be done via either a debt or equity claim that the financial institution would accept from the DEU in return for the money.

3-2. a) .10 rate on loans made - .05 rate paid to depositors = .05 = 5%interest rate spread

b) (.5 x .10) + (.5 x .12) = .11 = 11% weighted average loan rate (.5 x .05) + (.5 x .07) = .06 = 6% weighted average deposit rate 11% - 6% = 5% interest rate spread

3-3. ($48,300,000 - $7,800,000) x .03) + (($60,000,000 - $48,300,000) x .10) + ($20,000,000 x 0) + ($10,000,000 x 0) = $6,732,000

3-4. a) The FOMC should buy government securities in the open market. This would increase the reserves of the banking system and would put downward pressure on the federal funds rate.

b) The Fed’s trader at the New York Federal Reserve Bank would contact various government securities dealers and would buy the Treasury securities from them. Payment would be made by crediting the accounts at the Fed of these dealers. This would make more funds available and would tend to put downward pressure on the costof these funds, the federal funds rate.

3-5. a) ($1,000,000 x .08) – ($1,000,000 x .07) = $10,000 a profit of $10,000

20

b) ($1,000,000 x .08) – ($1,000,000 x .09) = -$10,000 a loss of $10,000

21

Chapter 4 Solutions

Answers to Review Questions

1. Why do total assets equal the sum of total liabilities and equity? Explain.

Assets = Liabilities + Equity

Assets are the items of value a business owns. Liabilities are claims on the business by non-owners, and equity is the owners' claim on the business. The sum of the liabilities and equity is thetotal capital contributed to the business, which, by definition, equals the total value of the assets.

2. What are the time dimensions of the income statement, the balance sheet, and the statement of cash flows? Hint: Are they videos or still pictures? Explain.

The income statement is like a video: It measures a firm's profitability over a period of time (which can be a week, a month, a year, or any other time period).

The balance sheet is like a still photograph. The balance sheet shows the firm's assets, liabilities, and equity at a given point in time.

This cash flow statement like the income statement, can be comparedto a video: It shows how cash flows into and out of a company over a given period of time.

3. Define depreciation expense as it appears on the income statement. How does depreciation affect cash flow?

Accounting depreciation is the allocation of an asset's initial cost over time. Depreciation expense on an income statement is the

22

amount of the asset=s initial cost allocated to the period covered by the income statement.

Depreciation expense is not a cash flow. Depreciation as an expense category affects cash flow, however, because it is tax-deductible. Depreciation expense lowers a company’s taxable incomeand, therefore its income tax liability. In this way depreciation reduces cash outflows..

4. What are retained earnings? Why are they important?

Retained earnings represents the sum of all the earnings available to common stockholders of a business during its entire history, minus the sum of all the common stock dividends which it has ever paid. Those earnings that were not paid out were, by definition, retained.

Retained earnings are important because they represent amounts reinvested in a company on behalf of the company’s owners instead of being paid out in the form of dividends.

5. Explain how earnings available to common stockholders and common stock dividends paid from the current income statement affect the balance sheet item retained earnings.

The change in the retained earnings account from one balance sheet to the next equals net income less preferred stock dividends (whichis the amount of earnings available to common stockholders) less common stock dividends.

6. What is accumulated depreciation?

Depreciation is the allocation of an asset's initial cost over time. Accumulated depreciation is the total of all the depreciation expense that has been recognized to date.

7. What are the three major sections of the statement of cash flows?

Cash flows from OperationsCash flows from investing activities

23

Cash flows from financing activitiesNet change in cash balanceCash balance at beginning of periodCash balance at end of period

8. How do financial managers calculate the average tax rate?

Average tax rates are calculated by dividing tax dollars paid by earnings before taxes (EBT).

9. Why do financial managers calculate the marginal tax rate?

Financial managers use marginal tax rates to estimate the future after-tax cash flows from investments. Since they are interested in how much of the next dollar earned from new investments will have to be paid in taxes, they use the marginal tax rate (rather than the average tax rate) to calculate the tax liability.

10. Identify whether the following items belong on the income statementor the balance sheet.

a. Interest Expense IS l. Cash BSb. Preferred Stock Dividends Paid IS m. Capital in Excess of Par BSc. Plant and Equipment BS n. Operating Income ISd. Sales IS o. Depreciation Expense ISe. Notes Payable BS p. Marketable Securities BSf. Common Stock BS q. Accounts Payable BSg. Accounts Receivable BS r. Prepaid Expenses BSh. Accrued Expenses BS s. Inventory BSi. Cost of Goods Sold IS t. Net Income ISj. Preferred Stock BS u. Retained Earnings BSk. Long-Term Debt BS

11. Indicate in which section the following balance items belong (current assets, fixed assets, current liabilities, long-term liabilities, or equity).

a. Cash CA h. Capital in Excess of Par EQb. Notes Payable CL i. Marketable Securities CA

24

c. Common Stock EQ j. Accounts Payable CLd. Accounts Receivable CA k. Prepaid Expenses CAe. Accrued Expenses CL l. Inventory CAf. Preferred Stock EQ m. Retained Earnings EQg. Plant and Equipment FA

Answers to End-of-Chapter Problems

4-1. CASE A CASE B

Revenues 200,000 110,000Expenses 160,000 70,000Net Income 40,000 40,000 Retained Earnings, Jan 1 300,000 100,000Dividends Declared 70,000 30,000Retained Earnings, Dec 31 270,000 110,000Current Assets, Dec 31 80,000 230,000Non-current Assets, Dec 31 850,000 180,000Total Assets, Dec 31 930,000 410,000Current Liabilities, Dec 31 40,000 60,000Non-current Liabilities, Dec 31 100,000 140,000Total Liabilities, Dec 31 140,000 200,000CS & Cap. in Excess of Par, Dec 31 520,000 100,000Total Stockholders’ Equity, Dec 31 790,000 210,000

4-2. CASE A CASE B

Sales 500,000 250,000COGS 200,000 100,000Gross Profit 300,000 150,000Operating Expenses 60,000 60,000Operating Income (EBIT) 240,000 90,000Interest Expense 10,000 10,000Earnings Before Taxes (EBT) 230,000 80,000Tax Expense (40%) 92,000 32,000Net Income 138,000 48,000

25

4-3. a) 15%; $48,000 X 0.15 = $7,200b) $7,200/$48,000 = 0.15 or 15%

4-4. a) Tax = $50,000 X 0.15 + $25,000 X 0.25 + $25,000 X 0.34 + $50,000 X 0.39

= $41,750

b) Effective tax rate = $41,750/$150,000 = 0.2783 or 27.83%

4-5. The marginal tax rate is the tax rate applied to the next dollar ofincome. Therefore, the marginal tax rate is 34%.The average tax rate is 34%50,000 * .15 = 7,50025,000 * .25 = 6,25025,000 * .34 = 8,500235,000 * .39 = 91,6502,865,000 * .34 = 974,100 $1,088,000$1,088,000/$3,200,000 = 34%

4-6. $1 + $400,000/200,000 = $3.00 per share

4-7. Sales $10,000,000- Operating Costs 5,200,000- Interest Expense 200,000= EBT $4,600,000

- Taxes (40%) 1,840,000 Net after-tax income $2,760,000

Simon’s net after-tax income was $2,760,000 for the year.

4-8. Depreciation expense in 2006 = $70,000 - $60,000 = $10,000.

4-9a) Cash + Marketable Securities + Inventory + Accounts Receivable +

Prepaid expenses.(11,000,000 + 9,000,000 + 11,000,000 + 3,000,000 + 1,000,000) = 35,000,000

26

Current Assets = $35,000,000

b) Fixed assets – depreciation30,000,000 – 8,000,000 = 22,000,000Net Fixed Assets = $22,000,000

c) Notes Payable + Accrued Expenses4,000,000 + 2,000,000 = 6,000,000Current Liabilities = $6,000,000

d) Current Assets – Current Liabilities(11,000,000 + 9,000,000 + 11,000,000 + 3,000,000 + 1,000,000) – (4,000,000 + 2,000,000) 35,000,000 – 6,000,000 = 29,000,000Net Working Capital = $29,000,000

4-10. a ) Gross Profit $440,000 - $200,000 = $240,000b ) Operating Income (EBIT) $240,000 - $40,000 - 85,000 =

$115,000c ) Earning Before Taxes (EBT) $ 115,000 - $40,000 = $75,000 d ) Income Taxes $ 75,000 X 0.4 = $30,000e ) Net Income $75,000 - $30,000 = $45,000

4-11 $1,500,000 – $200,000 = $1,300,000Simon and Pieman had a net worth of $1,300,000 at the end of the

year.

4-12 a ) 2006 Depreciation Expense for this process line

($131,000 + $12,000) X (0.245) = $35,035

b ) Amount of tax savings due to this investment.

$35,035 X 0.4 = $14,014

4-13.Operating Income (EBIT) = $768,000

27

+ Depreciation = $42,000+ Amortization = $15,000

$825,000

Target Telecom’s EBITDA = $825,000.

4-14 a ) The company's 2006 taxable income = ($400,000 - $130,000 X

0.2)

= $374,000

b ) Income tax = $374,000 X 0.34 = $127,160

4-15. a) Earnings = [($600,000 - 50,000) X (1 - .34) - $63,000] =

$300,000

Earnings per share = $300,000 / 100,000 = $3 per share

b) Addition to Retained Earnings = $300,000 - 100,000 = $200,000

4-16. a ) Current Assets: 2005: $5,534 + 14,745 + 10,733 + 952 + 3,234 = $35,198

2006: $9,037 + 15,943 + 11,574 + 1,801 + 2,357=$40,712

b ) Total Assets: 2005: $35,198+(57,340 - 29,080)+1,010+2,503 = $66,971

2006: $40,712+(60,374 - 32,478)+1,007+4,743 = $74,358

c ) Current Liabilities: 2005: $3,253 + 6,821 = $10,0742006: $2,450 + 7,330 = $9,780

d ) Total Liabilities: 2005: $10,074 + 2,389 = $12,4632006: $9,780 + 2,112 = $11,892

e ) Total Stockholders' Equity: 2005: $8,549 + 45,959 = $54,5082006: $10,879 + 51,587 = $62,466

28

4-17. 2005: $12,463 TL + $54,508 EQ = $66,971 TA

2006: $11,892 TL + $62,466 EQ = $74,358 TA

4-18. (Dollars)a ) Accumulated Depreciation 3,398 Inflowb ) Accounts Receivable (net) 1,198 Outflowc ) Inventories 841 Outflowd ) Prepaid Expenses 877 Inflowe ) Accounts Payable 803 Outflowf ) Accrued Expenses 509 Inflowg ) Plant and Equipment (gross) 3,034 Outflowh ) Marketable Securities 849 Outflow i ) Land 3 Inflowj ) Long Term Investments 2,240 Outflowk ) Common Stock 2,330 Inflowl ) Bonds Payable 277 Outflow

4-19. Pinewood Company and SubsidiariesStatement of Cash Flows

For the year 2006

Operations: Net Income 10,628Add: Depreciation Exp. 3,398

Decrease in Prepaid Expenses 877Increase in Accrued Expenses 509

Less:Increase in A/C Receivable (1,198)Increase in Marketable Securities ( 849)Increase in Inventories ( 841)Decrease in A/C Payable ( 803)

Total Cash Flow from Operations $11,721Investments:

Add: Decrease in Land 3Less:Increase in Plant and Equipment (3,034)

Increase in Long Term Investment (2,240)Total Cash Flow from Investments ($5,271)Financing:

Add: Increase in Common Stock 2,330Less:Common Stock Dividends (5,000)

29

Decrease in Bonds Payable ( 277)Cash Flow from Financing ($2,947)

Net Cash Flow $3,503

4-20. $3,503 = $9,037 end of ‘02 cash - $5,534 end of ‘01 cash Yes, the net cash flow figure from problem #16 gives the same answer as calculating the change in the cash figures from the end of 2005 to the end of 2006 balance sheets.

4-21. Sales 900,000COGS 300,000Gross Profit 600,000Operating Expenses 200,000

Operating Income (EBIT) 400,000Interest Expense 100,000Income before taxes (EBT) 300,000Tax Expense (30%) 90,000Net Income $210,000

4-22.Retained Earnings end of 2006$8,700,000

Retained Earnings end of 20058,000,000

Addition to retained earnings 2006700,000

Earnings Available to Common Stockholders $1,500,000-Addition to Retained Earnings -

700,000Dividends paid to Common Stockholders 2006 = $ 800,000

4-23.Year Deprec. % * Depreciable Base = Depreciation1 10% $385,000 $38,5002 18% $385,000 $69,3003 14.4% $385,000 $55,4404 11.5% $385,000 $44,2755 9.2% $385,000 $35,4206 7.4% $385,000 $28,490

30

7 6.6% $385,000 $25,4108 6.6% $385,000 $25,4109 6.5% $385,000 $25,02510 6.5% $385,000 $25,02511 3.3% $385,000 $12,705

4-24.Basis = $1,000,000 + $100,000 + $50,000 = $1,150,000Year 3 depreciation = $1,150,000 * .148 = $170,200

4-25.Year 1 $7,000,000 * .1 = $700,000Year 2 $7,000,000 * .18 = $1,260,000Year 3 $7,000,000 * .144 = $1,008,000Year 4 $7,000,000 * .115 = $805,000Year 5 $7,000,000 * .092 = $644,000Year 6 $7,000,000 *.074 = $518,000Year 7 $7,000,000 * .066 = $462,000Year 8 $7,000,000 * .066 = $462,000Year 9 $7,000,000 * .065 = $455,000Year 10 $7,000,000 * .065 = $455,000Year 11 $7,000,000 * .033 = $231,000

31

Chapter 5 Solutions

Answers to Review Questions

1. What is a financial ratio?

A financial ratio is a number that expresses the value of one financial variable relative to another. Put more simply, a financial ratio is the result you get when you divide one financialnumber by another. Calculating an individual ratio is simple, but each ratio must be analyzed carefully to effectively measure a firm's performance.

2. Why do analysts calculate financial ratios?

Ratios are comparative measures. Because the ratios show relative value, they allow financial analysts to compare information that could not be compared in its raw form. For example, ratios may be used to compare one ratio to a related ratio, a firm's performance to management's goals, a firm's past and present performance, or a firm's performance to similar firms

3. Which ratios would a banker be most interested in when considering whether to approve an application for a short-term business loan? Explain.

Bankers and other lenders use liquidity ratios to see whether to extend short-term credit to a firm. Liquidity ratios measure the ability of a firm to meet its short-term obligations. These ratiosare important because failure to pay such obligations can lead to bankruptcy. Generally, the higher the liquidity ratio, the more able a firm is to pay its short-term obligations.

4. Which ratios would a potential long-term bond investor be most interested in? Explain.

32

Current and potential lenders of long-term funds, such as banks andbondholders, are interested in debt ratios. When a business's debt ratios increase significantly, bondholder and lender risk increasesbecause more creditors compete for that firm's resources if the company runs into financial trouble.

5. Under what circumstances would market to book value ratios be misleading? Explain.

The Market to Book ratio is useful, but it is only a rough approximation of how liquidation and going concern values compare. This is because the Market to Book ratio uses accounting-based bookvalues. The actual liquidation value of a firm is likely to be different than the book value. For instance, the assets of a firm may be worth more or less than the value at which they are currently carried on the company's balance sheet. In addition, thecurrent market price of the company's bonds and preferred stock mayalso differ from the accounting value of these claims.

6. Why would an analyst use the Modified Du Pont system to calculate ROE when ROE may be calculated more simply? Explain.

Actually, an analyst would not use the Modified Du Pont equation to calculate ROE for precisely the reason stated above. What an analyst would use the Modified Du Pont equation for is to help analyze the factors that contribute to a firm's ROE. In other words, analysts use the Modified Du Pont system to “take apart” ROEto see what factors are influencing it.

7. Why are trend analysis and industry comparison important to financial ratio analysis?

Trend analysis helps financial managers and analysts see whether a company's current financial situation is improving or deteriorating.

Cross-sectional analysis, or industry comparison, allows analysts to put the value of a firm's ratios in the context of its industry.

33

Answers to End-of-Chapter Problems

5-1. a) Gross Profit Margin = Gross Profit/Sales 20,000,000/35,000,000 = .5714 Gross Profit margin = 57.14%

b) Operating Profit Margin = EBIT/Sales 16,000,000/35,000,000 = .4571 Operating Profit Margin = 45.71%

c) Net Profit Margin = Net Income/Sales 8,100,000/35,000,000 = .2314 Net Profit Margin = 23.14%

5-2. Current Ratio = Total Current Assets/Total Current Liabilities(5,000) / (500 +850 + 600) = 2.56Current Ratio = 2.56

Quick Ratio = (Total Current Assets - Inventory)/Total CurrentLiabilities

(5,000 – 900)/(500 + 850 + 600) = 2.10Quick Ratio = 2.10

5-3. Average Daily Credit Sales = Annual credit sales/3655,000,000/365 = $13,698.63

Average Collection Period = Accounts Receivable/Average DailyCredit Sales

$500,000/13,698.63 = 36.5Average Collection Period = 36.5 days

5-4. Inventory Turnover = Sales/Inventory35,000,000/2,400,000 = 14.58Inventory Turnover = 14.58 X

Total Asset Turnover = Sales/Total Assets35,000,000/(15,000,000 + 20,000,000) = 1Total Asset Turnover = 1 X

34

5-5. a) Book value per shareBook price per share = Common Stock Equity/Number of shares

Outstanding$4,500,000/650,000 =$6.92BPS = $6.92

b) Market to book value ratio Market to book value ratio = Market price per share/Bookvalue per share $25.00/$6.92 = 3.61 Market to book value ratio = 3.61

5-6. a) Gross profit margin: $47,378/$94,001 = 50.40%b) Operating profit margin $12,941/$94,001 = 13.77%c) Net profit margin $8,620/$94,001 = 9.17%d) Return on assets $8,620/$66,971 = 12.87%e) Return on equity $8,620/$54,508 = 15.81%

While the Net profit margin is higher than the industry average, the Return on assets is lower. Pinewood may consider increasing its debtto leverage profits.

5-7. a) Current assets = $5,534 + $14,745 + $10,733 + $952 + $3,234 = $35,198

Current ratio = $35,198/$10,074 = 3.494b) Quick ratio = ($35,198 - $10,733)/$10,074 = 2.429Pinewood seems highly capable of paying off short-term debts.

5-8. a) Total debt = $3,253 + $6,821 + $2,389 = $12,463 Debt to total assets = $12,463/$66,971 = 18.61%b) Times interest earned = $12,941/$48 = 270 times

Yes. The Pinewood has very low debt and its earnings are extremelyhigh compared to its interest expense.

35

5-9. a. Average collection period $14,745/($94,001 / 365) =57.25 days

b. Inventory turnover $94,001/$10,733 = 8.76c. Total asset turnover $94,001/$66,971 = 1.404

We would need to know the industry averages for these figures, and also know about Pinewood’s credit and inventory management practices to comment meaningfully on the above figures.

5-10. Modified Du Pont: ROE = Net Profit Margin X Total Asset Turnover XAssets over Equity

= 0.0917 X 1.404 X $66,971/$54,508 = 15.82%

5-11.a) EVA = EBIT (1- tax rate) – (invested capital * investor’s required rate of return)

EVA = $12,941,000 * (1 - 0.35) – ($77,389,000 * 0.10) = $672,750

b) Pinewood has a true economic profit of $672,750. This is the amount by which its earnings exceed the returned expected by the firm’s investors.

c) MVA = Total market value – invested capital MVA = ($75,000,000 + $2,389,000) – ($54,508,000 + $2,389,000) =

$20,492,000

d) Pinewood has a total market value that is $20,492,000 greater that the amount of capital invested in the firm.

5-12.a) EVA = EBIT (1 – Tax Rate) – (invested capital * investors required rate of return) EVA = $8,000 (.65) – ($33,000 * .12) = $5,200 – $3,960 EVA = $1,240

b) The economic value is positive; therefore, Eversharp earned asufficient amount during the year to provide more thanthe expected rate of return from the investors and lenders who

contributed to the capital of the company.

36

c) MVA = Total market value – invested capital MVA = $33,000 - $21,000 = $12,000

d) Eversharp’s total market value exceeds its invested capital by$12,000.

5-13.EVA & MVA Calculation:

Income tax rate 35% Cost of Capital 12% Ka Stock Price (ref) $9 Number of shares outstanding (ref)

3,000

Market Value of Common Equity (ref)

$27,000

Book Value of Common Equity

$15,210

Debt Capital (ref) $6,630 (Notes payable + Long-TermDebt )

Total Invested Capital (ref)

$33,630 (Debt + Common)

EVA MVA

a. EVA $189 EBIT(1-Tr) - (Invested Capital * Ka)

b. Comment on EVA: This year T & J earned enough toexceed the return expected by the

contributors of the firm's capital by $189.

5-14.a. Du Pont: ROA = Net Profit Margin X Total Asset Turnover

= (80/1,000) X (1,000/500) = 16% Modified Du Pont: ROE = Net Profit Margin X Total Asset

Turnover X Assets over Equity = ($80/$1,000) X ($1,000/$500) X (1/(1-0.5) =

32%

b. ROE = ($80/$1,000) X ($1,000/$500) X (1/(1-0.7) = 53.3%

c. ROE = ($80/$1,000) X ($1,000/$500) X (1/(1-0.9) = 160%

37

d. ROE = ($80/$1,000) X ($1,000/$500) X (1/(1-0.1) = 17.78%

5-15. Assets Liabilities + Equity

Cash $6,000 Accounts Payable $6,000Accounts Receivable 15,068 Notes Payable 2,739Inventory 6,667 Accrued Expenses 600Prepaid Expenses 282 Total Current Liabilities 9,339Total Current Assets 28,017 Bonds Payable 15,661Fixed Assets 34,483 Common Stock 16,000

Retained Earnings 21,500Total Assets $62,500 Total Liabilities + Equity

$62,500

Total Assets = Sales / Total Asset Turnover = $100,000/1.6 = $62,500Fixed Assets = Sales / Fixed Asset Turnover = $100,000/2.9 = $34,483Total Current Assets = $62,500 - $34,483 = $28,017Accounts Receivable = Sales/day X Ave. Collection Period = ($100,000/365)X 55 = $15,068Inventory = Sales / Inventory Turnover = $100,000/15 = $6,667Prepaid Expenses = $28,017 - ($15,068 + $6,667 + $6,000) = $282Total Debt = Total Assets X Debt to Asset Ratio = $62,500 X 0.4 = $25,000Total Current Liabilities = Total Current Assets / Current Ratio = $28,017/3 = $9,339Bonds Payable = Total Debt - Total Current Liabilities = $25,000 - $9,339 = $15,661Retained Earnings = $62,500 - ($16,000 + $25,000) = $21,500Notes Payable = $9,339 - ($600 + $6,000) = $2,739

5-16.NI/$5,000 = 0.10NI = $500TE = TA - TL = $10,000 - $6,000 = $4,000ROE = $500/$4,000 = .125 = 12.5%

38

5-17. Current Liability = $20,000 - $18,000 = $2,000Current Ratio = $5,000/$2,000 = 2.5 times

5-18. Return on Assets = Net Profit Margin X Total Asset Turnover0.12 = 0.04 X Total Asset TurnoverTotal Asset Turnover = 0.12/0.04 = 3

5-19. Gross Profit = 0.50 X $5,000,000 = $2,500,000

5-20.EBIT = $2,500,000 - $200,000 - $50,000 = $2,250,000Operating Profit Margin = $2,250,000/$5,000,000 = .45 = 45%

5-21.Net Income = 0.20 X $5,000,000 = $1,000,000

5-22.Net Income = 0.20 X $5,000,000 = $1,000,000ROA = $1,000,000/$20,000,000 = .05 = 5%

5-23.Net Income = 0.10 X $15,000,000 = $1,500,000

5-24.Current Ratio = (20,000,000 - 2,000,000)/4,000,000 = 4.5

5-25.Quick Ratio = ($20,000,000 - $2,000,000 - $3,000,000)/$4,000,000 = 3.75 times

5-26.Total Debt = 0.30 X $20,000,000 = $6,000,000Debt to Equity ratio = $6,000,000/$14,000,000 = 0.43

5-27. Inventory Turnover = 5,000,000/3,000,000 = 1.67

5-28.Return on Assets = 0.20 X 0.25 = 0.05 = 5%

5-29. a) Du Pont: ROA = Net Profit Margin X Total Asset Turnover

39

= ($200/$2,000) X ($2,000/$1,000) = .20 = 20%Modified Du Pont: ROE = Net Profit Margin X Total Asset Turnover X Assets over Equity

= ($200/$2,000) X ($2,000/$1,000) X (1/(1-0.6)) = .50 = 50%

b) ROE = ($200/$2,000) X ($2,000/$1,000) X (1/(1-0.8)) = 100%

c) ROE = ($200/$2,000) X ($2,000/$1,000) X (1/(1-0.2)) = 25%

5-30. Notoriously Niagara Niagara’s Notions

a) NPM = $100,000/$500,000 = 0.20 NPM = $10,000/$500,000 = 0.02

b) TATO = $500,000/$500,0000 = 0.10 TATO = $500,000/$500,000 =1.0

c) ROA = 0.20 X 0.10 = 0.02 ROA = 0.02 X 1.0 = 0.02d) Notoriously Niagara must have a higher net profit margin because their asset turnover is low compared to that of Niagara’s Notions even though they have the same ROA. Niagra’s Notions has a high asset turnover but a low net profit margin.

5-31.a ) $2,250,000/1,750,000=$1.29 b ) $40/$1.29 = 31 c ) $15,000,000/1,750,000 = $8.57 d ) $40/$8.57 = 4.67 e ) Yes, the market seems to believe that the company has going-

concern value as evidenced by the market to book ratio greaterthan 1.

5-32. Net Profit Margin Current Ratio Total Asset Turnover

Year NI/Sales CA/CL Sales/TA

2004 10.00% .94 1.052005 9.44% 1.02 1.15

40

2006 9.36% 1.08 1.18

Golden ProductsIndustry averages: 9.42% 1.13 2.00

The NPM is about average, although it is deteriorating. The liquidity, as measured by the current ratio, is below average but improving. Asset utilization, as measured by the total asset turnover isway below average.

5-33. The Industry averages are:

Fixed Asset Turnover Return on Assets Debt to Assets RatioReturn on equity

1.33 11.00% 0.60 26%

YEAR PM CR TATO FATO ROA D/A ROE2004 10.00% 0.94 1.05 1.21 10.53% 0.68 33.33%2005 9.44% 1.02 1.15 1.33 10.90% 0.64 30.36%2006 9.36% 1.08 1.18 1.36 11.00% 0.60 27.50%

Golden Products has an improving ROA that now equals that of the industry norm. The ROE has slipped a little, but is still above the industry norm in spite of the fact that Golden has a little less debt in its capital structure in 2006. Overall, Johnny shouldbe pleased.

5-34. ( Figures in $ '000) Mining Smelting Rolling Extrusion Whole Company

NPM 3.3% 8.7% 11.7% 10.0% 9.7%

ROA 4.2% 10.4% 17.9% 13.9% 13.4%

41

5-35. National Glass Company

Income Statement (in $ 000's) Ratios:2006

ACP 48.7 days

Sales $45,000 Inventory Turnover

9 X

Cost of Goods Sold 23,000 Debt to Assets 40%Gross Profit 22,000 Current Ratio 1.625

0 Selling and Admin Expenses

13,000 Total Asset turnover

1.50

Depreciation 3,000 Fixed Asset Turnover

2.6471

Operating Income 6,000 Return on Equity 19.33%

Interest Expense 200 Return on Assets 11.6%Earnings Before Tax 5,800 Operating Profit

Margin13.33

%Income Taxes 2,320 Gross Profit

Margin48.89

%Net Income $3,480

Preferred Dividends $0 Earnings Available to Common

$3,480

Balance Sheet (in $000's)

As of Dec 31

2006Assets

Current Assets: Cash $2,000 Accounts Receivable 6,000 Inventory 5,000 Total Current Assets 13,000 Plant & Equipment, Net 16,000 Land 1,000 Total Assets $30,000 Liabilities & Equity

Current Liabilities: Accounts Payable $2,000 Notes Payable 3,000 Accrued Expenses 3,000 Total Current Liabilities

8,000

Bonds Payable 4,000 Total Liabilities 12,000 Common Stock 4,000

42

Retained Earnings 14,000 Total Stockholders' Equity

18,000

Total Liabilities & Equity

$30,000

5-36. a.) (Industry) Kingston, 2006 Kingston, 2007

i. Gross Profit Margin (50%) 48.9% 48.9%ii. Operating Profit Margin (15%) 15.1%13.3%iii. Net Profit Margin (8%) 8.5% 7.5%iv. Return on Assets (10%) 11.56%9.97%v. Return on Equity (20%) 19.3% 16.3%vi. Current Ratio (1.5) 1.63 1.62vii. Quick Ratio (1.0) 1.00 1.04viii.Debt to Total Asset (0.5) .4 .39ix. Times Interest Earned (25) 15.5X14.6Xx. Average Collection Period (45 days) 53.5days61.6daysxi. Inventory Turnover (8) 8.18X 8.62Xxii. Total Asset Turnover (1.6) 1.4X1.3X

b.) Kingston has about the same net profit margin and return on equity as the industry norm. The return on assets ratio for Kingston is aboutthe same as than the industry norm.

c.) Determine the sources and uses of funds and prepare a statement of cash flows for 2007.

(1) Sources and Uses of Funds:

Change,2006 to2007BalanceSheet

Sources Uses

43

Net Income $3,353 Dividends paid $733 Depreciation $3,000 Cash ($200) $200

Accounts Receivable, Net $1,600 $1,60

0 Inventory $220 $220 Property, Plant & Equipment, Gross $5,000

$5,000

Land $0 Accounts Payable $600 $600 Notes Payable $300 $300 Accrued expenses $100 $100 Bonds Payable $0 Common Stock $0

Totals $7,553 $7,55

3

(2) Statement of Cash Flows:

Kingston Tool CompanyStatement of Cash Flows for the year 2007

( in $ 000s)

Cash Flows from Operations:

Net Income $3,353 Depreciation 3,000

Decrease(Increase) in AccountsReceivable

(1,600)

Decrease(Increase) in Inventory (220)Increase(Decrease) in Accounts

Payable 600 Increase(Decrease) in Notes

Payable 300 Increase(Decrease) in Accrued

Expenses 100 Total Cash Flows from

Operations $5,533 Cash Flows from Investments:

New Property, Plant, & Equipment($5,00

0)Total Cash Flows from

Investments($5,00

0)Cash Flows from Financing:

Dividends Paid ($733)Total Cash Flows from

Financing (733)-

44

Net Cash Flow ($200)

Beginning Cash Balance $2,000 Ending Cash Balance $1,800

d.) Profit margins are eroding and generally a little below the industry norm. Liquidity is about average. Debt is low, but interestcoverage is below the industry norm in spite of the low debt load. Inventory turnover is way below average. The negative cash flow of $200,000 came mainly from the buildup of accounts receivable and plant& equipment.

e.) The current ratio, quick ratio, and times interest earned would getthe most scrutiny from loan officers.

5-36b. EVA = EBIT * (1 – tax rate) – (invested capital * investor’srequired rate of return)

EVA = ($4,000 * 0.60) – ($60,000 * 0.10) = -$3,600EVA = -$3,600

MVA = Total market value – invested capitalMVA = $50,000 - $60,000 = -$10,000MVA = -$10,000

5.37.a) Accounts Receivable/Average Daily Credit Sales $564,000.00 / ($3,814,000 / 365)= 53.71 = 54 days

b) Super Dot Com was more profitable in 2006 than it was in 2004.

2004 2006_________ $519,000/$2,100,000$1,115,000/$3,814,000Net Profit Margin 24.71% 29.23%

$519,000/$2,859,000 $1,115,000/$5,316,000Return on Assets 18.15% 20.97%

Both the NPM and ROA ratios were better in 2006.

45

c) Super Dot Com was less liquid at the end of 2006 than it was at the end of 2004.

2004 2006_________ $981,000/$245,000 $1,720,000/$623,000Current Ratio 4.00 2.76

($981,000 - $307,000)/$245,000 ($1,720,000 - $960,000)/$623,000

Quick Ratio 2.75 1.22

46

Chapter 6 Solutions

Answers to Review Questions

1. Why do businesses spend time, effort, and money to produce forecasts? Explain.

Businesses succeed or fail depending on how well prepared they are to deal with the situations they confront in the future. Thereforethey expend considerable sums making estimates (forecasts) of what the future situation is likely to be. Businesses develop new products, set production quotas, and select financing sources basedon forecasts about the future economic environment and the firm's condition. If economists predict interest rates will be relativelyhigh, for example, firms may plan to limit borrowing and defer expansion plans.

2. What is the primary assumption behind the experience approach to forecasting?

The experience approach to forecasting is based on the assumption that things will happen a certain way in the future because they happened that way in the past. For instance, if it has always takenyou fifteen minutes to drive to the grocery store, then you will probably assume that it will take you about fifteen minutes the next time you drive to the store. Similarly, financial managers often assume sales, expenses, or earnings will grow at certain rates in the future because they grew at that rate in the past.

3. Describe the sales forecasting process.

Sales forecasting is a group effort. Sales and marketing personnel usually provide assessments of demand and the competition. Production personnel usually provide estimates of manufacturing capacity and other production constraints. Top management will make strategic decisions affecting the firm as a whole. Financial managers coordinate, collect, and analyze the sales forecasting

47

information. Figure 6-1 in the text shows a diagram of the process.

4. Explain how the cash budget and the capital budget relate to pro forma financial statements.

The cash budget shows the projected flow of cash in and out of the firm for specified time periods. The capital budget shows planned expenditures for major asset acquisitions. Forecasters incorporatedata from these budgets into pro forma financial statements under the assumption that the budget figures will, in fact, occur.

5. Explain how management goals are incorporated into pro forma financial statements.

Management sets a target goal, and forecasters produce pro forma financial statements under the assumption that the goal will be reached. For example, if management’s goal is to pay off all short-term notes during the coming year, forecasters would incorporate this into the pro forma balance sheet by setting Notes Payable to zero.

6. Explain the significance of the term additional funds needed.

When the pro forma balance sheet is completed, total assets and total liabilities and equity will rarely match. The discrepancy between forecasted assets and forecasted liabilities and equity results when either too little or too much financing is projected for the amount of asset growth expected. The discrepancy is calledadditional funds needed (AFN) when forecast assets exceed forecast liabilities and equity, and excess financing when forecast liabilities and equity exceed forecast assets.

7. What do financial managers look for when they analyze pro forma financial statements?

After the pro forma financial statements are complete, financial managers analyze the forecast to determine (1) what current trends suggest what will happen to the firm in the future, (2) what effect

48

management's current plans and budgets will have on the firm, and (3) what actions to take to avoid problems revealed in the pro forma statements

8. What action(s) should be taken if analysis of pro forma financial statements reveals positive trends? Negative trends?

When analyzing the pro forma statements, managers often see signs of emerging positive or negative conditions. If forecasters discover positive indicators, they will recommend that current plans be continued. If forecasters see negative indicators, they will recommend corrective action.

Answers to End-of-Chapter Problems

6-1. Sales Record for The Miniver Corporation

Sales in 2007 is expected to be approximately $215,000 following the trend of the last six years as shown above.

6-2. This year Next Year Forecasting Assumption

49

$0

$50,000

$100,000

$150,000

$200,000

$250,000

1997 1998 1999 2000 2001 2002 2003

Sales 100 120 Sales will grow 20%(100 X 1.2)- Variable Costs 50 60 Constant % of sales(120 X 0.5)- Fixed Costs 40 40 Remains same= Net Income 10 20 (120 - 60 - 40)

Dividends 5 10 Keep 50% PayoutRatio(20 X 0.5)

Current Assets 60 72 Constant % of sales(120 X 0.6) Fixed Assets 100 100 Remains same Total Assets 160 172 (100 + 72)

Current Liabs. 20 24 Constant % of sales (120 X 0.2) Long-term Debt 20 20 Remains same Common Stock 20 20 Remains same Retained Earns. 100 110 (100+20-10) Tot Liabs & Eq 160 174

AFN = 172-174= -2 (Negative AFN means there are excess funds.)

6-3. Jolly Joe's Pizza, Inc. Financial Status and Forecast

2006 Est. for 2007 Sales $10,000 20,000 COGS 4,000 8,000 Gross Profit 6,000 12,000 Fixed Expenses 3,000 3,000 Before-Tax Profit 3,000 9,000 Tax @ 33.33% 1,000 3,000 Net Profit $2,000 6,000 Dividends $0 0 Current Assets $25,000 50,000

50

Net Fixed Assets 15,000 15,000 Total Assets $40,000 65,000 Current Liabilities $17,000 34,000 Long-term debt 3,000 3,000 Common Stock 7,000 7,000 Retained Earnings 13,000 19,000 Total Liabs & Eq $40,000 63,000

Joe will need $2,000 in additional funds in 2007 ($65,000 - $63,000).

6-4. Sugar Cane Alley Financial Status and Forecast

2006 Est. for 2007

Sales $90,000 110,000 COGS 48,000 58,667 Gross Profit 42,000 51,333 Selling and marketing expenses 13,000 15,889 General and admini- strative expenses 5,000 5,000 Depreciation Expense 2,000 2,000

Operating Income 22,000 28,444 Interest Expense 800 EBT 27,644 Tax @ 30% 8,293 Net Profit 19,351 Dividends 10,000

Addition to RE 9,351

6-5. a ) Cash .111111 X $110,000 = $12,222Accounts Receivable .024667 X $110,000 = $2,713Inventory .088889 X $110,000 = $9,778

b ) Property and Equipment, gross $25,000

51

Accumulated Depreciation $6,000Property and Equipment, net $19,000Total Assets $19,000 + $12,222 + $2,713 + $9,778 = $43,713

c ) Accounts Payable .015333 X $110,000 = $1,687

d ) Total Liabilities = $8,000 + $1,687 = $9,687

e ) Total Liabilities and Equity = $9,687 + $9,351 + $5,000 + $26,840= $50,878

f ) Total Assets = $12,222 + $2,713 + $9,778 + $19,000 = $43,713AFN = $43,713 - $50,878 = -$7,165There are excess funds of $7,165.

g ) 2006: Net Profit Margin = $14,840/$90,000 = 16.49%2007: Net Profit Margin = $19,351/$110,000 = 17.6%

6-6.

Assets 2006 2007 Liabilities 2006 2007

Cash $10,000 $12,500 Accounts Payable $10,500 $13,125Acct Rec. 25,000 31,250 Notes Payable 10,000 12,500Inventory 20,000 25,000 Accrued Expenses 11,000 13,750Prepaid Exp 2,000 2,500 Long Term Debt 15,000 15,000Total Current Common Equity 38,500 38,500Assets 57,000 71,250 Total LiabilitiesFixed Assets 32,000 32,000 Equity 85,000 $92,875Depreciation 4,000 4,000Total Assets 85,000 $99,250

*Net Sales for 2007 = $150 million * 1.25 = $187.5 million Additional funds needed = $99,250 - $92,875 = $6,375

52

6-7. 2006 2007

Sales 1,000 1,250 Variable Costs 500 562.50 Fixed Costs 160 160 Net Income 340 527.50 Dividends 136 290.13

6-8. Pro Forma Balance SheetsEnd of YearAssets 2006 2007 Liabilities + Equity 2006 2007Cash $4,000 4,400 Accounts Payable $4,400

4,840Accounts Rec 10,000 11,000 Notes Payable 4,000 4,400Inventory 13,000 14,300 Accrued Expenses 5,000

5,500Prepaid Exp 400 440 Tot.Current Liabilities13,400 14,740Current Assets27,400 30,140 Bonds Payable 6,000 6,000Fixed Assets 11,000 11,000 Common Equity 19,000 21,468

Total Assets $38,400 $41,140 Tot.Liab. + Equity $38,400 $42,208

In 2007 there would be $1,068 ($42,208-$41,140) in excess funds. This assumes, as the problem states, that notes payable would increase by 10% along with other current liabilities. Notes payable usually does not increase with sales.

Year Total Sales PBT NIAddition to RE2007 $85,000 X 1.1 $93,500 X .11 $10,285 X .6 $6,171

X .40

53

= $93,500 = $10,285 = $6,171 = $2,468

6-9. Compute the following ratios for 2006 and 2007: 2006 2007Current Ratio 3 3Debt to Assets Ratio 25% 25.3%Sales to Assets Ratio 62.5% 66.27%Net Profit Margin 10% 13.64%Return on Assets 6.25% 9.04%Return on Equity 8.33% 12.10%

Liquidity seems strong and stable. Debt is modest and stable. Asset utilization is improving slightly while all the profit margins calculatedshow marked improvement.

6-10.BRIGHT FUTURE CORPORATION

Historical and Projected Income Statements

Historical Projected2006 2007

Sales $10,000,000

$12,000,000

Cost of goods Sold $4,000,000 $4,800,000

Gross Profit $6,000,000 $7,200,000

Selling & Admin. Expenses $800,000 $960,000 Depreciation Expense $2,000,000 $2,000,00

0 Operating Income (EBIT) $3,200,000 $4,240,00

0 Interest Expenses $1,350,000 $1,350,00

0 Earnings Before Tax (EBT) $1,850,000 $2,890,00

0 Income Tax (40%) $740,000 $1,156,00

0 Net Income (NI) $1,110,000 $1,734,00

0

Common Stock Dividends paid $400,000 $400,000 Addition to Retained earnings $710,000 $1,334,00

54

0 Earnings per Share (1,000,000 shares)

$1.11 $1.73

BRIGHT FUTURE CORPORATIONHistorical and Projected Balance Sheets

Projectionwith AFN

Historical Projected ExcessFinancing

Dec 31,2006

Dec 31,2007

Incorporated

ASSETSCurrent Assets: Cash $9,000,000 $10,800,0

00 $10,800,000

Marketable Securities $8,000,000 $9,600,000

$9,600,000

Accounts Receivable (gross) $1,200,000 $1,440,000

$1,440,000

Less: Allowance for bad Debts

$200,000 $240,000 $240,000

Accounts Receivable (Net) $1,000,000 $1,200,000

$1,200,000

Inventory $20,000,000

$24,000,000

$24,000,000

Prepaid Expenses $1,000,000 $1,200,000

$1,200,000

Total Current Assets $39,000,000

$46,800,000

$46,800,000

Plant and Equipment (gross) $20,000,000

$20,000,000

$20,000,000

Less: Accumulated Depreciation $9,000,000 $11,000,000

$11,000,000

Plant and equipment (net) $11,000,000

$9,000,000

$9,000,000

TOTAL ASSETS $50,000,000

$55,800,000

$55,800,000

LIABILITIES AND EQUITYCurrent Liabilities: Accounts payable $12,000,00

0 $14,400,0

00 $14,400,000

Notes Payable $5,000,000 $5,000,000

$5,000,000

Accrued Expenses $3,000,000 $3,600,000

$3,600,000

Total Current Liabilities $20,000,000

$23,000,000

$23,000,000

L-T Debt (Bonds Payable, 5%, due 2015)

$20,000,000

$20,000,000

$21,466,000

Total Liabilities $40,000,000

$43,000,000

$44,576,000

55

Common Stock (1,000,000 shares, $1 par)

$1,000,000 $1,000,000

$1,000,000

Capital in Excess of Par $4,000,000 $4,000,000

$4,000,000

Retained Earnings $5,000,000 $6,334,000

$6,334,000

Total Equity $10,000,000

$11,224,000

$11,224,000

TOTAL LIABILITIES AND EQUITY $50,000,000

$54,224,000

$55,800,000

Question 2a. Excess Financing (Additional Funds Needed)

$1,466,000

AFN is incorporated in L-T debt. If $1,466,000 of new L-T debt is issuedthe financing need will be met. Other financing sources could be used but we chose new L-T debt in this illustration.

Question 2, Ratios:2006 2007

b. Current Ratio 1.95 2.03

c. Total Asset Turnover

0.20 0.22

Inventory Turnover

0.50 0.50

d. Total Debt toAssets

0.80 0.77

e. Net Profit Margin

11.10%

14.45%

Return on Assets

2.22% 3.11%

Return on Equity

11.10%

15.30%

Question 3, Comments on liquidity, asset productivity, debt management, and profitability:

Liquidity is improving. Debt is high but stable. Inventory and overall asset utilization are stable. The net profit margin appears healthy. The return on assets ratio is much lower than the net profit margin because of the low asset turnover. The return on equity ratio is much higher than the return on assets because of the high debt load.

Question 4, Recommendations:

56

A 20% projected increase in sales is quite impressive. Management shouldprepare now, however, to raise the $1,466,000 that will be needed in 2007to support the necessary new investments if the projected sales increase is to be achieved.

57

Chapter 7 Solutions

Answers to Review Questions

1. What is risk aversion? If common stockholders are risk averse, how do you explain the fact that they often invest in very risky companies?

Risk aversion is the tendency to avoid additional risk. Risk-aversepeople will avoid risk if they can, unless they receive additional compensation for assuming that risk. In finance, the added compensation is a higher expected rate of return.

People are not all are equally risk averse. For example, some people are willing to buy risky stocks, while others are not. The ones that do, however, almost always demand an appropriately high expected rate of return for taking on the additional risk.

2. Explain the risk–return relationship.

The relationship between risk and required rate of return is known as the risk–return relationship. It is a positive relationship because the more risk assumed, the higher the required rate of return most people will demand.

Risk aversion explains the positive risk–return relationship. It explains why risky junk bonds carry a higher market interest rate than essentially risk-free U.S. Treasury bonds.

3. Why is the coefficient of variation often a better risk measure when comparing different projects than the standard deviation?

Whenever we want to compare the risk of investments that have different means, we use the coefficient of variation (CV). The CV represents the standard deviation's percentage of the mean. Because the CV is a ratio, it adjusts for differences in means, while the standard deviation does not. therefore the CV provides a

58

standardized measure of the degree of risk that can be used to compare alternatives.

4. What is the difference between business risk and financial risk?

Business risk refers to the uncertainty a company has with regard to its operating income (also known as earnings before interest andtaxes or EBIT). Business risk is brought on by sales volatility and intensified by the presence of fixed operating costs.

Financial risk is the additional volatility of net income caused bythe presence of interest expense. Firms that have only equity financing have no financial risk because they have no debt on whichto make fixed interest payments. Conversely, firms that operate primarily on borrowed money are exposed to a high degree of financial risk.

5. Why does the riskiness of portfolios have to be looked at differently than the riskiness of individual assets?

The riskiness of portfolios has to be looked at differently than the riskiness of individual assets because the weighted average of the standard deviations of returns of individual assets does not result in the standard deviation of a portfolio containing the assets. There is a reduction in the fluctuations of the returns ofportfolios which is called the diversification effect.

6. What happens to the riskiness of a portfolio if assets with very low correlations (even negative correlations) are combined?

How successfully diversification reduces risk depends on the degreeof correlation between the two variables in question. When assets with very low or negative correlations are combined in portfolios, the riskiness of the portfolios (as measured by the coefficient of variation) is greatly reduced.

7. What does it mean when we say that the correlation coefficient for two variables is -1? What does it mean if this value were zero? What does it mean if it were +1?

59

Correlation is measured by the correlation coefficient, representedby the letter r. The correlation coefficient can take on values between +1.0 (perfect positive correlation) to -1.0 (perfect negative correlation). The closer r is to +1.0, the more the two variables will tend to move with each other at the same time. The closer r is to -1.0, the more the two variables will tend to move opposite each other at the same time. An r value of zero indicatesthat the variables’ values aren't related at all. This is known asstatistical independence.

8. What is nondiversifiable risk? How is it measured?

Unless the returns of one-half the assets in a portfolio are perfectly negatively correlated with the other half—which is extremely unlikely—some risk will remain after assets are combined into a portfolio. The degree of risk that remains is nondiversifiable risk, the part of a portfolio's total risk that can't be eliminated by diversifying.

Nondiversifiable risk is measured by a term called beta (). The ultimate group of diversified assets, the market, has a beta of 1.0. The betas of portfolios, and individual assets, relate their returns to those of the overall stock market. Portfolios with betas higher than 1.0 are relatively more risky than the market. Portfolios with betas less than 1.0 are relatively less risky than the market. (Risk-free portfolios have a beta of zero.)

9. Compare diversifiable and nondiversifiable risk. Which do you thinkis more important to financial managers in business firms?

Diversifiable risk can be dealt with by, of course, diversifying. Nondiversifiable risk is generally compensated for by raising one’srequired rate of return. Both types of risk are important to financial managers.

10. How do risk-averse investors compensate for risk when they take on investment projects?

60

Because of risk aversion, people demand higher rates of return for taking on higher-risk projects.

11. Given that risk-averse investors demand more return for taking on more risk when they invest, how much more return is appropriate for, say, a share of common stock, than is appropriate for a Treasury bill?

Although we know that the risk–return relationship is positive, thequestion of much return is appropriate for a given degree of risk is especially difficult. Unfortunately, no one knows the answer for sure. One well-known model used to calculate the required rateof return of an investment, given its degree of risk, is the Capital Asset Pricing Model (CAPM).

12. Discuss risk from the perspective of the Capital Asset Pricing Model (CAPM).

The Capital Asset Pricing Model, or CAPM, can be used to calculate the appropriate required rate of return for an investment project given its degree of risk as measured by beta (). A project's betarepresents its degree of risk relative to the overall stock market.In the CAPM, when the beta term is multiplied by the market risk premium term, the result is the additional return over the risk-free rate that investors demand from that individual project. High-risk (high-beta) projects have high required rates of return, and low-risk (low-beta) projects have low required rates of return.

Answers to End-of-Chapter Problems

7-1.Cash Flow Probability

Estimate of Occurrence

CF P CF x P CF - mean (CF - mean)2 P x (CF -mean)2

61

$10,000 5.00% $500 ($9,000) $81,000,000 $4,050,000$13,000 10.00% $1,300 ($6,000) $36,000,000 $3,600,000$16,000 20.00% $3,200 ($3,000) $9,000,000 $1,800,000$19,000 30.00% $5,700 $0 $0 $0$22,000 20.00% $4,400 $3,000 $9,000,000 $1,800,000$25,000 10.00% $2,500 $6,000 $36,000,000 $3,600,000$28,000 5.00% $1,400 $9,000 $81,000,000 $4,050,000Sum of (R x P) = mean: $19,000

Sum of P x (CF- mean)2 = variance: $18,900,000 Square root of variance = standard deviation of

the variance: $4,347

Coefficient of Variation = std.dev./mean = 22.88%

7-2.EXPECTED VALUE, STANDARD DEVIATION AND COEFFICIENT OF VARIATION OF OPERATING INCOME

Operating

Sales Variable Fixed Income Prob.

Estimate

Expenses Expenses Estimate of Occurrence

CF P CF x P CF - mean (CF - mean)2 Px(CF - mean2

$500 $250 $250 $0 2.00% $0 ($350) $122,500 $2,450$700 $350 $250 $100 8.00% $8 ($250) $62,500 $5,000

$1,200 $600 $250 $350 80.00% $280 $0 $0 $0$1,700 $850 $250 $600 8.00% $48 $250 $62,500 $5,000$1,900 $950 $250 $700 2.00% $14 $350 $122,500 $2,450

a. Sum of (R x P) = mean: $350Sum of (CF- mean)2 x P= variance $14,900

b. Square root of variance = standard deviation: $122c. Coeff. of Variation = std.dev/mean: 34.88%

d. New expected value, standard deviation, and coefficient of variation based on revised sales forecast:

Operating

Sales Variable Fixed Income Probability

Estimate

Expenses Expenses Estimate of Occurrence

62

CF P CF x P CF - mean (CF - mean)2 Px(CF -mean)2

$500 $250 $250 $0 10.00% $0 ($350) $122,500 $12,250$700 $350 $250 $100 15.00% $15 ($250) $62,500 $9,375

$1,200 $600 $250 $350 50.00% $175 $0 $0 $0$1,700 $850 $250 $600 15.00% $90 $250 $62,500 $9,375$1,900 $950 $250 $700 10.00% $70 $350 $122,500 $12,250

a. Sum of (R x P) = mean: $350Sum of P x (CF - mean)2 = variance: $43,250

b. Square root of variance = standard deviation: $208c. Coeff. of Variation = std. dev./mean: 59.43%

e. Comments: Note how the increased possibilities that sales will be other than $1,200 caused the standard deviation and coefficient of variation of operating income to nearly double.

7-3. Mean:.10(1,000) + .2(5,000) + .45(10,000) + .15(15,000) + .10(20,000) =100 + 1,000 + 4,500 + 2,250 + 2,000Mean = $9,850

Standard Deviation:ơ 2 = .1(1,000 – 9,850)2 + .2(5,000 – 9,850)2 + .45(10,000 – 9,850)2

+ .15(15,000 – 9,850)2 + .1(20,000 – 9,850)2

ơ 2 = 7,832,250 + 4,704,500 + 10,125 + 3,978,375 + 10,302,250 ơ 2 = 26,827,500

ơ = √ 26,827,500 ơ = 5,179.53

Standard deviation = 5,179.537-4.I. EQUITY EDDIE'S COMPANY:Operati

ng

Income Interest Before-Tax Net Probability

Estimate

Expense Income Taxes Income of

Occurrence

CF P CF x P CF - mean (CF -mean)2

Px(CF -mean)2

$100 $0 $100 $28 $72 5.00% $4 ($216) $46,656 $2,333$200 $0 $200 $56 $144 10.00% $14 ($144) $20,736 $2,074$400 $0 $400 $112 $288 70.00% $202 $0 $0 $0$600 $0 $600 $168 $432 10.00% $43 $144 $20,736 $2,074

63

$700 $0 $700 $196 $504 5.00% $25 $216 $46,656 $2,333

a. Sum of (R x P) =mean: $288

Sum of P x (CF - mean)2 = variance: $8,813

b. Square root of variance = standard deviation: $94c. Coeff. of Variation = std. dev./mean: 32.60%

II. BARRY BORROWER'S COMPANY:Operati

ng

Income Interest Before-Tax Net Probability

Estimate

Expense Income Taxes Income of

Occurrence

CF P CF x P CF - mean (CF -mean)2

Px(CF -mean)2

$110 $40 $70 $19.6 $50.4 5.00% $2.52 ($237.60) $56,453.76 $2,822.69$220 $40 $180 $50.4 $129.6 10.00% $12.96 ($158.40) $25,090.56 $2,509.06$440 $40 $400 $112.0 $288.0 70.00% $201.60 $0.00 $0.00 $0.00$660 $40 $620 $173.6 $446.4 10.00% $44.64 $158.40 $25,090.56 $2,509.06$770 $40 $730 $204.4 $525.6 5.00% $26.28 $237.60 $56,453.76 $2,822.69

a. Sum of (R x P) =mean: $288.00

Sum of P x (CF - mean)2 = variance:

$10,663.49

b. Square root of variance = standard deviation= $103.26c. Coeff. of Variation = std. dev./ mean: 35.86%

e. Comments: Note how Barry Borrower's use of debt financing causes his company to have a higher standard deviation and coefficient of variation of net income than Equity Eddie's.

7-5.STANDARD DEVIATION AND COEFFICIENT OF VARIATION OF CASH FLOWS FOR THE GO-RILLA PROJECT

Cash Flow Probability

Estimate ofOccurrence

CF P CF x P CF - mean (CF - mean)2 P x (CF - mean)2

$20,000 1.00% $200 ($6,000) $36,000,000 $360,000$22,000 12.00% $2,640 ($4,000) $16,000,000 $1,920,000$24,000 23.00% $5,520 ($2,000) $4,000,000 $920,000$26,000 28.00% $7,280 $0 $0 $0

64

$28,000 23.00% $6,440 $2,000 $4,000,000 $920,000$30,000 12.00% $3,600 $4,000 $16,000,000 $1,920,000$32,000 1.00% $320 $6,000 $36,000,000 $360,000

Sum of (R x P) = mean: $26,000

Sum of P x (CF - mean)2 = variance: $6,400,000

a. Square root of variance = standard deviation: $2,530

b.Coefficient of Variation = std. dev./mean: 9.73%

c. Comment: Given that the average coefficient of variation of George's other product lines is 12%, we would say that the Go-Rilla project is LESS risky than average

7-6.Effect of Adding Asset B to Existing Portfolio A

Correlation coefficient r between existingportfolio A and new asset B:

0

Amount invested in Portfolio A: $700,000Amount invested in Asset B: $200,000Total value of combined portfolio: $900,000

Weight of existing assets incombined portfolio: 77.8%

Weight of new asset B in combinedportfolio: 22.2%

Expected Return of existing portfolio A: 9.00%

Standard deviation of existing portfolio A: 3.00%

Coefficient of variation of existing portfolio A: 33.33%

Expected Return of new asset B: 12.00%Standard deviation of new asset B: 4.00%Coefficient of variation of new asset B: 33.33%

65

Expected Return of combined portfolio per equation 7-1: 9.67%

Standard deviation of combined portfolio per equation7-5: 2.50%

Coefficient of Variation of combined portfolio: 25.83%

a. Comparison of standard deviations of existing portfolio A and the new combinedportfolio:

Standard deviation of existingportfolio A: 3.00%

Standard deviation of combinedportfolio: 2.50%

a. Comparison of coefficients of variation of existing portfolio A and the new combined portfolio:

Coefficient of variation of existingportfolio A: 33.33%

Coefficient of variation of combinedportfolio: 25.83%

7-7. Coefficient of variation (CV) = Standard Deviation/Mean288/1,200 = .24CVzazzle = 24%

7-8. Total Portfolio = $10,000 Weights: Stock A: 4,000/10,000 = .4 Stock B: 6,000/10,000 = .6

.4(13) + .6(9) = 10.6%Expected Rate of Return = 10.6%

7-9. ơp = √ (0.3)2 * (0.05)2 + (0.7)2 * (0.02)2 + ( 2 * 0.3 * 0.7 * 0.6 * 0.05 * 0.02)

ơp = √0.000673ơp = 0.0259ơp = 2.59%

66

7-10.Effect of Adding PROJ1 to Existing PortfolioExpected Return of existing portfolio: 11.00%

Standard deviation of existing portfolio: 4.00%a. Coefficient of variation of existing portfolio: 36.36%

Expected Return of new PROJ1: 13.00%Standard deviation of new PROJ1: 5.00%

b. Coefficient of variation of new PROJ1: 38.46%

Amount invested in existing portfolio: $820,000Amount invested in PROJ1: $194,000

Total value of combined portfolio: $1,014,000

c. Weight of existing assets in combined portfolio: 80.9%

d. Weight of new PROJ1 in combined portfolio: 19.1%

Correlation coefficient r between existingportfolio and new PROJ1: 0

e. Standard deviation of combined portfolio: 3.37%(lower than existing portfolio)

:Expected Return of combined portfolio per

equation 7-1 11.38%

f.: Coefficient of Variation of combined portfolio 29.63%(lower than existing portfolio)

g. Firm's risk decreases with the addition of PROJ1 to the portfolio

7-11.Required Rate of Return per theCAPM

Risk free rate(kRF) 5.0%

Expected rate of return on the market (km)

15.0%

Beta 1.2

Required rate of return on stock per the CAPM:

17.0%

67

(equation 7-6)

7-12.kl = 4.5 + .5(12.5) = 10.75% ka = 4.5 + 1.0(12.5) = 17% kh = 4.5 + 1.6(12.5) = 24.5%

7-13.Effect on CAPM Required Rate of Return of Adding a New Project

Risk free rate (kRF) 5.0%Expected rate of return on the market (km) 15.0%

Existing firm's Beta 1.5New Project's Beta 0.8

a. Required rate of return on company per the CAPM:

20.0%

b. Required rate of return on new project per the CAPM:

13.0%

Weight of new project in firm's portfolio: 20.0%

Weight of firm's other assets: 80.0%

c. Beta of firm with new project 1.36

7-14.STANDARD DEVIATION AND COEFFICIENT OF VARIATION OF PSC SALES REVENUE

Sales ProbabilityEstimate of Occurrence

CF P CF x P CF - mean (CF - mean)2 P x (CF -mean)2

$800 2.00% $16 ($1,200) $1,440,000 $28,800$1,000 8.00% $80 ($1,000) $1,000,000 $80,000$1,400 20.00% $280 ($600) $360,000 $72,000$2,000 40.00% $800 $0 $0 $0

68

$2,600 20.00% $520 $600 $360,000 $72,000$3,000 8.00% $240 $1,000 $1,000,000 $80,000$3,200 2.00% $64 $1,200 $1,440,000 $28,800

Sum of (R x P) = exp val: $2,000

Sum of P x (CF - mean)2 = variance: $361,600

Square root of variance = standard deviation: $601

Coefficient of Variation = std. dev./mean: 30.07%

7-15.COEFFICIENT OF VARIATION OF PSC'S OPERATING INCOME

Operating

Sales Variable Fixed Income ProbabilityEstimate Expenses Expenses Estimate of Occurrence

CF P CF x P CF - mean (CF -mean)2

Px(CF -mean)2

$800 $480 $0 $320 2.00% $6 ($480) $230,400 $4,608$1,000 $600 $0 $400 8.00% $32 ($400) $160,000 $12,800$1,400 $840 $0 $560 20.00% $112 ($240) $57,600 $11,520$2,000 $1,200 $0 $800 40.00% $320 $0 $0 $0$2,600 $1,560 $0 $1,040 20.00% $208 $240 $57,600 $11,520$3,000 $1,800 $0 $1,200 8.00% $96 $400 $160,000 $12,800$3,200 $1,920 $0 $1,280 2.00% $26 $480 $230,400 $4,608

Sum of (R x P) = mean: $800

Sum of P x (CF - mean)2 = variance: $57,856

Square root of variance = standard deviation: $241

Coefficient of Variation = std. dev./mean: 30.07%

7-16.COEFFICIENT OF VARIATION OF PSC'S OPERATING INCOME

69

Operating

Sales Variable Fixed Income Probability

Estimate

Expenses Expenses Estimate of Occurrence

CF P CF x P CF - mean (CF -mean)2

Px(CF -mean)2

$800 $480 $400 ($80) 2.00% ($2) ($480) $230,400 $4,608$1,000 $600 $400 $0 8.00% $0 ($400) $160,000 $12,800$1,400 $840 $400 $160 20.00% $32 ($240) $57,600 $11,520$2,000 $1,200 $400 $400 40.00% $160 $0 $0 $0$2,600 $1,560 $400 $640 20.00% $128 $240 $57,600 $11,520$3,000 $1,800 $400 $800 8.00% $64 $400 $160,000 $12,800$3,200 $1,920 $400 $880 2.00% $18 $480 $230,400 $4,608

Sum of (R x P) = mean: $400

Sum of P x (CF - mean)2 = variance: $57,856

Square root of variance = standarddeviation: $241

Coefficient of Variation = std. dev./mean: 60.13%

Comment:

Note how the addition of fixed costs caused the coefficient of variation of PSC's operating income to double from what it was in problem 7-10

7-17. MEASURING PSC'S FINANCIAL RISK

I. Expected value, standard deviation, and coefficient of variation of PSC's net income whenno interest expense is present

Sales Variable

Fixed Operating

Interest

Before-Tax

Probability of

Estimate

Expenses

Expenses

Income Expense

Income Taxes

NetIncome

Occurrence

NI P NI X P NI -mean

(NI - mean)2

P X (NI - mean)2

$800 $480 $400 -$80 $0 -$80 -$56 -$24 2% $0 -$144 $20,736 $415$1,000 $600 $400 $0 $0 $0 $0 $0 8% $0 -$120 $14,400 $1,152$1,400 $840 $400 $160 $0 $160 $112 $48 20% $10 -$72 $5,184 $1,037

70

$2,000 $1,200 $400 $400 $0 $400 $280 $120 40% $48 $0 $0 $0$2,600 $1,560 $400 $640 $0 $640 $448 $192 20% $38 $72 $5,184 $1,037$3,000 $1,800 $400 $800 $0 $800 $560 $240 8% $19 $120 $14,400 $1,152$3,200 $1,920 $400 $880 $0 $880 $616 $264 2% $5 $144 $20,736 $415

Sum of (NI X P) = mean $ 120

Sum of P X (CF - mean)2 =variance:

$5,207

Square root of variance = standarddeviation:

$72

Coefficient of Variation = std.dev./mean:

60.1%

II. Expected value, standard deviation, and coefficient of variation of PSC's net income when interest expense is present

Sales Variable

Fixed Operating

Interest

Before-Tax

Probability of

Estimate

ExpensesExpenses

Income Expense

Income Taxes

Net Income

Occurrence

NI P NI X P NI -mean

(NI - mean)2

P X (NI - mean)2

$800 $480 $400 ($80) $60 ($140) ($42)

($98) 2% ($2) ($336) $112,896

$2,258

$1,000 $600 $400 $0 $60 ($60) ($18)

($42) 8% ($3) ($280) $78,400 $6,272

$1,400 $840 $400 $160 $60 $100 $30 $70 20% $14 ($168) $28,224 $5,645 $2,000 $1,200 $400 $400 $60 $340 $102 $238 40% $95 $0 $0 $0 $2,600 $1,560 $400 $640 $60 $580 $174 $406 20% $81 $168 $28,224 $5,645 $3,000 $1,800 $400 $800 $60 $740 $222 $518 8% $41 $280 $78,400 $6,272 $3,200 $1,920 $400 $880 $60 $820 $246 $574 2% $11 $336 $112,89

6 $2,258

Sum of (R X P) = mean=

$ 238

Sum of P X (CF - mean)2 = variance=

$28,349

Square root of variance = standarddeviation =

$168

Coefficient of variation equals std.dev./mean =

70.7%

7-18.I. New coefficient of variation of PSC's operating income:

Operating

Sales Variable

Fixed Income Probability of

Estimate Expenses

Expenses

Estimate

Occurrence

EBIT P EBIT X EBIT (EBIT P X (EBIT -

71

P -mean

- mean)2

mean)2

$800 $480 $250 $70 1% $0.70 ($480)

$230,400

$2,304

$1,000 $600 $250 $150 6% $9.00 ($400)

$160,000

$9,600

$1,400 $840 $250 $310 13% $40.30 ($240)

$57,600

$7,488

$2,000 $1,200 $250 $550 60% $330.00

$0 $0 $0

$2,600 $1,560 $250 $790 13% $102.70

$240 $57,600

$7,488

$3,000 $1,800 $250 $950 6% $57.00 $400 $160,000

$9,600

$3,200 $1,920 $250 $1,030 1% $10.30 $480 $230,400

$2,304

Sum of (EBIT X P) = mean=$550.0

0

Sum of P X (CF - mean)2 =variance =

$38,784

Square root of variance = standarddeviation =

$197

Coefficient of Variation = std.dev./mean =

35.8%

II. New coefficient of variation of PSC's net income when no interest expense is present

Sales Variable

Fixed Operating

Interest Before-Tax

Net Probability of

Estimate Expenses

Expenses

Income Expense Income Taxes Income Occurrence

NI P NIX P

(NI - mean)2

P X (NI - mean)2

$800 $480 $250 $70 $0 $70 $21 $49 1% $0 $112,896 $1,129

$1,000 $600 $250 $150 $0 $150 $45 $105 6% $6 $78,400 $4,704 $1,400 $840 $250 $310 $0 $310 $93 $217 13% $28 $28,224 $3,669 $2,000 $1,200 $250 $550 $0 $550 $165 $385 60% $23

1 $0 $0

$2,600 $1,560 $250 $790 $0 $790 $237 $553 13% $72 $28,224 $3,669 $3,000 $1,800 $250 $950 $0 $950 $285 $665 6% $40 $78,400 $4,704 $3,200 $1,920 $250 $1,030 $0 $1,030 $309 $721 1% $7 $112,89

6 $1,129

Sum of (NI X P) = mean = $385

Sum of P X (CF - mean)2 = variance = $19,004

72

Square root of variance = standarddeviation =

$138

Coefficient of Variation = std.dev./mean =

35.8%

III. New coefficient of variation of PSC's net income when interest expense is present

Sales Variable

Fixed Operating

Interest B-T Net Probability of

Estimate Expenses

Expenses

Income Expense Income Taxes Income Occurrence

NI P NIX P

(NI - mean)2

P X (NI - mean)2

$800 $480 $250 $70 $40 $30 $9 $21 1% $0 $112,896$1,129

$1,000 $600 $250 $150 $40 $110 $33 $77 6% $5 $78,400 $4,704 $1,400 $840 $250 $310 $40 $270 $81 $189 13% $25 $28,224 $3,669 $2,000 $1,200 $250 $550 $40 $510 $153 $357 60% $21

4 $0 $0

$2,600 $1,560 $250 $790 $40 $750 $225 $525 13% $68 $28,224 $3,669 $3,000 $1,800 $250 $950 $40 $910 $273 $637 6% $38 $78,400 $4,704 $3,200 $1,920 $250 $1,030 $40 $990 $297 $693 1% $7 $112,89

6 $1,129

Sum of (NI X P) = mean = $357

Sum of P X (CF - mean)2 =variance =

$19,004

Square root of variance = standarddeviation =

$138

Coefficient of Variation = std.dev./mean =

38.62%

Summary:

Old Coefficient of variation of operatingincome (business risk)

60.13%

New coefficient of variation of operatingincome (business risk)

35.81%

Old difference between the coefficient of variation of net income with andwithout interest expense (financial risk)

-10.6%

New difference between the coefficient of variation of net income with andwithout interest expense (financial risk)

-2.8%

Comments: The effect of PSC's risk reduction measures was to lower business risk substantially, but financial risk increased slightly. Managers must evaluate this trade-off and proceed accordingly.

73

Chapter 8 Solutions

Answers to Review Questions

1. What is the time value of money?

The time value of money means that money you hold in your hand today is worth more than money you expect to receive in the future.Similarly, money you must pay out today is a greater burden than the same amount paid in the future.

2. Why does money have time value?

Positive interest rates indicate that money has time value. When one person lets another borrow money, the first person requires compensation in exchange for reducing current consumption. The person who borrows the money is willing to pay to increase current consumption. The required rate of return on an investment reflectsthe pure time value of money, an adjustment for expected inflation,and any risk premiums present.

3. What is compound interest? Compare compound interest to discounting.

Compound interest occurs when interest is earned on interest and onthe original principal of an investment. Discounting is the inverse of compounding. Compound interest causes the value of a beginning amount to increase at an increasing rate. Discounting causes the present value of a future amount to decrease at an increasing rate.

4. How is present value affected by a change in the discount rate?

Present value is inversely related to the discount rate. In other words, present value moves in the opposite direction of the discount rate. If the discount rate increases, present value

74

decreases. If the discount rate decreases, present value increases.

5. What is an annuity?

An annuity is a series of equal cash flows, spaced evenly over time.

6. Suppose you are planning to make regular contributions in equal payments to an investment fund for your retirement. Which formula would you use to figure out how much your investments will be worthat retirement time, given an assumed rate of return on your investments?

To figure out how much your investments will be worth at retirementtime, given an assumed rate of return on your investments, you would use the future value of an annuity formula:

Future Value of an Annuity Formula

FVA=PMT[ (1+k)n−1k ]

where: FVA = Future Value of an AnnuityPMT = Amount of each annuity payment k = Interest rate per time period n = Number of annuity payments

7. How does continuous compounding benefit an investor?

The effect of increasing the number of compounding periods per yearis to increase the future value of the investment. The more frequently interest is compounded, the greater the future value. The smallest compounding period is used when we do continuous compounding--compounding that occurs every tiny unit of time (the smallest unit of time imaginable).

75

8. If you are doing PVA and FVA problems, what difference does it makeif the annuities are "ordinary annuities" or "annuities due"?

In FVA or a PVA of annuity due problems, annuity payments earning interest one period sooner than in ordinary annuity problems. So, higher FVA and PVA values result with an annuity due. The first payment occurs sooner in the case of a future value of an annuity due. In present value of annuity due problems, each annuity paymentoccurs one period sooner, so the payments are discounted less severely.

9. Which formula would you use to solve for the payment required for acar loan if you know the interest rate, length of the loan, and theborrowed amount? Explain.

To solve for k when the known values are PVA, n, and PMT, start with the present value of an annuity formula, Equation 8-3b, as follows:

Present Value of an Annuity Formula, Table Method

PVA = PMT(PVIFA k, n)

Next, rearrange terms and solve for (PVIFA k, n) as follows

PVA / PMT = (PVIFA k, n)

Now refer to the PVIFA values in the text, Table IV. You know n, so find the n row corresponding to the number of periods in your problem on the left hand side of the table. You have also determined the PVIFA, so move across the n row until you find (or come close to) the value of PVIFA that you have solved for. The percent column in which the value is located is the interest rate.

Answers to End-of-Chapter Problems

8-1. $1,000 X (1 + 0.07)5 = $1,402.55

76

8-2. a) 0% $50,000 X (1 + 0.00)10 = $50,000.00b) 5% $50,000 X (1 + 0.05)10 = $81,444.73c) 10% $50,000 X (1 + 0.10)10 = $129,687.12d) 20% $50,000 X (1 + 0.20)10 = $309,586.82

8-3. $5,000 * (1 + 0.08)10 = $10,794.62

8-4. a) 3% $100,000 * (1 + 0.03)15 = $155,796.74b) 6% $100,000 * (1 + 0.06)15 = $239,655.82c) 9% $100,000 * (1 + 0.09)15 = $364,248.25d) 12% $100,000 * (1 + 0.12)15 = $547,356.58

8-5 a) 50,000 $50,000 * (1 + 0.10)25 = $541,735.30b) 75,000 $75,000 * (1 + 0.10)25 = $812,602.95

c) 100,000 $100,000 * (1 + 0.10)25 = $1,083,470.59d) 125,000 $125,000 * (1 + 0.10)25 = $1,354,338.24

8-6 a) 5 years $60,000 * (1 + 0.12)5 = $105,740.50b) 10 years $60,000 * (1 + 0.12)10 = $186,350.89c) 15 years $60,000 * (1 + 0.12)15 = $328,413.95d) 20 years $60,000 * (1 + 0.12)20 = $578,777.59

8-7. PV = $20,000 X [1/(1 + .12)10] = $6,439.46

8-8. a) 0% $60,000 X [1/(1+0.00)20] = $60,000.00b) 5% $60,000 X [1/(1+0.05)20] = $22,613.37c) 10% $60,000 X [1/(1+0.10)20] = $8,918.62d) 20% $60,000 X [1/(1+0.20)20] = $1,565.04

8-9 $9,000 * [1/(1+0.08)4] = $6,615.27

8-10 a) 3% $25,000 * [1/(1 + 0.03)10] = $18,602.35b) 6% $25,000 * [1/(1 + 0.06)10] = $13,959.87c) 9% $25,000 * [1/(1 + 0.09)10] = $10,560.27

77

d) 12% $25,000 * [1/(1 + 0.12)10] = $8,049.33

8-11.a)$50,000 $50,000 * [1/(1 + 0.06)15 = $20,863.25b)$75,000 $75,000 * [1/(1 + 0.06)15 = $31,294.88c)$100,000 $100,000 * [1/(1 + 0.06)15 = $41,726.51d)$125,000 $125,000 * [1/(1 + 0.06)15 = $52,158.13

8-12.a) 5 years $80,000 * [1/(1 + 0.09)5] = $51,994.51b) 10 years $80,000 * [1/(1 + 0.09)10] = $33,792.86c) 15 years $80,000 * [1/(1 + 0.09)15] = $21,963.04d) 20 years $80,000 * [1/(1 + 0.09)20] = $14,274.47

8-13. PVA = $500 X [(1-1/1.0610)/0.06] = $3,680.04

8-14. a) 0% $10,000 X 30 = $300,000b) 10% $10,000 X [(1-1/1.1030)/0.10] = $94,269.14c) 20% $10,000 X [(1-1/1.2030)/0.20] = $49,789.36d) 50% $10,000 X [(1-1/1.5030)/0.50] = $19,999.90

8-15.$20,000 * [(1-1/1.0710)/0.07] = $140,471.63

8-16.a) 9 % $10,000 * [(1-1/1.095)/0.09] = $38,896.51b) 13% $10,000 * [(1-1/1.135)/0.13] = $35,173.31c) 15% $10,000 * [(1-1/1.155)/0.15] = $33,521.55d) 21% $10,000 * [(1-1/1.215)/0.21] = $29,259.84

8-17.FVA = $500 X [(1.095-1)/.09] = $2,992.36

8-18. a) 0% $6,000 X 12 = $72,000.00b) 2% $6,000 X [(1.0212-1)/0.02] = $80,472.54c) 10% $6,000 X [(1.1012-1)/0.10] = $128,305.70d) 20% $6,000 X [(1.2012-1)/0.20] = $237,483.01

8-19.$5,000 * [(1.0610 – 1)/0.06] = $65,903.97

78

8-20.$5,000 * [(1.118 – 1)/0.11] = $59,297.17

8-21.a) $1,000 $1,000 * [(1.105 – 1)/0.10] = $6,105.10b) $10,000 $10,000 * [(1.105 – 1)/0.10] = $61,051.00c) $75,000 $75,000 * [(1.105 – 1)/0.10] = $457,882.50d) $125,000 $125,000 * [(1.105 – 1)/0.10] = $763,137.50

8-22.$1,200 X [(1.1240 – 1)/.12] X 1.12 = $1,030,970.87

8-23.$500 * [(1.085 – 1)/0.08] * 1.08 = $3,167.96

8-24.$56,370.93 * 1.06 = $59,753.19

8-25.$80 X [(1-1/1.1220)/.12] X 1.12 = $669.26

8-26.$30,000 * [1-1/1.0925)/0.09] * 1.09 = $321,198.35

8-27.$1,300 * [1-1/1.00583333180)/0.00583333] * 1.00583 = $144,632.74

8-28. $185,361 = FVIF10,k% X $50,000FVIF10,k% = 3.7072; from Table I, k = 14%

8-29. $1,000 X (1 + k)5 = $773.78(1 + k)5 = $773.78/$1,000(1 + k)5 = .77378 1 + k = .95k = -.05 = -5%

8-30.$50,000 * (1 + k)10 = $246,795(1 + k)10 = $246,795/$50,000(1 + k)10 = 4.9359

79

1 + k = 1.173104k = .1731 = 17.31%

8-31.PV = $50/0.08 = $625

8-32.$80/0.09 = $888.89

8-33.$65/0.085 = $764.71

8-34. FV = $10 X (1.08)200 = $48,389,496

8-35.PVA = PMT X PVIFA k,n

$24,000 = $4,247.62 X PVIFA k,10

5.6502 = PVIFA k,10

k = 12%

8-36 PVA = PMT X PVIFA k,n

$200,000 = $1,330.61 X PVIFA k,360

150.3070 = PVIFA k,360

k = .5833% per month X 12 = 7% annual rate

8-37. a ) 5 years? 10,000/(1+.07)5 = 7130b ) 10 years? 10,000/(1+.07)10 = 5083c ) 20 years? 10,000/(1+.07)20 = 2584

8-38. PV = $16,850.58 X [1/(1+.11)5]PV = $10,000

8-39. a ) FV = $20,000 X (1 + .05)7 = $28,142.01 b ) FV = $20,000 X (1 + .07)10 = $39,343.03

8-40. $55.00 = $67.73 X [PVIFk%, 12 years ]

80

.8120 = PVIFk%, 12 years; k = 1.75%

8-41. 1,000 = 2653.3 X [PVIF5%, ?]; .3769 = PVIF5%,?; ? = 20 semi-annual periods, so it will take 10

years.

8-42. PV = 20,000 X [(1-1/1.0615)/0.06] = $194,244.98

8-43.$4,000 * [(1.0920 –1)/0.09] * 1.09 = $223,058.12

8-44.$100 * [(1.0220 –1)/0.02] = $2,429.74

8-45. $2,000 X [((1+.08)10 - 1)/.08] = $2,000 X 14.4866 = $28,973

8-46.a ) $300 X [((1+.02)120 - 1)/.02] = $146,477 b ) $146,477 = $6,000 X [PVIFA2%, n quarters ] PVIFA2%, n quarters = 24.4128; n = 34 quarters or 8.5 years

8-47. $30,000 = PMT X [(1-1/(1+0.1)7)/0.1];$30,000 = PMT X 4.86841882; PMT = $6,162.16

8-48. $10,000/.12 = $83,333.33

8-49. FV = $500 X e.05 x 23 = $1,579.10

8-50.FVIF k=8%, n=? = 2n = 9 years

8-51.PVA = PMT X PVIFA k,n

81

$4,000 = $200 X PVIFA k=.195/12, n=?

20 = PVIFA k=.01625, n=?

n = 24.39 months

8-52. $14,568.50 = $5,000 X [PVIFAk%,4 years], assuming payments start one year from the date of borrowing[PVIFAk%,4 years] = 2.9137; k = 14%

8-53. a) FVA = $1,000 X [[(1+.02)60 - 1]/.02] = $114,051.54 b) $114,051.54 = $6,000 X PVIFA.02, n quarters

PVIFA.02, n quarters = 19.00859; n = 24 quarters = 6 years

8-54. Option 1) PV = $5,650Option 2) PV = $6,750 X [1/1.028] = $5,761.06Option 3) PV = $800 X [(1-(1/(1+.02)8)/.02] = $5,860.39Option 4) PV = $1,000 + $5,250 X (1/(1+.02)8) = $5,480.82Option 4) is the one with lowest cost to Jack.

8-55.n = 30 X 12 = 360k = .09/12 =0 .0075 or 0.75%$120,000 = PMT X [(1-1/1.0075360)/0.0075]PMT = $120,000/124.28186568 = $965.55

8-56.PVA = PMT [(1-1/1.005 180)/.005]$250,000 = PMT X 118.5035147PMT = $2,109.64

8-57.a) n = 4 X 12 = 48 k = .06/12 =0 .005 or 0.5% $18,000 = PMT X [(1-1/1.00548)/0.005] PMT = $18,000/42.58031778 = $422.73

b) n = 6 X 12 = 72 k = .06/12 =0 .005 or 0.5% $18,000 = PMT X [(1-1/1.00572)/0.005] PMT = $18,000/60.33951394 = $298.31

82

8-58, Missing Cash Flow Problem

I. Given Information:Discount

Rate10%

Known Cash FlowsTime 0 Time 1 $100Time 2 $150Time 3 Time 4 $100

Total Present Value of all Cash Flows, including the missing cash flow

$320.74

II. Solution: The value of the missing cash flow at Time 3:Known Cash

FlowsPresent Value of Known Cash

FlowsTim

e0

Time1 $100 $90.9091

Time2 $150 $123.9669

Time3

Time4 $100 $68.3013

Total Present Value of all Cash Flows, includingthe missing cash flow

$320.74 (given)

Total present value of known cash flows only $283.1774 Difference (Present Value of missing cash flow) $37.5657

Future Value of Missing Cash Flow at Time 3 $50

8-59.a) n = 5*12 = 60 k = .08/12 = .0066666 $22,000 = PMT * [1-1/1.00666666760)/0.006666667] PMT = $22,000/118.5035147 = $446.0806 = $446.08

83

Chapter 9 Solutions

Answers to Review Questions

1. Which is lower for a given company: the cost of debt or the cost of equity? Explain. Ignore taxes in your answer.

The cost of debt is always less than the cost of equity for a givenfirm. This is because the debt investor is taking a lower risk than the equity investor and therefore the required rate of return is lower.

2. When a company issues new securities, how do flotation costs affectthe cost of raising that capital?

When a company issues new securities flotation costs increase the cost of raising the capital. The company receives a smaller amountof the proceeds from the new issues, the greater the flotation costs.

3. What does the “weight” refer to in the weighted average cost of capital?

The weight referred to in weighted average cost of capital refers to the portion of the total capital raised by the firm that comes from a given source such as debt, preferred stock or equity.

4. How do tax considerations affect the cost of debt and the cost of equity?

Because interest on debt is tax deductible to the issuing firm, thehigher the tax rate the lower the after tax cost of debt financing.Tax considerations do not enter into the cost of equity calculationsince dividends paid to stockholders are not tax deductible to the firm.

84

5. If dividends paid to common stockholders are not legal obligations of a corporation, is the cost of equity zero? Explain your answer.

Although common stockholders do not have a contractual claim on dividends the funds supplied by stockholders definitely have a cost. Equity investors are paid last and so they are taking the greatest risk among all the suppliers of capital. If the company does not earn a higher rate of return on equity funds to compensatefor the higher risk taken by equity investors, the price of the stock will fall and therefore the value of the firm.

6. What is the investment opportunity schedule (IOS)? How does it help financial managers make business decisions?

The investment opportunity schedule shows graphically proposed capital budgeting projects depicting the IRR and dollar amount of investment for each project. This helps the financial manager makebusiness decisions since the investment opportunity schedule and the marginal cost of capital schedule can be plotted together, withthose projects on the IOS schedule above the MCC being acceptable.

7. What is a marginal cost of capital schedule (MCC)? Is the schedulealways a horizontal line? Explain.

The marginal cost of capital schedule is a graphic depiction of theweighted average cost of capital at different levels of financing. The MCC schedule is not always a horizontal line. For many firms the MCC schedule increases, usually at discreet intervals, as the amount of funds to be raised increases.

8. For a given IOS and MCC, how do financial managers decide which proposed capital budgeting projects to accept, and which to reject?

For a given IOS and MCC, all independent projects that plot on the IOS above the MCC are accepted. Those projects on the IOS below the MCC are rejected.

85

Answers to End-of-Chapter Problems

9-1. a) (i) YTM = 7% AT kd = .07(1-.40) = 4.2% (ii) YTM = 11% AT kd = .11(1-.40) = 6.6%(iii) YTM = 13% AT kd = .13(1-.40) = 7.8%

b) (i) YTM = 7% AT kd = .07(1-.34) = 4.62% (ii) YTM = 11% AT kd = .11(1-.34) = 7.26%(iii) YTM = 13% AT kd = .13(1-.34) = 8.58%

9-2. a ) AT kd = .10(1-.00) = 10.0%

b ) AT kd = .10(1-.22) = 7.8%

c ) AT kd = .10(1-.34) = 6.6%

9-3. Company YTM Tax Rate(%) AT kd

A 8% 34% 0.8(1-.34) = 5.28% B 11% 40% 0.11(1-.40) = 6.60% C 14% 30% 0.14(1-.30) = 9.80%

9-4. YTM AT kd AT kd

T=40% T=34%8% 0.08(1-.40) = 4.80% 0.08(1-.34) = 5.28%14% 0.14(1-.40) = 8.40% 0.14(1-.34) = 9.24%16% 0.16(1-.40) = 9.60% 0.16(1-.34) = 10.56%

9-5. a ) kd = 13%

b ) AT kd = .13(1-.40) = 7.8%

9-6. kd = .095 * (1 - .35) = .06175 = 6.2%

86

9-7. kp = $2/($26 - $0.75) = $2/$25.25 = 7.92%

9-8. kp = $8.00/($61.00 - $1.00) = 13.3%AT kd = .11(1-.40) = 6.6%Leo is correct. The cost of debt is lower.

9-9. a) kp = $6/$50 = 12%b) kp = $6/($50 - $2.25) = 12.57%

9-10. kp = $100 X 0.12/($89 - ($89 X 0.05)) = $12/($89 - $4.45) = 14.19%

9-11.kp = $0.75/($27 - $1) = 2.88%

9-12.a) ks = ($7/$143) + 0. 13 = 17.90%b) kn = ($7/($143 - $4) + 0. 13 = 18.04%

9-13. AT kd = 0.14(1-.30) = 9.80%

ks = ($1.50/$39.00) + 0.04 = 7.85%

The cost of the company's retained earnings is lower. This would lead you to reevaluate the estimated numbers, or question the applicability of the valuation models used here, since ks cannot be lower than AT kd for a given company.

9-14.ks = .045 + 1.4(.12 - .045) = 15%

9-15.a) ks = ($7/$65) + 0.10 = 20.77%b) kn = ($7/($65 - $3)) + 0.10 =21.29%

Yes. Floatation costs make cost of capital from new common stock higher.

87

9-16. ks = kRF + (kM - kRF) X = .03 + (.11 - .03) X 1.6 = 15.8%

9-17. a ) ks = ($2 X 1.05)/$30 + .05 = 12.0%

b ) kn = ($2 X 1.05)/($30-$2) + .05 = 12.5%

c ) ks = .03 + (.12 - .03) x 1.4 = 15.6%

9-18. AT kd = 0.10(1-0.4) = 6%kp = $2/($31 - $1) = 6.67%kn = $4/($100 -$4) + .06 = 10.17%ka = (0.3)(6) + (0.15)(6.67) + (0.55)(10.17) = 8.394%

9-19. AT kd = 0.11(1-0.4) = 6.60%kp = $2/($26 - $0.75) = 7.92%kn = $7/($143 - $4) + .13 = 18.04%ka = (300,000/600,000)(0.066) + (100,000/600,000)(0.0792) + (200,000/600,000)(0.1804) = 10.63%

9-20. ka = (600/1250)(0.12(1-0.04)) + (250/1250)(0.14) + (400/1250)(0.16)

= (.48 X .072) + (.20 X .14) + (.32 X .16) = .03456 + .028 + .0512 = 11.38%

9-21. AT kd = .10(1-.35) = 6.5%kp = $2/($25 - $1.00) = $2/$24 = 8.33%kn = ($5/($140 - $4) + 0.10 = 13.68%ka = (300,000/1,000,000)(0.065) + (100,000/1,000,000)(0.0833)

+(600,000/1,000,000)(0.1368) = 10.99% = minimum expected rate of return needed to satisfy the

suppliers of capital.

9-22.0.60(0.05) + 0.10(0.08) + 0.30(0.12) = 0.074 = 7.4%

88

9-23.kd = .095(1 - 0.35) = .06175 = 6.2%kp = $10/$50 = 0.20 = 20%ks = 0.04 + 1.1(0.12 – 0.04) = 12.8%

Weight:Debt = 230,000/430,000 = 0.54Preferred Stock = 100,000/430,000 = 0.23Common Equity = 100,000/430,000 = 0.23

WACC = .54(0.062) + .23(0.20) + .23(0.128) = .10892 = 10.892%

9-24. a ) $200,000/0.40 = $500,000 equity break-point

b ) $500,000/0.60 = $833,333 debt break-point

9-25. $1,000,000/.4 = $2,500,000 total capital raised before BPd1 is reached.

$2,000,000/.4 = $5,000,000 total capital raised before BPd2 is reached.$2,750,000/.5 = $5,500,000 total capital raised before BPe is reached.

a ) ka = (0.40)(0.11(1 - 0.40)) + (0.5)(0.13) + (0.1)(0.12) = 10.34%

b ) ka = (0.40)(0.13(1 - 0.40)) + (0.5)(0.13) + (0.1)(0.12) = 10.82%

c ) ka = (0.40)(0.15(1 - 0.40)) + (0.5)(0.13) + (0.1)(0.12) = 11.30%

9-26. INVESTMENT OPPORTUNITIES

PROJECT INVESTMENT RETURNOPTIMAL CAPITAL

STRUCTURE:DEBT 35.00% EQUITY 65.00% A $500,000 0.16

TAX RATE 40.00% B $1,600,000 0.12 NET INCOME NEXT YEAR: $1,200,0

00 C $600,000 0.15

89

ADDITION TO RETAINED EARNINGS $1,000,000

D $1,500,000 0.18

LOAN INTEREST RATE 10.00% FOR LOAN UPTO $750,000 $4,200,00012.00% FOR LOAN ABOVE

$750,000COMMON STOCK PRICE PER SHARE $50

DIVIDEND PER SHARE $5 GROWTH RATE 9.00%

FLOATATION COST 8.00%

a.

COST OF NEW EQUITY 19.87%

COST OF RETAINED EARNINGS 19.00%AT COST OF DEBT 6.00% FOR LOAN UPTO $750,000

7.20% FOR LOAN ABOVE $750,000

b.

EQUITY BREAK POINT $1,538,462

DEBT $538,462 DEBT BREAK POINT $2,142,8

57 DEBT $750,000 TOTAL EQUITY $1,392,8

57

c.

MCC UPTO TOTAL CAPITAL OF $1,538,462 = 14.45%

MCC BETWEEN $1,538,462 AND $2,142,857 15.02%

MCC ABOVE $2,142,857 15.44%

d.

INVESTMENT OPPORTUNITY SCHEDULE

PROJECT INVESTMENT RETURND $1,500,000 0.18 A $500,000 0.16 C $600,000 0.15 B $1,600,000 0.12

e.

90

f. Only Projects D and A would be chosen. They are the ones with IRRvalues on the IOS schedule that plot above the MCC schedule.

91

M CC/IO S ScheduleStone W ood Products

11.00%12.00%13.00%14.00%15.00%16.00%17.00%18.00%19.00%

$0 $500 $1,000 $1,500 $2,000 $2,500 $3,000 $3,500 $4,000 $4,500

Capital Budget Size ($000s)

Costs of Ca

pital and IRRs

M CC IOS

Project D

AC

B

Chapter 10 Solutions

Answers to Review Questions

1. How do we calculate the payback period for a proposed capital budgeting project? What are the main criticisms of the payback method?

We calculate the payback period for a proposed project by adding a project’s positive cash flows, one period at a time, until the sum equals the initial investment. The number of time periods it takesto cover this investment is the payback period. The main criticisms of the payback method are that cash flows after the payback period are ignored and the time value of money is not considered.

2. How does the net present value relate to the value of the firm?

The net present value is the dollar amount of the change to the value of the firm if the project under consideration is accepted.

3. What are the advantages and disadvantages of the internal rate of return method?

The internal rate of return method is a discounted cash flow methodand a number expressed as a percentage. These are typically seen as advantages. The main disadvantage of the internal rate of return is that it is somewhat more difficult to calculate, althoughthis is less true with the ready availability of financial calculators.

4. Provide three examples of mutually exclusive projects.

Mutually exclusive projects are projects that compete against each other for our selection. If a firm were considering the purchase

92

of a new computer, needing only one computer, then the proposals made by the sales reps from Hewlett-Packard, Compaq, and Toshiba would be mutually exclusive projects vying for our selection.

5. What is the decision rule for accepting or rejecting proposed projects when using net present value?

When using the net present value decision rule any project with a net present value greater than or equal to zero would be acceptable. Any project with a negative net present value would berejected.

6. What is the decision rule for accepting or rejecting proposed projects when using internal rate of return?

Whenever the internal rate of return is greater than or equal to the required rate of return, the hurdle rate, the project is accepted. When the internal rate of return is less than this required rate of return, the project is rejected.

7. What is capital rationing? Should a firm practice capital rationing? Why?

Capital rationing is the practice of setting dollar limits on what will be invested in new capital budgeting projects. Proprietorships, partnerships and private corporations are in a position to do whatever the owners wish. It can be argued, however, that for a publicly traded corporation capital rationing may not be consistent with maximizing the value of the firm. This is because some value adding projects may be rejected if they wouldcause the firm to exceed its self imposed capital rationing limit.

8. Explain how to resolve a “ranking conflict” between the net presentvalue and the internal rate of return. Why should the conflict be resolved as you explained?

Whenever there is a ranking conflict between net present value and internal rate of return we generally suggest that the project with

93

the highest net present value be chosen. This is because the net present value method ties more directly with the primary financial goal of the firm, to maximize firm value.

9. Explain how to measure the firm risk of a capital budgeting project.

The firm risk of a capital budgeting project measures the impact ofadding a new project to the existing projects of the firm.

10. Why is the coefficient of variation a better risk measure to use than the standard deviation when evaluating the risk of capital budgeting projects?

The coefficient of variation is a better risk measure than the standard deviation alone because the CV adjusts for the size of theproject. The CV measures the standard deviation divided by the mean and therefore puts the standard deviation into context. For example, a standard deviation of .05 may be considered large relative to a mean of .02 but would be considered a small value relative to a mean value of 8.

11. Explain why we measure a project’s risk as the change in the CV.

We measure a project’s risk as the change in the coefficient of variation because this focuses on the change in the riskiness of the firm’s existing portfolio.

12. Explain how using a risk-adjusted discount rate improves capital budgeting decision making compared to using a single discount rate for all projects?

The risk-adjusted discount rate improves capital budgeting decisionmaking compared to the single discount rate approach because the RADR allows us to set a higher hurdle for the high risk project anda lower hurdle for the low risk project thus aligning our capital budgeting decision making process more closely with the goal of maximizing the value of the firm.

94

Answers to End-of-Chapter Problems

10-1.a ) Peter: (10,000) + 4,000 + 4,000 + 4,000 ----> 2.5 yearsPaul: (10,000) + 2,000 +8,000 + 2,000 ----> 2.0 yearsMary: (10,000) + 10,000 + 1,000 + 1,000 ----> 1 year

b ) Mary's project is most liquid using payback as the liquidity measure.

10-2.CF0 = -20,000,000CF1-25 = $2,000,000I = 8%NPV = $1,349,552

10-3.IRR = 8.78%

10-4.CF0 = -20,000,000CF25 = 146, 211,879.90MIRR = 8.28%

10-5. Expected Cash Flows Year Weights Cum. CF Waters Cum. CF0 -

$200,000(200,000

)-

$300,000(300,000

)1 100,000 (100,000

)200,000 (100,000

)2 75,000 (25,000) 150,000 50,0003 50,000 25,000 150,000 200,0004 100,000 125,000 150,000 350,000

Project Weights: 2.5 yearsProject Waters: 1.67 yearsProject Waters is the better project according to payback because it recoups its investment in a shorter time.

95

10-6. Expected Cash Flows Year Weights Waters0 -

$200,000-

$300,0001 100,000 200,0002 75,000 150,0003 50,000 150,0004 100,000 150,000

k= 10%NPVweights (k = 10%) = $58,759.65NPVwaters (k= 10%) = $220,934.36

10-7.a ) NPV = (17,291.42) + 5,000[1/1.121] + 8,000[1/1.122] + 10,000[1/1.123] =

$668.22

b ) NPV = (17,291.42) + 5,000[1/(1+k)1] + 8,000[1/(1+k)2] + 10,000[1/(1+k)3] = $0

IRR = k = 14%

c ) Yes. NPV is positive and IRR > Cost of capital

10-8.a ) Rifle Stock: NPV = -9,000 + 2,000 X .8850 + 5,000 X .7831 + 1,000 X .6931 + 4,000 X .6133 = -168.20Fork Lift: NPV = -12,000 +5,000 X .8850 + 4,000 X .7831 +6,000 X .6931 + 2,000 X .6133 = +942.60Packaging Equip. NPV = -18,200 + 5,000 X .7831 + 10,000 X .6931 + 12,000 X .6133 = +6.10

b ) Fork Lift and Packaging; both have positive NPVs.

96

10-9. a ) Cal's Project: NPV = -100,000 + 22,611 X (4.5638) = $3,192.08

Aron's Project:NPV = -300,000 + 63,655 X (4.5638) = -$9,491.31

b ) Cal's Project: 100,000/22,611 = PVIFAk,7 year; k = 13% = IRRAron's Project:300,000/63,655 = PVIFAk,7 year; k = 11% = IRR

c ) Cal's Project; NPV is positive and IRR > Cost of Capital.

d ) No.

10-10.

Time Cash Flow

FV of Cash Flowat T-10 if

reinvested @ 15%cost of capital(per Equation 8-

1a)

Years togo

Initialinvestment ($10,000)

T-1 $4,000 $14,072 9T-2 $4,000 $12,236 8T-3 $4,000 $10,640 7T-4 $4,000 $9,252 6T-5 $4,000 $8,045 5T-6 $4,000 $6,996 4T-7 $4,000 $6,084 3T-8 $4,000 $5,290 2T-9 $4,000 $4,600 1T-10 $4,000 $4,000 0

TerminalValue $81,215

a.) IRR 38%

b.) MIRR 23.3%

97

10-11. a ) Printer #1: Payback = 2 yearsPrinter #2: Payback = 1.77 years

b ) Printer #1: NPV = -2,000 +900 X .9091 + 1,100 X .8264 + 1,300 X .7513 = +703.92Printer #2: NPV = -2,500 + 1,500 X .9091 + 1,300 X .8264

+ 800 X .7513 = +539.01

c ) Printer #1: NPV = 0 = -2,000 + 900 X [1/(1+k)1] + 1,100 X [1/(1+k)2] + 1,300 X [1/(1+k)3]

IRR = k = .2782 = 27.82%

Printer #2: NPV = 0 = -2,500 + 1,500 X [1/(1+k)1] + 1,300 X [1/(1+k)2] + 800 X [1/(1+k)3]

IRR = k = .2325 = 23.25%

d ) Printer #1 with higher NPV and higher IRR

e ) Printer #1: NPV = -2,000 + 900 X .8621 + 1,100 X .7432 + 1,300X .6407 = +426.32

Printer #2: NPV = -2,500 + 1,500 X .8621 + 1,300 X .7,432 +800 X .6407

= +271.87

No. NPV of Printer #1 is still higher.

10-12. Expected Cash Flows Year Program0 -

20,000,000

1 1,000,000

2 2,000,000

98

3 5,000,000

4 6,000,000

5 6,000,000

6 6,000,000

7 6,000,000

8 6,000,000

9 6,000,000

10 6,000,000

I = 15%NPV = $2,082,694.77IRR = 17.14%MIRR = 16.14%TV = $89,336,820

10-13. a ) Project A: NPV = -11,000 + 4,000 X (3.9975) = $4,990.00Project B: NPV = -17,000 + 4,500 X (3.9975) = $988.75Project A. Project A should be selected because it has the

higher NPV

b) Both. Both the NPVs are positive.

c ) Project A: IRR : 11000/4000 = PVIFAk,6 year; k =28.16 %Project B: IRR : 17000/4500 = PVIFAk,6 year; k = 15.07%

d ) Mutually exclusive: Project A has the higher IRR and would beselected. Independent: Select both. (IRR > Cost of Capital for

both the projects)

e ) Project C: NPV = -17,000 + 37,500X(PVIF13%,6 year) = -17,000 + 37,500 X .4803 = $1,011.25

99

Project C. NPV of Project C > NPV of Project B Project C would be chosen of Project B if these projects were mutually exclusive.

f ) Project C: IRR: 17,000/37,500 = .4533 = PVIFk,6 year; k = 14.09 %IRR of Project C < IRR of Project B Project B would be chosen because it has the higher IRR value.

g) Selections based on NPV and IRR method contradict each other. Since NPV method is generally preferred, select Project C.

10-14. a ) NPV = -5M + 1.85M(2.7982) -.25M(.4761) = $57,645

b ) Multiple IRRs are possible because of two sign changes in thecash flow series. In this case there is only one IRR, however. Doan NPV profile if you don’t believe us. The IRR is 16.59%.

TI BAII PLUS Financial CalculatorSolution

IRR

Keystrokes Display

[CF] CF0 = old contents

[2nd][CLR Work] CF0 = 0.00

5000000[+/-][ENTER]

CF0 =-5,000,000.00

[]1850000[ENTER][]4

C01 = 1,850,000.00F01 = 4.00

[]250000[+/-][ENTER][]

C02 = -250,000.00F02 = 1.00

[IRR] IRR = 0.00

[CPT] IRR = 16.59

c ) The IRR of 16.59% is greater than the required rate of return of 16%, so the project would get a positive recommendation.

100

10-15. a) Hydroelectric Project Geothermal Project CF0 = ($100,000) CF0 = ($100,000) CF1 = $20,000 CF1 = $60,000 CF2 = $30,000 CF2 = $40,000 CF3 = $40,000 CF3 = $20,000 CF4 = $90,000 CF4 = $10,000

b) NPVhydro (k = 6%) = $50,441.02 NPVgeo (k = 6%) = $44,181.07 Accept the Geothermal Upgrade project.

c) NPVhydro (k = 15%) = $17,834.06 NPVgeo (k = 15%) = $25,253.98 Neither project would be accepted because the NPV for each is

negative.

d) Approximately 9.58%. This is where the NPV profiles cross.

e) Greater than 21.66%

f) Greater then 31.92%

10-16. The Chalk Line Machine, Gel Padded Glove, Insect Repellant, and Recycled Base Cover projects collectively have initial cash outlaysof $90,000 (under the budget limit) and have NPVs that sum to $12,950. No other combination of projects gives a higher total NPVand stays under the budget limit.

10-17.Given Information:

Initial investment $5,669.62 Yearly net cash flows:

Year 1 $2,200 Year 2 $2,200 Year 3 $2,200

Required rate of return 12%a. NPV of the investment:

Year: 0 1 2 3

101

Annual cash flows ($5,670) $2,200 $2,200 $2,200 PV of cash flows ($5,670) $1,964 $1,754 $1,566

NPV ($386)b. Comment on the acceptability of the investment:

c. NPV Profile:Discount rate 0% 5% 10%

NPV 930 322 (199)

Comment:

d. Comment:

10-18. STD(IRR) = [(.05x(0%-6%)2 + .1x(1%-6%)2 + .2x(3%-6%)2 + .3x(6%-6%)2 + .2x(9%-6%)2 + .1x(11%-6%)2 + .05x(12%-6%)2].5 = 3.49%

COEFF. OF VARIATION (CV) = 3.49%/6% = .5817

10-19. PORT. STD. WITH A = [(.22x.022)+(.82x.032)+(.82x.032)+(2x.2x.8x.5x.02x.03)].5

= .0262 = 2.62%E(IRR) of portfolio with A = (.2x14%)+(.8x13%) = 13.2%CV of portfolio with A = 2.62%/13.2% = .1985

102

Comment: According to the NPV profile, the discount rate would haveto be less than about 8% in order for the project's NPV to be positive.

Comment: The IRR is that discount rate which produces an NPV of zero. Therefore, the IRR could be calculated to determine the "hurdle rate" below which the project's NPV would be positive (8% inthis case).

NPV PROFILE

930

322

(199)

($500)($300)($100)$100$300$500$700$900

0% 5% 10%DISCOUNT RATE

NPV

PORT. STD. WITH B = [(.22x.062)+(.82x.032)+(2x.2x.8x.5x.02x.03)].5 = .0317 = 3.17%E(IRR) of portfolio with B = (.2x16%)+(.8x13%) = 13.6%CV of portfolio with B = .0317/.136 = .2331

PORT. STD. WITH C = [(.22x.052)+(.82x.032)+(2x.2x.8x.5x.05x.03)].5 = .0303 = 3.03%E(IRR) of portfolio with C = (.2x11%)+(.8x13%) = 12.6%CV of portfolio with C = .0303/.126 = .2405

PORT. STD. WITH D = [(.22x.042)+(.82x.032)+(2x.2x.8x.5x.04x.03)].5 = .0288 = 2.88%

E(IRR) of portfolio with D = (.2x14%)+(.8x13%) = 12.6%CV of portfolio with B = .0288/.132 = .2182

Project A has the lowest risk and Project C the highest as measuredby the CV.

10-20. a) 1. CVA = 2%/10% = .22. E(IRR) of new combined portfolio = (700,000/900,000 x 10%) + (200,000/900,000 x 11% = 10.22%3. STD. Of new combined portfolio = [(.7782x.022)+(.2222x.03)+(2x.222x.778x.9x.02x.03)].5 = .0218 = 2.18%4. CV of new combined portfolio = .0218/.1022 = .2133

b) .2133 - .20 = .0133 change in CV

c) average risk

d) 1. Ave. Risk: NPV = $55,000/1.131 + 55,000/1.132 + 55,000/1.133

+ 100,000/1.134 - 200,000 = ($8,805)2. High Risk: NPV = $55,000/1.161 + 55,000/1.162 + 55,000/1.163

+ 100,000/1.164 - 200,000 = ($21,247)3. Low Risk: NPV = $55,000/1.101 + 55,000/1.102 + 55,000/1.103 + 100,000/1.104 - 200,000 = $5,078

10-21. a) (.125x2%)+(.20x5%)+(.35x9%)+(.20x13%)+(.125x16%) = 9.00%b) .125(.02-.09)2 + .2(.05-.09)2 + .35(.09-.09)2 + .2(.13-.09)2

+ .125(.16-.09)2 = .001865 = .1865%

103

square root of .001865 = .0432 = 4.32% standard deviation

c) CV of existing portfolio = .02/.08 = .25d) E(IRR) of new combined portfolio = (.8x8%)+(.2x9%) = 8.20%e) STD. DEV. of the combined portfolio = [(.22x.04322)+(.82x.022)+(2x.2x.8x1x.0432x.02)].5 = .0246 = 2.46%f) CV of combined portfolio = .0246/.0820 = .3005g) .3005 - .25 = .0505 increase in CV

10-22. Proj. A Proj. B Proj.C Proj. D

Standard deviation of existing portfolio: 4.00% 4.00% 4.00% 4.00%Standard deviation of new project: 9.00% 5.00% 3.00% 1.00%

Standard deviation of combined portfolio 3.71% 3.63% 3.61% 3.60%

expected return E(R) of new project: 18.00% 15.00% 11.00% 8.00%expected return E(R) of existing portfolio: 12.00% 12.00% 12.00% 12.00%expected return E(R) of combined portfolio: 12.60% 12.30% 11.90% 11.60%

a. Coefficient of variation of existingportfolio:

33.33%

b.&c. Coefficient of variation of combinedportfolio:

29.45% 29.55% 30.36% 31.05%

d. A IS THE D IS THELOWEST HIGHEST

RISK RISKPROJECT PROJECT

10-23. a. CV of existing portfolio = 5%/15% = 33.33%b. WT of existing portfolio of PROJ1 is added = $820,000/($820,000+$194,000) = .809 = 8.09%c. WT of PROJ1 if added to existing portfolio = 1 - .809 = .191 = 19.1%d. STD. DEV. of combined portfolio = [(.8092x.052)+(.1912x.09)+(2x.809x.191x1x.05x.09)].5 = .0577 = 5.77%The combined portfolio standard deviation is higher than that of the existing portfolio (5.77% versus 5.00%).e. CV of the combined portfolio = 5.77%/[(.809x15%)+(.191x18%)] = 5.77%/15.57%

104

= .3706

10-24.

= $298,500 x 6.7100814 - $2,000,000= $2,002,959.30 - $2,000,000= $2,959.30

= $298,500 x 6.14456711 - $2,000,000= $1,834,153.28 - $2,000,000= ($165,846.72)

c) The project should not be adopted.

d)

NPV = $0 = CF x 6.14456711 - $2,000,000$2,000,000 = CF x 6.14456711CF = $325,490.79

10-25 (Comprehensive Problem)Given Information:

105

a) NPV = $298,500 x [1 - 1

(1.08 )10

.08 ]

b) NPV = $298,500 x [1 - 1

(1.10 )10

.10 ]

NPV = $0 = CF x [1 - 1(1.10)10

.10 ]

Project 1 Project 2 Yearly net cash flows:

Initialinvestment

($200,000) ($200,000)

Year 1 $0 $90,000 Year 2 $0 $70,000 Year 3 $20,000 $50,000 Year 4 $30,000 $30,000 Year 5 $40,000 $10,000 Year 6 $60,000 $10,000 Year 7 $90,000 $10,000 Year 8 $100,000 $10,000

Weighted average cost ofcapital

7.2%

a. NPVs of the projects:

Project 1 Project 2

NPV $19,398 $33,705

b. IRRs of the projects:

Project 1 Project 2

IRR 8.8% 14.4%

c. NPV Profiles:

Discount rate 2% 4% 6% 8% 10% 12% 14% 16%NPV Project 1 99,769 65,182 35,340 9,502 (12,943) (32,504) (49,605) (64,600)NPV Project 2 65,526 52,380 40,396 29,435 19,376 10,118 1,573 (6,337)

106

NPV PROFILES

($80,000)($60,000)($40,000)($20,000)

$0$20,000$40,000$60,000$80,000$100,000$120,000

2% 4% 6% 8% 10% 12% 14% 16%

DISCOUNT RATE

NPV Project 1

Project 2

Comment: Both projects have the same NPV at a discount rate of approximately 5.5%. At that discount rate the NPV of both projects is about $45,000.

d. Project selection at other WACCs:

Select Reasoni. WACC > 5.4%

Project 2 Project 2's NPV is higher

ii. WACC > 8.81%

Project 2 Project 1's NPV is negative

iii. WACC > 14.39%

Neither project

The NPV for both projects is negative

e) Look at the NPV profile. If the discount rate is 5%, this is to the left of the crossover point. Project 1 would have a higher NPV than Project 2. This would create a ranking conflict if the projects were mutually exclusive. Project 2 has a higher IRR (14.3% for Project 2 versus 8.81 percent for Project 1).

At a discount rate below 5.4%, NPV and IRR give conflicting rankingsignals. At a discount rate of 5.4% or more, the ranking of the two projects is the same.

f) a. Both projects would be accepted at a 7.2% cost of capital.

107

Chapter 11 Solutions

Answers to Review Questions

1. Why do we focus on cash flows instead of profits when evaluating proposed capital budgeting projects?

We focus on cash flows instead of profits when evaluating proposed capital budgeting projects because it is cash flow that changes thevalue of a firm. You can spend cash but you can not spend profit.

2. What is a sunk cost? Is it relevant when evaluating a proposed capital budgeting project? Explain.

A sunk cost is a cash flow that has already occurred, or that will occur, whether a project is accepted or rejected. It is irrelevantwhen evaluating a proposed project.

3. How do we estimate expected incremental cash flows for a proposed capital budgeting project?

We estimate expected incremental cash flows for a proposed project by estimating the changes in sales and expenses that are incremental to the project, adding back the incremental depreciation expense since depreciation expense is a non-cash expense.

4. What role does depreciation play in estimating incremental cash flows?

Depreciation expense is a tax deductible expense and therefore affects cash flow through its effect on taxes. Depreciation expense that is incremental to a proposed project therefore affectsincremental cash flows.

108

5. How and why does working capital affect the incremental cash flow estimation for a proposed large capital budgeting project? Explain.

Many large projects require additional working capital. This investment in additional working capital becomes part of the initial investment. This investment is recovered at the end of theproject’s life. There may be some spontaneous increase in current liabilities associated with a project, but the change in net working capital, if any, is likely to be a positive value requiringan increase in the initial investment of that amount.

6. How do opportunity costs affect the capital budgeting decision-making process?

Opportunity costs reflect the foregone benefits of the alternative not chosen when a capital budgeting project is selected. Any decrease in the cash flows of the firm directly tied to the selection of a new project could be part of the opportunity cost value and included in our capital budgeting analysis.

7. How are financing costs generally incorporated into the capital budgeting analysis process?

Financing costs are usually captured in the discount or hurdle ratewhen doing NPV or IRR analysis. The operating cash flows usually do not include financing costs because this would be double counting.

Answers to End-of—Chapter Problems

11-1.Price of Selected Model $6,000Attachments 5,000Paint Name 300Garage and Maint. Facility12,000

$23,300

Cash Flow t0 = ($23,300)

109

11-2. (a) Resale Price: $60,000

Price of Equipment: $200,000Resale Value: $60,000Years Used: 3MACRS Classification: 5 yearsIncome Tax: 40%Accumulated Depreciation: 20% + 32% + 19.2% = 71.2% or,

0.712 X $200,000 = $142,400Book Value: $200,000 - $142,400 = $57,600Taxable Gain (Loss): $60,000 - $57,600 = $2,400Tax (Refund): $2,400 X 0.40 = $960Net Cash Flow: $60,000 - $960 = $59,040

(b) Resale Price: $80,000

Price of Equipment: $200,000Resale Value: $80,000Years Used: 3MACRS Classification: 5 yearsIncome Tax: 40%Accumulated Depreciation: 20% + 32% + 19.2% = 71.2% or,

0.712 X $200,000 = $142,400Book Value: $200,000 - $142,400 = $57,600Taxable Gain (Loss): $80,000 - $57,600 = $22,400Tax (Refund): $22,400 X 0.40 = $8,960Net Cash Flow: $80,000 - $8,960 = $71,040

11-3. a) $10,000 - $3,000 - $2,000 = $5,000

b ) $5,000 X .35 = $1,750

c ) $5,000 - $1,750 = $3,250

d ) $3,250 + $2,000 depr. add back = $5,250

e ) Interest Expense. It is included in the cost of funds when calculating NPV and when setting the IRR hurdle rate.

110

11-4.Mower $20,000Annual Revenues (increase) 100,000Operating Costs (increase) 30,000

Year 1 $100,000

- 30,000 70,000

Depreciation (20%) - 4,000EBIT 66,000Taxes (35%) - 23,100

42,900Depreciation + 4,000Net operating CF $46,900

The net operating incremental cash flow for year 1 is $46,900

11-5.$2,000 * (1 – 0.35) = $1,300

11-6. Initial Cost of new

Equipment$375,000

End of year: 1 2 3 4 5 6

Earnings BeforeDepreciation

and Taxes (EBDT) $120,000 $90,000 $70,000 $70,00

0 $70,00

0 $70,00

0

Discount rate 13%Tax rate 40%

Year 1 2 3 4 5 6 MACRS depreciationpercentages for five-year class

20.00% 32.00% 19.20% 11.50% 11.50% 5.80%

life equipment

Calculations:

Incremental Cash Flows:

Year 1 2 3 4 5 6

EBDT $120,000 $90,000 $70,000 $70,00

0 $70,00

0 $70,00

0

111

New depreciationexpense

(75,000) (120,000)

(72,000)

(43,125)

(43,125)

(21,750)

Change in OperatingIncome

45,000 (30,000)(2,000

)26,875 26,875 48,250

Income tax on newincome

(18,000) 12,000 800 (10,750)

(10,750)

(19,300)

Change in earningsafter tax

27,000 (18,000)(1,200

)16,125 16,125 28,950

Add back depreciation 75,000 120,000 72,000 43,125 43,125 21,750

Net incrementaloperating cash flows $102,000 $102,00

0 $70,80

0 $59,25

0 $59,25

0 $50,70

0

Present value of cashflows

$90,265 $79,881 $49,068 $36,33

9 $32,15

9 $24,35

2

Total present valueof cash flows $312,064

Less initial cost ($375,000)

= NPV ($62,936)

The NPV is negative so the project should be rejected.

11-7. Rhodes Manufacturing Corporation (with salvage value)

Given:

Initial Cost of newEquipment

$375,000

End of year: 1 2 3 4 5 6

Earnings BeforeDepreciation

and Taxes (EBDT) $120,000 $90,000 $70,000

$70,000

$70,000

$70,000

Discount rate 13%Tax rate 40%

Year 1 2 3 4 5 6 MACRS depreciationpercentages for five-year class

20.00% 32.00% 19.20% 11.50% 11.50% 5.80%

life equipmentResale value of $50,000 at the end of the

112

equipment sixth year

Calculations:Incremental Cash Flows:

Year 1 2 3 4 5 6

EBDT $120,000 $90,000 $70,000

$70,000

$70,000

$70,000

New depreciationexpense

(75,000) (120,000)

(72,000)

(43,125)

(43,125)

(21,750)

Change in OperatingIncome

45,000 (30,000)

(2,000)

26,875 26,875 48,250

Income tax on newincome

(18,000) 12,000 800 (10,750)

(10,750)

(19,300)

Change in earningsafter tax

27,000 (18,000)

(1,200)

16,125 16,125 28,950

Add back depreciation 75,000 120,000 72,000 43,125 43,125 21,750

Net incrementaloperating cash flows $102,000 $102,00

0 $70,80

0 $59,25

0 $59,25

0 $50,70

0

Resale value ofequipment

50,000

Less income tax onsale

(20,000)

Net cash flow fromequipment sale

30,000

Total Net cash flows $102,000 $102,000

$70,800 $59,25

0 $59,25

0 $80,70

0

Present value ofcash flows

$90,265 $79,881 $49,06

8 $36,33

9 $32,15

9 $38,76

2

Total present valueof cash flows $326,474

Less initial cost ($375,000)

= NPV ($48,526)

The NPV is negative so the project should be rejected.

11-8. a ) $85,000 + $20,000 = $105,000

b ) $125,000 X .10 = $12,500

113

Oper. Exp. -20,000Depr. Exp. -10,500

-18,000Tax Saving@40% 7,200

-10,800Add back Depr. 10,500

Net Incremental Oper. Cash flow -300

c ) End of year 5 at the time of the sale

11-9.GHOST SQUADRON HISTORICAL AIRCRAFT, INC.

ASSUMPTIONS:Yr 1 Yr 2 Yr 3 Yr 4 Yr 5 Yr 6 Yr 7 Yr 8

MACRS Depreciation 14.3% 24.5% 17.5% 12.5% 8.9% 8.9% 8.9% 4.5%

Tax rate 35%Cost of capital 12%

ESTIMATED INCREMENTAL CASH FLOWS:

Initial Investment at t=0:

Crew transport & wreckagecollection

($100,000)

Transport to restorationfacility

($35,000)

Plane restoration ($600,000)

Total Initial Investment ($735,000)

Year: 1 2 3 4 5 6 7

New Revenues $70,000 $70,000 $70,000 $70,000 $70,000 Additional operating

expenses($40,000

)($40,000

)($40,000

)($40,00

0)($40,00

0)($40,00

0)($40,00

0)Depreciation on plane ($105,10

5)($180,07

5)($128,62

5)($91,87

5)($65,41

5)($65,41

5)($65,41

5)Change in Operating Income ($145,10

5)($220,07

5)($98,625

)($61,87

5)($35,41

5)($35,41

5)($35,41

5)Tax on new income $50,787 $77,026 $34,519 $21,656 $12,395 $12,395 $12,395

Change in Earnings aftertax

($94,318)($143,04

9)($64,106

)($40,21

9)($23,02

0)($23,02

0)($23,02

0)Add back depreciation $105,105 $180,075 $128,625 $91,875 $65,415 $65,415 $65,415

Net Incremental Cash Flows $10,787 $37,026 $64,519 $51,656 $42,395 $42,395 $42,395

114

Additional Cash Flows at the end of year 7:

Proceeds from sale of plane $500,000 Book value of plane $33,075 Taxable gain(loss) $466,925

Tax on gain $163,424

Net cash flow from sale ofplane

$336,576

(Salvage value less taxon gain)

SUMMARY OF NET CASH FLOWS:Time: 0 1 2 3 4 5 6 7

($735,000)

$10,787 $37,026 $64,519 $51,656 $42,395 $42,395 $378,972

Net present Value: ($400,138)

Internal rate of Return: -2.7%

11-10. a ) NWC = Current Assets - Current Liabilities= ($8,000 + $10,000 + $12,000) - ($6,000 + $2,500) = $21,500

b ) Outflow

c ) Beginning of year 1

11-11.Given:

Initial Cost of new Equipment $90,000 End of year: 1 2 3 4

New revenues $50,000 $30,00

0 $20,00

0 $20,00

0 Discount rate 11%

Tax rate 30%

Year 1 2 3 4 MACRS depreciation percentages forthree-year class life equipment 33.30% 44.50% 14.80% 7.40%

Resale value of equipment $10,000 at the end of the fourth year

Calculations:

115

Incremental Cash Flows:Year 1 2 3 4

Revenues $50,000 $30,00

0 $20,00

0 $20,00

0

New depreciation expense(29,970

)(40,05

0)(13,32

0)(6,660

)

Change in Operating Income 20,030 (10,05

0) 6,680 13,340

Income tax on new income (6,009) 3,015 (2,004

)(4,002

)

Change in earnings after tax 14,021 (7,035

) 4,676 9,338 Add back depreciation 29,970 40,050 13,320 6,660

Net incremental operating cashflows $43,991

$33,015

$17,996

$15,998

Resale value of equipment 10,000

Less income tax on sale(3,000

)Net cash flow from

equipment sale 7,000

Total Net cash flows $43,991 $33,01

5 $17,99

6 $22,99

8

Present value of cash flows $39,632 $26,79

6 $13,15

9 $15,14

9 Total present value of cash flows $94,735

Less initial cost($90,00

0)= NPV $4,735

11-12.MACRS 3 YEARS 33.30% 44.50% 14.80

%7.40

%

PRICE OF NEW EQUIPMENT: 90,000

YEARS USED 4

INCOME TAX RATE: 40.00% COST OF CAPITAL 10.00%

SALVAGE VALUE OF NEW EQPT. 10,000

MACRSCLASSIFICATION:

3 YEARS 1

2

3 4

DEPRECIATION RATE 33.30% 44.50%

14.80%

7.40%

116

Comments: Yes, since the project has a positive NPV at the company's cost of capital, Flower Belle should recommend that it be accepted.

ACCUM. DEP. %) 100%

CASH FLOW FROM SALE OF NEW EQUIPMENT 6,000

CASH FLOW FROM SALE OF OLD EQUIPMENT

SALE PRICE 10,000 BOOK VALUE 20,000

TAXABLE GAIN (LOSS) (10,000)

TAX (REFUND) (4,000)

NET CASH FLOW 14,000

INCREMENTAL CASH FLOWYEAR 1 1 2 3 4

REVENUE STREAM 50,000 30,000 20,00

0 20,00

0 DEPRECIATION EXPENSE 29,97

0 40,050 13,32

0 6,660

CHANGE IN OPERATING INCOME 20,030 (10,05

0)6,680 13,34

0 TAX ON NEW INCOME 8,012 (4,020

)2,672 5,336

CHANGE IN EARNINGS 12,018 (6,030

)4,008 8,004

ADD BACK DEPRECIATION 29,970 40,050 13,32

0 6,660

NET INCREMENTAL OP. CASH FLOW 41,988 34,020 17,32

8 14,66

4

NET CASH FLOW0 1 2 3 4

NEW EQUIPMENT (90,000)OLD EQUIPMENT 14,000

OPERATING CF 41,988 34,020 17,32

8 14,66

4 SALVAGE VALUE

NET CASH FLOW (76,000) 41,988 34,020 17,32

8 14,66

4

DISCOUNT RATE 10.00%NPV $17,419.18

117

11-13. Initial Cost of new

equipment$90,000

End of year: 1 2 3 4 New revenues $50,000 $30,000 $20,000 $20,00

0 Discount rate 10%

Tax rate 40%

Year 1 2 3 4 MACRS depreciation percentages forthree-year classlife equipment

33.30% 44.50% 14.80% 7.40%

Book value of old equipment $20,000 Resale value of old

equipment$10,000

Resale value of newequipment

$10,000 at the end of the fourth year

Additional current assetsrequired

$10,000

Expected increase in currentliabilities

$5,000

Calculations:Incremental Cash Flows:

Gain(loss) on sale of oldequipment

($10,000)

(Tax)refund on transaction $4,000 Net cash received for old

equipment$14,000

Cost of New Equipment ($90,000)

Net Cash Outflow at T-0 forequipment

($76,000)

Additional net workingcapital required

($5,000)

Total Net Cash Outflow at T-0

($81,000)

Year 1 2 3 4 Revenues $50,000 $30,000 $20,000 $20,00

0 New depreciation expense (29,970) (40,050

)(13,320

)(6,660

)Change in Operating Income 20,030 (10,050

)6,680 13,340

Income tax on new income (8,012) 4,020 (2,672) (5,336)

Change in earnings after tax 12,018 (6,030) 4,008 8,004

118

Add back depreciation 29,970 40,050 13,320 6,660

Net incremental operatingcash flows

$41,988 $34,020 $17,328 $14,664

Resale value of equipment 10,000 Less income tax on sale (4,000

)Net cash flow from

equipment sale6,000

Recovery of net working capital investment 5,000

Total Net cash flows $41,988 $34,020 $17,328 $25,664

Present value of cash flows $38,171 $28,116 $13,019 $17,529

Total present value of cashflows

$96,834

Less initial cash outflow atT-0

($81,000)

= NPV $15,834

11-14. a ) Book Value = $20,000 - $12,000 = $8,000

b ) Taxable Gain = $18,000 - $8,000 = $10,000

c ) Tax on Gain = $10,000 X 0.3 = $3,000

d ) Cash Flow = $18,000 - $3,000 = $15,000This is an inflow.

e ) Incremental Cash Flow for to = $40,000 - $15,000 = $25,000 outflow

f ) b ) Taxable Income (Loss) = $6,000 - $8,000 = ($2,000)

c ) Tax Credit on Loss = $2,000 X 0.3 = $600

d ) Cash Flow = $6,000 + $600 = $6,600This is an inflow.

e ) Incremental Cash Flow for to = $40,000 - $6,600 = $33,400 outflow

119

11-15.MACRS 3 YEARS 33.30% 44.50

%14.80

%7.40%

PRICE OF NEW EQUIPMENT: $22,000 PRICE OF OLD EQUIPMENT: 0 RESALE VALUE OF OLD EQPT: 0 YEARS USED 4

INCOME TAX RATE: 40.00% COST OF CAPITAL 14.00%

SALVAGE VALUE OF NEW EQUIP. $0

MACRS CLASSIFICATION: 3 YEARS 1

2 3 4

DEPRECIATION RATE 33.30% 44.50%

14.80%7.40%

ACCUMULATED DEPR.(%) 100%

CASH FLOW FROM CHANGE IN NWC

CHANGE IN CURRENT ASSETS 5000 CHANGES IN CURRENT

LIABS.3000

CHANGE IN NWC 2000

INCREMENTAL CASH FLOWYEAR 1 2 3 4 CHANGE IN SALES: 20,000 20,00

0 10,00

0 10,00

0 INCREASE IN OPERATING EXPENSES (4,000

)(4,00

0)(2,00

0)(2,00

0)TOTAL INFLOW 16,000 16,00

0 8,000 8,000

DEPRECIATION EXPENSE 7,326 9,790 3,256 1,628

CHANGE IN OPERATING INCOME 8,674 6,210 4,744 6,372 TAX ON NEW INCOME 3,470 2,484 1,898 2,549

CHANGE IN EARNINGS 5,204 3,726 2,846 3,823

ADD BACK DEPRECIATION 7,326 9,790 3,256 1,628 NET INCREMENTAL OP. CASH FLOW 12,530 13,51

6 6,102 5,451

NET CASH FLOW0 1 2 3 4

a. NEW EQUIPMENT (22,000)

NWC (2,000) 2,000 b. OPERATING CF 12,530 13,51 6,102 5,451

120

6

c. NET CASH FLOW (24,000)12,530 13,51

6 6,102 7,451

DISCOUNT RATE 14.00%NPV $5,922.

36 IRR 27.24%

Yes, Brenners should add this machine to their factory.

121

11-16.Given:

Initial Cost of newEquipment $150,000

Installation andcalibration costs $7,500

Decrease in operatingexpenses $50,000

annually

Discount rate 10%Tax rate 35%

Year 1 2 3 4 MACRS depreciationpercentages for three-yearclass 33.30% 44.50% 14.80% 7.40%life equipment

Calculations:

Incremental Cash Flows at T-0:

Cost of New Equipment($150,00

0)Installation andcalibration costs ($7,500)

Total Net Cash Outflow atT-0

($157,500)

Incremental cash flows in years 1 - 5:

Year 1 2 3 4 5

Reduction in operatingcosts $50,000

$50,000

$50,000

$50,000

$50,000

New depreciation expense (52,448)(70,08

8)(23,31

0)(11,65

5) 0

Change in Operating Income (2,448)(20,08

8) 26,690 38,345 50,000

Income tax on new income 857 7,031 (9,342

)(13,42

1)(17,50

0)Change in earnings after

tax (1,591)(13,05

7) 17,349 24,924 32,500 Add back depreciation 52,448 70,088 23,310 11,655 0

Net incremental

operating cash flows $50,857 $57,03

1 $40,65

9 $36,57

9 $32,50

0

122

a. NPV of the investment:Present value of cash

flows $46,233 $47,13

3 $30,54

7 $24,98

4 $20,18

0

Total present value ofcash flows $169,077

Less initial cash outflowat T-0

($157,500)

a. NPV = $11,577 b. Yes, since the NPV of the investment is positive at RHPS's cost of capital, Weiss and Majors should go forward with the project.

123

11-17.Chemical Company of Baytown

Given:

Original cost of old equipment $40,000 on Dec 31, 2004Resale value of old equipment $4,000 on Dec 31, 2006

Discount rate 6%Tax rate 40%

Year 1 2 3 4 MACRS depreciationpercentages for three-year class

33.30% 44.50% 14.80% 7.40%

life equipment

Calculations:

a. Cash flows from sale of old equipment:

Year 2005 Year 2006

Depreciation expense on oldequipment

$13,320 $17,800

Total accumulated depreciation $31,120 Book value of old equipment $8,880 on Dec 31, 2006

Resale value of old equipment $4,000 on Dec 31, 2006Gain(loss) on sale of old

equipment($4,880)

(Tax)refund on transaction $1,952 Net cash received for old

equipment$5,952

b. New net working capital requirements:

Additional current assetsrequired:

Cash $1,000 Receivables $5,000

Inventory $10,000 Total $16,000

Expected increase in currentliabilities:

Accounts payable $6,000 Accrued expenses $3,000

Total $9,000

Incremental cash flow for net workingcapital

$7,000

c. Net cash outflow at the end of 2006 if new process line is installed:

124

Cost of New Equipment $180,000 Additional net working capital $7,000 Less proceeds from sale of old

equipment($5,952)

Net cash outflow at the end of2006

$181,048

d. Incremental cash flows for 2007 - 2010:

End of year: 2007 2008 2009 2010

New revenues $60,000 $60,000 $60,000 $60,000

Reduction in operating expenses 6,000 6,000 6,000 6,000 New depreciation expense (59,940) (80,100) (26,640) (13,32

0)Change in Operating Income 6,060 (14,100) 39,360 52,680

Income tax on new income (2,424) 5,640 (15,744) (21,072)

Change in earnings after tax 3,636 (8,460) 23,616 31,608 Add back depreciation 59,940 80,100 26,640 13,320

Net incremental operating cashflows

$63,576 $71,640 $50,256 $44,928

e. NPV and IRR of the investment:

(Given) Resale value of newequipment

$20,000 at the end of the fourth year

Resale value of equipment 20,000 Less income tax on sale (8,000

)Net cash flow from equipment

sale12,000

Recovery of net working capitalinvestment

7,000

Total Net cash flows $63,576 $71,640 $50,256 $63,928

Year 1 2 3 4

Present value of cash flows $59,977 $63,759 $42,196 $50,637

Total present valueof cash flows $216,570

Less initial cash outflow at T- ($181,048)

125

0

= NPV $35,522

Summary of all cash flows:Year 0 1 2 3 4

Net Cash Flow ($181,048) $63,576 $71,640 $50,256 $63,92

8

IRR 14.4%

f. NPV ProfileYear 0 1 2 3 4

Net Cash Flow ($181,048)

$63,576 $71,64

0 $50,25

6 $63,92

8

Assumed cost ofcapital

0% 1% 2% 3% 4% 5% 6% 7% 8% 9% 10%

NPV of cash flows $68,352 $62,338 $56,55

6 $50,99

4 $45,64

1 $40,48

7 $35,52

2 $30,73

6 $26,12

2 $21,67

2 $17,37

7

126

NPV Profile, Chem ical Com pany of Baytow n Project

$68,352$62,338

$56,556$50,994

$45,641$40,487$35,522

$30,736$26,122$21,672$17,377

$0$10,000$20,000$30,000$40,000$50,000$60,000$70,000$80,000

0% 1% 2% 3% 4% 5% 6% 7% 8% 9% 10%Cost of Capital

NPV

11-18.PROBLEM 11-18

Real Options Decision Tree NPV AnalysisJ & T's Double Diamond Brewhouse

|-------- Part b.--------|

|- Partc. -|

Time Time Time Time Time Time Joint Path

t0 1 2 3 4 5Probability NPV

JP xNPV

50%$400,00

0 $400,00

0 $400,00

0 12.5% $666,954 $83,369

25%$200,00

0 100%$100,00

0 30%$200,00

0 $200,00

0 $200,00

0 7.5% $309,669 $23,225 20% $90,000 $90,000 $90,000 5.0% $113,163 $5,658

($300,000) 50%

$100,000 100%

$100,000 100%

$100,000

$100,000

$100,000 50.0% $43,308 $21,654

0%($40,00

0) 100%($40,00

0)($40,00

0)($40,00

0) 0.0%($437,32

3) $0

25%($40,00

0)

100% $0 100% $0 $0 $0 25.0%($335,08

8)($83,77

2)

TotalNPV of

theDeal: $50,135

Cost ofCapital: 14%

The Time 2 cash flow for the smash hit

127

scenariois $200,000 from operations minus $100,000for the expansion.

128

Chapter 12 Solutions

Answers to Review Questions

1. Describe the general pattern of cash flows from a bond with a positive coupon rate.

Cash flows from a bond with a positive coupon rate consist of periodic interest payments and the face value payment at maturity. Coupon interest payments occur at regular intervals throughout the life of the bond. The face value payment occurs on the maturity date.

2. How does the market determine the fair value of a bond?

The fair value of a bond is the present value of the bond's coupon interest payments plus the present value of the face value payment at maturity, discounted at the market’s required rate of return forthe bond in question. Equation 9-1 in the text is use to solve forthe fair (present) value of a bond.

3. What is the relationship between a bond's market price and its promised yield to maturity? Explain.

A bond's market price depends on its yield to maturity (YTM). Whena bond has a YTM greater than its coupon rate, it sells at a discount from its face value. When the YTM is equal to the coupon rate, the market price equals the face value. When the YTM is lessthan the coupon rate, the bond sells at a premium over face value.

4. All other things held constant, how would the market price of a bond be affected if coupon interest payments were made semiannuallyinstead of annually?

Most bonds issued in the United States pay interest semiannually (twice per year). With semiannual interest payments, we must

129

adjust the bond valuation model (Equation 9-1 in the text) by multiplying n, the number of years to maturity, by two, and dividing k, the annual interest rate, by two.

5. What is the usual pattern of cash flows for a share of preferred stock? How does the market determine the value of a share of preferred stock, given these promised cash flows?

Preferred stock has no maturity date, so it has no maturity value. Its future cash payments are dividend payments that are paid to preferred stockholders at regular time intervals for as long as they (or their heirs) own the stock. Cash payments from preferred stock dividends are scheduled to continue forever. To value preferred stock, we adapt the discounted cash flow model to reflectthat preferred stock dividends are a perpetuity. See Equation 9-4 in the text.

6. Name two patterns of cash flows for a share of common stock. How does the market determine the value of the most common cash flow pattern for common stock?

Cash flows for a share of common stock consist of dividend paymentsand the price received for the eventual sale of the share. Common stock valuation is complicated by the fact that common stock dividends are difficult to predict compared to the interest and principal payments on a bond, or dividends on preferred stock. Indeed, corporations may pay common stock dividends irregularly, ornot pay dividends at all.

As with bonds and preferred stock, the market values common stock by estimating the present value of the expected future cash flows from the common stock. See Equation 9-6 in the text.

7. Define the P/E valuation method. Under what circumstances should a stock be valued using this method?

The P/E ratio indicates how much investors are willing to pay for each dollar of a stock's earnings. A high P/E ratio indicates that

130

investors believe the stock's earnings will increase, or that the risk of the stock is low, or both.

Financial analysts often use a P/E model to estimate common stock value for businesses that are not public. First, analysts compare the P/E ratios of similar companies within an industry to determinean appropriate P/E ratio for companies in that industry. Second, analysts calculate an appropriate stock price for firms in the industry by multiplying each firm's earnings per share (EPS) by theindustry average P/E ratio. See Equation 9-9 in the text.

8. Compare and contrast the book value and liquidation value per sharefor common stock. Is one method more reliable? Explain.

The Book Value of a firm's common stock is found by subtracting thevalue of the firm's liabilities, and preferred stock, if any, as recorded on the balance sheet, from the value of its assets. The result is the book value or net worth of the company's common stock. To find the book value per share of common stock, divide the company's book value by the number of outstanding common stock shares. See Equation 9-10 in the text.

The liquidation value and book value valuation methods are similar,except that the liquidation method uses the market values of the assets and liabilities, not book values. The market values of the assets are the amounts the assets would earn on the open market if they were sold (or liquidated). The market values of the liabilities are the amounts of money it would take to pay off the liabilities.

Since it is based on market values, the liquidation value method ismore reliable than the book value method. However, liquidation value is a worst-case valuation assessment. A company's common stock should be worth at least the amount generated per share at liquidation.

9. Answer the following questions about the discounted free cash flow model illustrated in Figure 12-4:

a. What are “free cash flows?”

131

Free cash flows represent the total cash flows from business operations that are available to be distributed to the suppliers ofa firm’s capital each year either in the form of interest to the debt holders, or dividends to the stockholders.

b. Explain the terminal value calculation at the end of the forecastperiod. Why is it necessary?

The firm whose business operation is being valued is not expected to suddenly cease operating at the end of the discrete forecasting period, but to continue operating indefinitely into the future as agoing concern. The terminal value calculation estimates the valuesof the cash flows that occur in the year following the discrete forecasting period and beyond.

c. Explain the term “present value of the firm’s operations” (alsoknown as Enterprise Value). What does this number represent?

The present value of the company’s free cash flows represents the market value of the firm’s core income producing operations. In the world of finance and investing this is sometimes called the firm’s Enterprise Value. It is not the total market value of the entire company, however, or the total market value of the company’sassets, because the current, or non-operating assets of the companyhave not yet been accounted for.

d. Explain the adjustments necessary to translate enterprise value tothe total present value of common equity.

To obtain the value of the company’s common stock, add the value ofthe firm’s current assets to the enterprise value (this producesthe value of the firm’s total assets). Next, subtract the valuesof current liabilities, long-term debt, and preferred stock. Theresult is the present value of common equity.

10. Explain the difference between the discounted free cash flow model as it is applied to the valuation of common equity and as it is applied to the valuation of complete businesses.

132

The Free Cash Flow Model values the complete business as a part ofthe procedure to value common equity. The value of a completebusiness is the sum of the values of the operating, or income-producing assets, plus the value of the non-operating, or currentassets. All that is necessary to use the Free Cash Flow Model tovalue a complete business, then, is to add the value of thecompany’s operations to the value of the company’s current assets.

11. Why is the replacement value of assets method not generally used to value complete businesses?

The replacement value of assets method is not often applied to complete business valuations because it is frequently very difficult to locate similar assets for sale on the open market, andbecause some of a business’s assets are difficult to define and quantify.

Answers to End-of-Chapter Problems

12-1. a ) $1,000 X .06 = $60b ) $60 X [(1-1/1.0810)/.08] + $1,000 X [1/1.0810] = $865.80c ) Yes.

12-2. a ) $1,000 X .12 = $120b ) $120 X [(1-1/1.0815)/.08] + $1,000 X [1/1.0815] =

$1,342.38c ) $60 X [(1-1/1.0430)/.04] + $1,000 X [1/1.0430] = $1,345.84

12-3. 3 X $2,000 = $6,000

12-4. Semi-annual interest payment = .10 X $1,000 X 6/12 = $50Price = $50 X [(1-1/1.0410)/.04] + $1,000 X [1/1.0410] = $1,081.11

133

12-5.Since $1,100 > $1,000, YTM < Coupon Rate; YTM < 9%$90 * [1-1/ (1 + k)10/k] + $1,000 * (1/1 + k10) = $1,100k = 7.54%

12-6. a ) Since, $1,125 > $1,000, YTM < Coupon Rate YTM < 12%b ) $120 X [(1-1/1.1010)/.10] + $1,000 X [1/1.1010] = $1,122.89; YTM 10%c ) YTM = 12%; YTM = Coupon Rate if Market Price = Par

12-7. Since $872 < $1,000, k > 7%$70 X [(1-1/(1+k)10/k] + $1,000 X [1/(1+k)10] = $872If k=8%, VB = $932.90If k=9%, VB = $871.65. So, k 9%.

12-8. $10/0.12 = $83.33 per share

12-9. kP = $1.75 /$ 20 = 0.0875 or 8.75%

12-10. a ) VP = $8/0.13 = $61.54 per shareb ) kP = $8/$50 = 16 %

12-11. a ) $4/(.16-.01) = $26.67b) $4/$26.67 = 15%

12-12. $2/$15 + .04 = 17.33%

12-13. a ) P0 = $8/(.14 - .03) = $72.72b ) ks = $8/$65 + .03 = 15.31%

12-14. $90 * [1-1/ (1.12)5/0.12] + $1,000 * (1/1.125) = $891.86

134

12-15. $35 * [1-1/ (1.055)20/0.055] + $1,000 * (1/1.05520) = $761

12-16. $80/ (1 + 0.23)1 = $65.04 $150/ (1 + 0.23)2 = $99.15 $1,500/ (1 + 0.23)3 = $806.08 $65.04 + $99.15 + $806.08 = $970.27

12-17. $3.00/0.12 = $25.00

12-18. $2.20/(0.18 - 0.09) = $2.20/0.09 = $24.44

12-19. D6 = $1.22 (1+ 0.10) = $1.342 $1.342/ (0.12 – 0.10) = $67.10

$0.70 / (1+0.12)1 + $0.83 / (1+0.12)2 + $0.96 / (1+0.12)3 + $1.09 / (1+0.12)4 + $1.22 / (1+0.12)5 + $67.10 / (1+.12)6

0.625 + 0.66167 + 0.6833 + 0.6927 + 0.6923 + $33.99=$37.34

12-20. D1 = 3.82(1 + 0.07) = 4.09 k = 4.09/82 + 0.07 k = .1199 = 11.99%

12-21. $85 * [1-1/ (1 + k)10/k] + $1,000 * (1/1 + k10) = $1,250 k = 5.23%

12-22. $2,100,000 / (0.18 – 0.09) = $23,333,333.33

12-23. Find the present values of cash flows for each year. Add themtogether to get the present value of the firm.

Year 1 = $1,231,920,000 * [1/ (1 + 0.12)1] = $1,099,928,571Year 2 = $1,453,665,600 * [1/ (1 + 0.12)2] = $1,158,853,316Year 3 = $1,686,252,096 * [1/ (1 + 0.12)3] = $1,200,240,935Year 4 = $1,922,327,389 * [1/ (1 + 0.12)4] = $1,221,673,808

135

Year 5 = $2,153,006,676 * [1/ (1 + 0.12)5] = $1,221,673,809Year 6 CF = Year 5 CF * (1 + .10) = $2,153,006,676 * 1.10 = 2,368,307,344Year 6 - ∞ = (((2,368,307,344 / (0.12 - 0.10)) * (1/1.12)5 ) = $67,192,060,000

Enterprise value of the firm today = $1,099,928,571 + $1,158,853,316 + $1,200,240,935 + $1,221,673,808 + $1,221,673,809 +$67,192,060,000 = $73,094,430,440

12-24. a ) $675,000 - $120,000 = $555,000b ) $555,000/100,000 = $5.55 per share

12-25. a ) Net Worth = $38,400 - ($13,400 + $6,000) = $19,000 (in '000 dollars)

b ) Book Value = $19,000,000/500,000 = $38 per share c ) EPS = $5,610,000/500,000 = $11.22

d ) Stock Price = EPS X P/E ratio = $11.22 X 6 = $67.32e ) Since $67.32 (the stock price) > $38.00 (the book value), the

firm seems to have going-concern value.f ) ($50,000,000 - $20,000,000)/500,000 = $60 per share.

12-26. a) Corporate Bond

Let YTM = k$130 X [(1-1/(1+k)16)/k] + $1,000 X [1/(1+k)16] = 1,147.58Solving, kd = 11%

b) Preferred Stock

k = $14/$140 kp = 10%

c) Common Stock

Let ks be the required rate of return for a similar common stockks = D1/Po + g = $39/$300 + .03 = .16 = 16%

136

Remember that these are three different companies. The cost of preferredstock for Supernova could be lower than the cost of debt for Star, as suggested by the above numbers. Lucky should choose the alternative that, in his opinion, gives the best return/risk tradeoff. There is no clear answer here as to which investment is the best.

12-27.The Nonconstant, or Supernormal Dividend Growth Model

Flash in the Pan Corporation

Given:Year Year Year Year Year Yea

r1 2 3 4 5 6 and

onDividend growth rates 20% 30% 20% 10% 5%

Dividend expected in 1 year $3.00 Assumed required rate of

return15%

Calculations:

a. Present value of Dividends during the supernormal growth period:

Expected future dividends duringthe supernormal growth period $3.00 $3.60 $4.68 $5.62 $6.18

Present values of dividends duringthe supernormal growth period $2.61 $2.72 $3.08 $3.21 $3.07

Total $14.69

b. Present value of dividends during the normal growth period(year 6 and on)

Terminal value at end of year 5per Equation 12-7 $64.8

6

Present value of terminal $32.25

137

value

c. Total present value per share of Flash in the Pan Corp. stock

$46.94

138

12-28. The Discounted Free Cash Flow Model for Total Common Equity

Hardi-Pets Corporation

Forecasting Variables:2007 2008 2009 2010 2011 2012 2013 2014 2015 2016

Revenue growth factor 10% 15% 20% 25% 30% 25% 20% 15% 10% 5%Expected gross profit margin 50% 50% 50% 50% 50% 50% 50% 50% 50% 50%

S, G, & A expense % of revenue 20% 20% 20% 20% 20% 20% 20% 20% 20% 20%Depr. & Amort. % of revenue 10% 10% 10% 10% 10% 10% 10% 10% 10% 10%

Capital expenditure growth factor 10% 10% 10% 10% -10% -10% -10% -10% -10% -10%Net working capital to sales ratio 10% 10% 10% 10% 10% 10% 10% 10% 10% 10%

Income tax rate 40%Assumed long-term sustainable growth

rate5% per year after

2016Discount rate 20%

FORECAST: Years Ending December 31Actual |--------------------------------------------------------- Forecast

----------------------------------------------------------------------------------|2006 2007 2008 2009 2010 2011 2012 2013 2014 2015 2016

Total revenue $1,000,000 $1,100,000 $1,265,000

$1,518,000

$1,897,500

$2,466,750

$3,083,438

$3,700,125

$4,255,144

$4,680,658

$4,914,691

Cost of Goods Sold 500,000 550,000 632,500 759,000 948,750 1,233,375

1,541,719

1,850,063

2,127,572

2,340,329

2,457,346

Gross profit 500,000 550,000 632,500 759,000 948,750 1,233,375

1,541,719

1,850,062

2,127,572

2,340,329

2,457,345

Selling, general and administrative expenses

200,000 220,000 253,000 303,600 379,500 493,350 616,688 740,025 851,029 936,132 982,938

Earnings before interest, taxes, depr. & amort. (EBITDA)

300,000 330,000 379,500 455,400 569,250 740,025 925,031 1,110,037

1,276,543

1,404,197

1,474,407

Depreciation and amortization 100,000 110,000 126,500 151,800 189,750 246,675 308,344 370,013 425,514 468,066 491,469

139

Earnings before Interest and taxes (EBIT)

200,000 220,000 253,000 303,600 379,500 493,350 616,687 740,024 851,029 936,131 982,938

Federal and State Income Taxes 80,000 88,000 101,200 121,440 151,800 197,340 246,675 296,010 340,412 374,452 393,175 Net Operating Profit After-Tax (NOPAT) 120,000 132,000 151,800 182,160 227,700 296,010 370,012 444,014 510,617 561,679 589,763

Add back depreciation and amortization 100,000 110,000 126,500 151,800 189,750 246,675 308,344 370,013 425,514 468,066 491,469 Subtract Capital Expenditures (15,000) (16,500) (18,150) (19,965) (21,962) (19,766) (17,789) (16,010) (14,409) (12,968) (11,671)Subtract New Net Working Capital (10,000) (16,500) (25,300) (37,950) (56,925) (61,669) (61,669) (55,502) (42,551) (23,403)Free Cash Flow $205,000 $215,500 $243,650 $288,695 $357,538 $465,994 $598,898 $736,348 $866,220 $974,226 $1,046,1

58

Terminal value, 2016 $7,323,106

Present Value of Free Cash Flows @ 20% 179,583 169,201 167,069 172,424 187,273 200,570 205,501 201,455 188,812 1,351,683

Total Present Value of Company Operations

$3,023,571

Plus Current Assets 100,000 from Hardi-Pets December 31, 2006 Balance SheetLess Current Liabilities (80,000) from Hardi-Pets December 31, 2006 Balance Sheet

Less Long-Term Debt (500,000) from Hardi-Pets December 31, 2006 Balance SheetLess Preferred Stock 0 from Hardi-Pets December 31, 2006 Balance Sheet

Net Market Value of Common Equity $2,543,571

12-29. The Discounted Free Cash Flow Model for a Complete Business

Great Expectations Company

Forecasting Variables:2007 2008 2009 2010 2011 2012 2013 2014 2015 2016

Revenue growth factor 20% 30% 40% 50% 60% 50% 40% 30% 20% 10%Expected gross profit margin 50% 51% 52% 53% 54% 55% 56% 57% 58% 59%

S, G, & A expense % of revenue 50% 40% 30% 29% 28% 27% 26% 25% 24% 23%Depr. & Amort. % of revenue 10% 10% 10% 10% 10% 10% 10% 10% 10% 10%

Capital expenditure growth factor 40% 35% 30% 25% 20% -10% -15% -20% -25% -30%Net working capital to sales ratio 19% 18% 17% 16% 15% 14% 13% 12% 11% 10%

140

Income tax rate 40%Assumed long-term sustainable

growth rate5% per year after

2016Discount rate 20%

FORECAST: Years Ending December 31Actual |---------------------------------------------------------------------- Forecast

-----------------------------------------------------------------------------|2006 2007 2008 2009 2010 2011 2012 2013 2014 2015 2016

Total revenue $2,000,000 $2,400,000

$3,120,000

$4,368,000

$6,552,000

$10,483,200

$15,724,800

$22,014,720

$28,619,136

$34,342,963

$37,777,260

Cost of Goods Sold 1,200,000 1,200,000

1,528,800

2,096,640

3,079,440 4,822,272 7,076,160 9,686,477 12,306,22

8 14,424,04

5 15,488,67

6 Gross profit 800,000 1,200,00

0 1,591,20

0 2,271,36

0 3,472,56

0 5,660,928 8,648,640 12,328,24

3 16,312,90

8 19,918,91

8 22,288,58

4

Selling, general and administrative expenses

1,200,000 1,200,000

1,248,000

1,310,400

1,900,080 2,935,296 4,245,696 5,723,827 7,154,784 8,242,311 8,688,770

Earnings before interest, taxes, depr. & amort. (EBITDA)

(400,000) 0 343,200 960,960 1,572,480 2,725,632 4,402,944 6,604,416 9,158,124 11,676,60

7 13,599,81

4

Depreciation and amortization 200,000 240,000 312,000 436,800 655,200 1,048,320 1,572,480 2,201,472 2,861,914 3,434,296 3,777,726 Earnings before Interest and taxes (EBIT) (600,000) (240,000

)31,200 524,160 917,280 1,677,312 2,830,464 4,402,944 6,296,210 8,242,311 9,822,088

Available tax-loss carryforwards 0 (600,000)

(840,000)

(808,800)

(284,640)

0 0 0 0 0 0

Net taxable earnings 0 0 0 0 632,640 1,677,312 2,830,464 4,402,944 6,296,210 8,242,311 9,822,088

Federal and State Income Taxes 0 0 0 0 253,056 670,925 1,132,186 1,761,178 2,518,484 3,296,924 3,928,835 Net Operating Profit After-Tax (NOPAT) (600,000) (240,000

)31,200 524,160 664,224 1,006,387 1,698,278 2,641,766 3,777,726 4,945,387 5,893,253

Add back depreciation and amortization 200,000 240,000 312,000 436,800 655,200 1,048,320 1,572,480 2,201,472 2,861,914 3,434,296 3,777,726 Subtract Capital Expenditures (1,000,000

)(1,400,0

00)(1,890,0

00)(2,457,0

00)(3,071,2

50)(3,685,50

0)(3,316,95

0)(2,819,40

8)(2,255,52

6)(1,691,64

5)(1,184,15

1)Subtract New Net Working Capital 76,000 129,600 212,160 349,440 589,680 733,824 817,690 792,530 629,621 343,430 Free Cash Flow ($1,400,00

0)($1,324,

000)($1,417,

200)($1,283,

880)($1,402,

386)($1,041,1

13)$687,632 $2,841,52

1 $5,176,64

4 $7,317,65

9 $8,830,25

7

Terminal value, 2016 $61,811,7

141

99

Present Value of Free Cash Flows @ 20% (1,103,333)

(984,167)

(742,986)

(676,305)(418,400) 230,287 793,016 1,203,922 1,418,211 11,409,08

6

Total Present Value of Company Operations $11,129,331

Plus Current Assets 500,000 from Great Expectations' December 31, 2006 Balance Sheet

Total Market Value of Great Expectations'Assets

$11,629,331

142

Chapter 13 Solutions

Answers to Review Questions

1. What is the operating leverage effect and what causes it? What arethe potential benefits and negative consequences of high operating leverage?

The operating leverage effect is the phenomenon whereby a small change in sales triggers a relatively large change in operating income. It is caused by the presence of fixed operating costs. The potential benefits are that if sales are rising operating income will rise more quickly. The negative consequences are that falling sales will cause operating income to fall more quickly, including negative values.

2. Does high operating leverage always mean high business risk? Explain.

High operating leverage does not always mean high business risk. If the companies sales are quite stable then the variation in operating income would be small even if the degree of operating leverage were large.

3. What is the financial leverage effect and what causes it? What arethe potential benefits and negative consequences of high financial leverage?

Financial leverage is the additional volatility of net income caused by the presence of fixed-cost funds. The potential benefits are that if operating income is rising net income will rise more quickly. The negative side is that if operating income is falling net income will fall more quickly, including possibly negative values.

143

4. Give two examples of types of companies likely to have high operating leverage. Find examples other than those cited in the chapter.

Long distance telephone companies and electricity generating companies are likely to have operating leverage. These two types of companies have very high fixed costs, because they are capital intensive, and have relatively low variable costs.

5. Give two examples of types of companies that would be best able to handle high debt levels.

Companies that handle local telephone service and those that handlenatural gas delivery to consumers would be expected to comfortably be able to handle high debt levels. This is because the sales of these two types of companies tend not to react very much to the business cycle. Their sales tend to grow with the population. They are often regulated and protected from competition, although this is not so much true as it was a few years ago.

6. What is an LBO? What are the risks for the equity investors and what are the potential rewards?

A leveraged buyout is a purchase of a publicly owned corporation bya small group of investors using a large amount of borrowed money. The risks for the equity investors are those that exist whenever a high degree of financial leverage exists. So too are the rewards, where small returns become large returns because of leverage.

7. If an optimal capital structure exists, what are the reasons why too little debt is as undesirable as is too much debt?

Too little debt may be as undesirable as too much debt because if a firm has a very conservative capital structure it may be losing the opportunity to reap the positive benefits of financial leverage. A company with a bright future is probably not maximizing shareholder wealth if it has a very small amount of debtin its capital structure. A more aggressive capital structure may create more value for the owners.

144

Answers to End-of-Chapter Problems

13-1.a) Breakeven sales per month = $2,300/($50 – 5.75) = 51.98 unitsb) New Breakeven sales per month = $2,300 x .7 / ($45 – 5.75) =

41.02 units

13-2.a) 30 x $125 + 30 x $90 + 30 x 55 = $8,100b) $10,000 + 90 x $15 = $11,350c) 10 x $125 + 15 x $90 + 35 x $55 = $4,525d) $10,000 + 60 x $15 = $10,900

13-3.DOL = (17,900,000 – 9,220,000) / 9,220,000 ÷ (25,000,000 – 15,000,000) / 15,000,000 = 1.41

or DOL = (15,000,000 – 1,980,000) / (15,000,000 – 1,980,000 – 3,800,000) = 1.41

13-4.DOL = (11,333,000 – 5,257,000) / 5,257,000 ÷ (17,900,000 – 9,220,000) / 9,220,000 = 1.23

or DOL = 9,220,000 / (9,220,000 – 1,710,000) = 1.23

13-5.a) Contribution Margin = $28 - $16 = $12

b) Unit Sales b.e. = $20,000/($28 - $16) = 1,666.67 units; 1,667 units rounded up

DOLLARS b.e = $28 x 1,667 units = $46,676

c) (i) Operating profit (loss) = 1,500 units X $12/unit - $20,000= ($2,000)

(ii) Operating profit (loss) = 3,000 units X $12/unit - $20,000 = $16,000

145

d)

13-6. a) Contribution Margin = $28 - $20 = $8

b) Unit Sales b.e. = $10,000/($28 - $20) = 1,250 unitsDOLLARS b.e = $28 x 1,250 units = $35,000

c) (i) Operating profit (loss) = 1,500 units X $8/unit - $10,000 = $2,000

(ii) Operating profit (loss) = 3,000 units X $8/unit - 10,000 = $14,000

d)

146

$0$20,000$40,000$60,000$80,000$100,000$120,000$140,000$160,000$180,000

0 500 1,000

1,500

2,000

2,500

3,000

3,500

4,000

4,500

5,000

5,500

6,000

FIXED COST VAR. COST TOT. COST REVENUE

e) Howard Beal Co., having higher fixed costs, and a lower variable cost per unit, has a higher profit potential once they break-even. However, they have a greater loss potential, and need to achieve a highersales level to break even, because of the high fixed costs.

13-7. YEAR 2006 2007 SALES IN UNITS 3000 3300SALES IN DOLLARS $84,000 $92,400 VAR.COST, $16/unit $48,000 $52,800 FIXED COST $20,000 $20,000 OP. INCOME (EBIT) $16,000 $19,600 INTEREST EXP. $2,000 $2,000 EBT $14,000 $17,600 TAX @30% $4,200 $5,280 NET INCOME $9,800 $12,320

%CHANGE IN SALES 10.00%%CHANGE IN EBIT 22.50%%CHANGE IN NI 25.71%

a) Percentage change in operating income = ($19,600 - $16,000)/$16,000= 22.5%

Percentage change in sales = ($92,400 - $84,000)/$84,000 = 10%

147

$0$20,000$40,000$60,000$80,000$100,000$120,000$140,000$160,000$180,000

0 500 1,000 1,500 2,000 2,500 3,000 3,500 4,000 4,500 5,000 5,500 6,000

FIXED COST VAR. COST TOT. COST REVENUE

b) Due to presence of fixed costs a given percentage change in sales gives a higher percentage change in operating income (EBIT) (10% and 22.5% respectively). This is the operating leverage effect.

c) (i) DOL = % EBIT/% SALES = [($19,600 - $16,000)/$16,000]/[($92,400 -$84,000)/$84,000] = 22.5%/10% = 2.25(ii) DOL = (SALES-VC)/(SALES-VC-FC) = ($84,000 - $48,000)/($84,000 - $48,000-$20,000) = $36,000/$16,000 = 2.25

d) (i) shows the effect of operating leverage -- EBIT varies at a larger percentage than sales.(ii) pinpoints the source of operating leverage -- fixed operating

costs.

13-8. a) Percentage change in NI = ($12,320 - $9,800)/$9,800= 25.71%Percentage change in operating income = ($19,600 - $16,000)/$16,000

= 22.5%

b) Due to presence of fixed interest expense a given percentage change in EBIT gives a higher percentage change in net income (22.5% and 25.71% respectively). This is the financial leverage effect.

c) (i) DFL = % NI/% EBIT = [($12,300 - $9,800)/$9,800]/[($19,600 - $16,000)/$16,000] = 25.71%/22.5% = 1.14(ii) DFL = EBIT/(EBIT - I) = $16,000/($16,000 - $2,000) = 1.14

d) (i) shows the effect of financial leverage -- NI varies by a largerpercentage than operating income (EBIT).(ii) pinpoints the source of financial leverage -- fixed interest

expense.

13-9. a ) $9,000/($15 - $1.50) = 666.67 sq. yards

b ) Break-even point in sales units

c ) 666.67 X $15 = $10,000

148

d ) $9,000/($18 - $1.50) = 545.5 sq. yards, break-even units545.5 X $18 = $9,819, break-even dollar sales

e)SALES IN UNITS 14,000 sq.yardsSALES IN DOLLARS, units x $18 each $252,000 VAR.COST, units x $1.50 each $21,000 FIXED COST $9,000 OP. INCOME (EBIT) $222,000 INTEREST EXP. $3,000 EBT $219,000 TAX @40% $87,600 NET INCOME $131,400

13-10. a ) Contribution Margin = $800 - $250 = $550 per unitSale of 600 suits: Op. Income = 600 X $550 - $200,000 = $130,000Sale of 3,000 suits: Op. Income = 3,000 X $550 - $200,000 =

$1,450,000

b ) Sale of 600 suits: DOL= [600 X ($800 - $250)]/[600 X ($800 - $250) - $200,000] = 2.5

Sale of 3,000 suits: DOL = [3,000 X ($800 - $250)]/[3,000 X ($800 - $250) - 200,000] = 1.1

c ) Unit Sales b.e. = $200,000/$550 = 363.64 units; rounded up to 364

DOLLARS b.e = 364 suits X $800 price per suit = $291,200

d ) Unit Sales b.e. = $200,000/($800 - $350) = 444.44 units; roundedup to 445

DOLLARS b.e = 445 suits X $800 price per suit = $356,000

e ) Let P be the selling price per unit.3,000 units X (P - $350) - $200,000 fixed costs = $1,450,000

op. incomeP - $350 = ($1,450,000 + $200,000)/$3,000 = $550P = $550 + $350 = $900Tom should increase price per unit by $100 ($900 - $800)

13-11. COMPANY A COMPANY B COMPANY CSALES IN UNITS 12,000 12,000 12,000SALES IN DOLLARS, units x $10 each $120,000 $120,000 $120,000VARIABLE COST, $5, $4, and $1 per

149

unit respectively $60,000 $48,000 $12,000FIXED COST $0 $10,000 $40,000OPERATING INCOME (EBIT) $60,000 $62,000 $68,000

b ) C, B, A.

13-12. a ) Year 1: $30 X 50,000 = $1,500,000Year 1: $30 X 60,000 = $1,800,000

b ) ($1,800 - $1,500)/$1,500 = 0.2 or 20%

c ) METHOD 1 METHOD 2

YEAR 1 YEAR 2 YEAR 1 YEAR 2UNITS 50,000

60,000 50,000

60,000SALES, units x $29 each

1,500,000 1,800,000 1,500,000 1,800,000

FC 700,000 700,000 100,000 100,000

VC, units x $6 each for Method 1 and x $16.50 each for Method 2

300,000 360,000 825,000 990,000

EBIT $500,000 $740,000 $575,000 $710,000

d ) METHOD 1: % EBIT = ($740,000 - $500,000)/$500,000 = 0.48 OR 48% METHOD 2: % EBIT = ($710,000 - $575,000)/$575,000 = 0.235 OR

23.5%

e ) METHOD 1: DOL = 0.48/0.20 = 2.4METHOD 2: DOL = 0.235/0.20 = 1.175

f ) METHOD 1: DOL = ($1,500,000 - $300,000)/($1,500,000 - $300,000- $700,000) = 2.4METHOD 2: DOL = ($1,500,000 - $825,000)/($1,500,000 - $825,000 -

$100,000)= 1.175

g ) METHOD 1

150

h ) The high fixed operating costs

i ) METHOD 1 METHOD 2

YEAR 1 YEAR 2 YEAR 1 YEAR 2UNITS 50,000

53,000 50,000

53,000SALES, units x $30 each

1,500,000 1,590,000 1,500,000 1,590,000

FC 700,000 700,000 100,000 100,000

VC, units x $6 each for Method 1 and x $16.50 each for Method 2

300,000 318,000 825,000 874,500

EBIT $500,000 $572,000 $575,000 $615,500

% SALES = ($53,000 - $50,000)/$50,000 = .06 or 6%

METHOD 1: % EBIT = ($572,000 - $500,000)/$500,000 = 0.144 or 14.4%

METHOD 2: % EBIT = ($615,500 - $575,000)/$575,000 = 0.0704 or 7.04%

METHOD 1: DOL = 0.144/0.06 = 2.4 METHOD 2: DOL = 0.0704/0.06 = 1.17

13-13. a ) C, B, A.

b ) COMPANY A: DFL = $100,000/($100,000 - $0) = 1.0COMPANY B: DFL = $100,000/($100,000 - $2,000) = 1.02COMPANY C: DFL = $100,000/($100,000 - $40,000) = 1.67

Answer to part a) was correct.

c ) COMPANY A COMPANY B COMPANY C

CAPITAL STRUCTURE ALL EQUITY 90% EQUITY 10% EQUITYEBIT $100,000 $100,000 $100,000INTEREST EXP. $0 $2,000 $40,000EBT $100,000 $98,000 $60,000TAXES @40% $40,000 $39,200 $24,000NET INCOME $60,000 $58,800 $36,000

151

13-14.a.

MICHAEL DORSEY DOROTHY MICHAELS

YEAR 1 YEAR 2 YEAR 1 YEAR 2

EBIT $50,000 $60,000 $50,000 $60,000 INTEREST EXPENSE $9,100 $9,100 $900 $900

EBT $40,900 $50,900 $49,100 $59,100 TAXES @40% $16,360 $20,360 $19,640 $23,640 NET INCOME $24,540 $30,540 $29,460 $35,460

b.%CHANGE in NI 24.45% 20.37%

c.%CHANGE in EBIT 20.00% 20.00%

d.DFL 1.22 1.02

e.DFL 1.22 1.02

f.MICHAEL DORSEY'S COMPANY

g.HIGHER DFL DUE TO GREATER AMOUNT OF INTEREST EXPENSE

h.

MICHAEL DORSEY DOROTHY MICHAELS

YEAR 1 YEAR 2 YEAR 1 YEAR 2

EBIT $50,000 $53,000 $50,000 $53,000 INTEREST EXPENSE $9,100 $9,100 $900 $900

EBT $40,900 $43,900 $49,100 $52,100 TAXES @40% $16,360 $17,560 $19,640 $20,840 NET INCOME $24,540 $26,340 $29,460 $31,260

DFL 1.22 1.02

13-15. DCL = (200,000 – 75,000) / 75,000 ÷ (400,000 – 230,000) / 230,000 = 2.25

13.16.

152

FUNNY GIRLS COMICS

153

YEAR 1 YEAR 2

SALES $200,000 $225,000

EBIT $95,000

NET INCOME $30,000

DOL 1.35

DFL 1.09

a.

%CHANGE in SALES = 12.50%

%CHANGE in EBIT = DOL X %CHANGE in SALES 16.8750%

b.

EBIT IN YEAR 2 =1.1688*95,000 =

$111,031.25

c.

%CHANGE in NI = DFL X %CHANGE in EBIT 18.3938%

d.

NI in YEAR 2 = 1.1839 X30,000 =

$35,518.13

e.

DCL = DOL X DFL=

1.4715

f.

%CHANGE in SALES = 20.00%

%CHANGE in NI = DCL X %CHANGE in SALES 29.43%

NI in YEAR 2 = 1.2943 X 30,000 =

$38,829.00

13-17. Interest Expense = $2,000,000 X 0.10 = $200,000 DCL = DOL X DFL =1.4 X [$600,000/($600,000 - $200,000)= 1.4 X 1.5= 2.1

13-18. a) DOL = ($5,000,000 - $700,000)/($5,000,000 - $700,000 - $300,000) = 1.075

b) Interest Expense = $16,666,666.67 X 0.09 = $1,500,000EBIT = $2,500,000 + $1,500,000 = $4,000,000

154

DFL = $4,000,000/($4,000,000 - $1,500,000) = 1.600Also, DFL = DCL/DOL = 1.720/1.075 = 1.600

c) %NI = $Sales x DCL %NI = 20% x 1.72

= 34.4%

13.19.Soccer International, Inc.

Given:2005 2006

Sales $560,000 $616,000 Variable Costs $240,000 $264,000

Fixed Costs $160,000 $160,000

Interest Expense $40,000 $40,000

Price of each soccerball

$16

a. Completed income statements:2005 2006

Sales $560,000 $616,000 Variable Costs $240,000 $264,000

Fixed Costs $160,000 $160,000 EBIT $160,000 $192,000

Interest Expense $40,000 $40,000 EBT $120,000 $152,000

Income Taxes (30%) $36,000 $45,600 Net Income $84,000 $106,400

b. Breakeven point in units:

2005 2006

Number of balls sold 35,000 38,500 Variable cost per ball $6.86 $6.86

Contribution margin $9.14 $9.14

Breakeven point inunits

17,500 17,500

155

c. Breakeven point in dollars:

2006 2007

Breakeven point indollars

$280,000 $280,000

d. Unit sales required to produce $200,000 in operating income in 2005:

Fixed costs $160,000 Operating profit

requirement$200,000

Total dollars needed $360,000 Contribution margin, each

ball$9

Number of balls needed tobe sold

39,375

e. Effect on operating profit of greater or lesser sales in 2005:

Assumed number of balls sold 18,000

24,000

Total contributionmargin

$164,571 $219,429

Fixed costs $160,000 $160,000 Operating profit $4,571 $59,429

f. Degree of Operating Leverage (DOL):2005 2006

DOL 2.0 1.83

h. Degree of financial leverage (DFL):2005 2006

DFL 1.33 1.26

j. Degree of combined leverage (DCL):2005 2006

DCL 2.67 2.32

k. Effect of a price increase that produces higher sales:2005 2006

Sales $560,000 $650,000 given

16.1%

% increase in net 42.86%

156

income

Net income in dollars $84,000 $120,000

13-20. %NI = % x DFL%NI = ($50,000 - $35,000)/$35,000 x 1.71%NI = $15,000/$35,000 x 1.71%NI = .429 x 1.71%NI = .733, or 73.3%

157

Chapter 14 Solutions

Answers to Review Questions

1. How does a mortgage bond compare to a debenture?

A mortgage bond is a secured bond while a debenture is an unsecuredbond.

2. How does a sinking fund function in the retirement of an outstanding bond issue?

A sinking fund is where a company puts payments that are then used to buy back outstanding bonds.

3. What are some examples of restrictive covenants that might be specified in a bond’s indenture?

An indenture might include limitations on future borrowings, restrictions on dividend payments, and/or requirements that workingcapital be maintained at least at some minimum level.

4. Define the following terms that relate to a convertible bond: conversion ratio, conversion value, and straight bond value.

The conversion ratio is the number of shares of common stock that would be obtained if a convertible bond were converted. The conversion value is the total value of the common stock that would be obtained. The straight bond is the value a convertible bond would have without the conversion feature.

5. If a convertible bond has a conversion ratio of 20, a face value of$1,000, a coupon rate of 8 percent, and the market price for the company’s stock is $15 per share, what is the convertible bond’s conversion value?

158

The conversion value would equal the conversion ratio of 20 times the $15 market price of the stock or $300.

6. What is a callable bond? What is a putable bond? How do each of these features affect their respective market interest rates?

A callable bond can be retired early at the discretion of the issuer. A putable can be retired early at the discretion of the investor. A call provision increases the market interest rate and a put provision decreases it.

Answers to End-of-Chapter Problems

14-1. VB = $80 X [(1 - 1/1.1210)/.12] + $1,000 X 1/1.1210 = $773.99

14-2. VB = $40 X [(1 - 1/1.0620)/.06] + $1,000 X 1/1.0620 = $770.60

14-3. Conversion Value = $60 X 20 = $1,200

14-4.$32 * 26.5 = $848$848 * 6 = $5,088.00

14-5.22.5 [ 1-(1/(1.0375)60)/0.0375] + 1,000/(1.0375)60

22.5 * 23.7379 + 109.828 = $643.93

14-6.$85 * 30 = $2,550

14-7. 33.75 [1 – (1/1.0430)/0.04] + 1,000/1.0430 = $891.92

159

14-8. Funds required to buy 1,000 bonds from the open market = $800 X 1,000 = $800,000. Therefore savings from buying the bonds back instead of depositing $1 million in the sinking fund = $1,000,000 -$800,000 = $200,000.

14-9.(7.0% - 5.0%) * 30,000 * 1,000 = $600,000

14-10. Yearly savings = (10% - 8%) X 20,000 X $1,000 = $400,000

14-11. Face Value + Call Premium = $1,000 + 0.5 X $1,000 = $1,050Annual interest paid over last ten years = 0.10 X $1,000 = $100$950 = $100 X (PVIFAk,10) + $1,050 X (PVIFk,10)Realized return for Brooks = k = 11.15%

YEARS0 1 2 3 4 5 6 7 8 9 10 1

1 12 13 14

15 16

17 18 19

20

-950 100 100 100 100 100 100 100 100 100 100

1050

-950 100 100 100 100 100 100 100 100 100 115

0

11.15%

IRR

14-12. a) Annual interest to be paid over next ten years = 0.08 X $1000

= $80$950 = $80 X (PVIFAk,10) + $1,000 X (PVIFk,10)Return for Brooks for the newly issued bond = k = 8.77%Overall return if the is bond held to maturity = 10.52% (See table below)

b) Return on the bond in Problem #4 if they had not been called =10.61% (See table below). Brooks didn't welcome the recall (10.61% > 10.52%).

YEARS

160

0 1 2 3 4 5 6 7 8 9 10 11 12 13 14 15 16 17 18 19 20 PROB 14-

12a-950 80 80 80 80 80 80 80 80 80 80

1000

-950 80 80 80 80 80 80 80 80 80 1080

8.77% IRR

-950 100 100 100 100 100 100 100 100 100 100 80 80 80 80 80 80 80 80 80 80

1050

1000

-950

-950 100 100 100 100 100 100 100 100 100 200 80 80 80 80 80 80 80 80 80 108

0 10.52

%IRR

PROB 14-12b

-950 100 100 100 100 100 100 100 100 100 100 10

0 100 100 100 100 100 100 100 100 100

1000

-950 100 100 100 100 100 100 100 100 100 100 10

0 100 100 100 100 100 100 100 100 1100

10.61%

IRR

14-13. Conversion Value = $70 X 20 = $1,400

14-14. VB = $90 X [(1 - 1/1.0714)/.07] + $1,000 X 1/1.0714 = $1,174.90

He would consider converting, but since the market value of the convertible bond would be greater than the larger of the conversionvalue or straight bond value he would sell the bond instead if he wanted to cash out.

14-15. Conversion Value = $30 X 30 = $900VB = 110 X [(1 - 1/1.135)/.13] + 1,000 X 1/1.135 = $929.66

161

No, he should not convert. The straight bond value is greater thanthe conversion value.

14-16. VB = $90 X [(1 - 1/1.1314)/.13] + $1,000 X 1/1.1314 = $747.90

Since the putable bond can be redeemed at a higher price, i.e., $900, Ms. Carter should redeem the bond.$1,000 = $90 X (PVIFAk,6) + $900 X (PVIFk,6)Realized return for Ms. Carter = k = 7.62%

14-17. VB = $90 X [(1 - 1/1.145)/.14] + $1,000 X 1/1.145 = $828.34

Since the bond can be redeemed at a higher price, i.e., $900, Dianashould redeem the bond.

$1,000 = $90 X (PVIFAk,5) + $900 X (PVIFk,5)Realized return for Diana from original bond = k = 7.27%

$900 = $130 X (PVIFAk,5) + $1,000 X (PVIFk,5)Realized return for Diana from new bond = k = 16.06%

YEARS 0 1 2 3 4 5 6 7 8 9 10 -1000 90 90 90 90 90

900 -900 130 130 130 130 130

1000 CF -1000 90 90 90 90 90 130 130 130 130 1130

0.1051 =10.51%IRR

Realized overall return for Diana = k = 10.51%

14-18. VB = $80 X [(1 - 1/1.2510)/.25] + 1,000 X 1/1.2510 = $393.01

14-19. Claim Received

1st Mortgage bonds $5 million 5 million

162

2nd Mortgage bonds 5 million 5 millionSenior Debentures 10 million 10 millionSubordinated Debentures 4 million 0Common Stock 10 million 0

Total34 million 20 million

14-20. a) Call Premium paid $60,000,000 * .04 = $2,400,000 New Bond Underwriting Costs $60,000,000 * .03 = $1,800,000 Total incremental Cash Outflow $4,200,000 b) Savings = (8% - 6%) = 2% annually Total Savings in interest payments = 2% * $60,000,000 =

$1,200,000

c) Interest on old bonds: $60,000,000 * .08 = $4,800,000

Interest on new bonds: $60,000,000 * .06 = $3,600,000 $1,200,000 difference each year for 10 years

Less taxes on the additional income at 40%: $1,200,000 * (1 - .40) = $720,000 Net Savings = $720,000 per year d) Present value of the net savings for 10 years at 3.6% $720,000 * ((1-(1/(1.036)10))/.036) = $720,000 * 8.274844044 =

$5,957,887.71

e) Note: Call premiums are tax deductible and amortized over the life of the bond

$60,000,000 * .04 * .40 = $960,000 Amortized over 10 years = $960,000/10 = $96,000 per year

f) Present Value of the annual tax savings for 10 years: $96,000 * ((1-(1/(1.036)10))/.036) = $96,000 * 8.274844044 =

$794,385.03

163

g) Unamortized amount = $60,000,000 * .02 * (10/20) = $600,000 current deduction

PV of unamortized amount if bond is not called: ($600,000/10) * ((1-(1/(1.036)10))/.036 = $60,000 * 8.274844044 = $496,490.64 Net Tax Savings = $600,000 - $496,490.64 = $103,509.36

h) ($60,000,000 * .03)/10 = $180,000 annual write off Tax Savings = $180,000 * .40 = $72,000

i) PV of tax Savings = $72,000 * ((1-(1/(1.036)10))/.036 = $72,000 * 8.274844044 =

$595,788.77

j) PV Total Inflows = $5,957,887.71 + $794,385.03 + $103,509.36 + $595,788.77 = $7,451,570.87

k) NPV of the bond proposal = PV of total cash inflows – Total outflows

NPV = $7,451,570.87 - $4,200,000 = $3,251,570.87

164

14-21.Aurora Glass Fibers Lease-Buy Analysis

Part a, the buy option:

Assumptions:Cost of new computers $800,000

Expected Life 4 yearsSalvage value $100,000

Amount to be borrowed $800,000 Interest rate on loan 10%

MACRS Depreciation: Yr 1 Yr 2 Yr 3 Yr 4(3-year asset class) 33.3% 44.5% 14.8% 7.4%

Cost of capital 6% (after-tax cost of debt)

Tax rate 40%

Estimated Incremental Cash Flows to Equity:

Year: 0 1 2 3 4Cost of new computers ($800,000)Amount to be borrowed 800,000

Depreciation on new computers ($266,400)

($356,000) ($118,400)

($59,200)

Tax savings on depreciation 106,560 142,400 47,360 23,680 Interest payments on loan (80,000) (80,000) (80,000) (80,000)

Tax savings on interest 32,000 32,000 32,000 32,000 Repayment of principal on loan (800,000)Salvage value of new computers 100,000

Tax on gain (40,000)Net Incremental Cash Flows $0 $58,560 $94,400 ($640) ($764,320

)PV of Cash Flows $0 $55,245 $84,016 ($537) ($605,413

)

Total PV of Cash Flows Associated With the Buy Option = ($466,689)

Part b, the lease option:

Assumptions:Annual lease payment ($200,000) paid at the end of each year

Lease term 4 yearsValue at termination of lease $0

Estimated Incremental Cash Flows to Equity:

Year: 0 1 2 3 4

165

Lease payment ($200,000)

($200,000) ($200,000)

($200,000)

Tax savings on lease payment $80,000 $80,000 $80,000 $80,000 Net Incremental Cash Flows $0 ($120,000

)($120,000) ($120,000

)($120,000

)PV of Cash Flows $0 ($113,208

)($106,800) ($100,754

)($95,051)

Total PV of Cash Flows Associated With the Lease Option = ($415,813)

Part c, comparison of alternatives and decision:

Total PV of Cash Flows Associated With the Buy Option = ($466,689)

Total PV of Cash Flows Associated With the Lease Option = ($415,813)

Net Advantage to Leasing (NAL) = $50,877

Decision: Lease

166

Chapter 15 Solutions

Answers to Review Questions

1. What are some of the government requirements imposed on a public corporation that are not imposed on a private, closely held corporation?

Public corporations must submit audited financial statements to thegovernment for release to the public. Private corporations can keep their financial information confidential.

2. How are the members of the board of directors of a corporation chosen and to whom do these board members owe their primary allegiance?

Members of a corporation’s board of directors are elected by the common stockholders and owe their allegiance to these stockholders

3. What are the advantages and the disadvantages of a new stock issue?

A new stock issue raises funds and decreases the riskiness of the firm. It also tends to send a negative signal to the market since many investors believe a company would only sell new stock if future financial prospects were dim.

4. What does an investment banker do when underwriting a new security issue for a corporation?

When underwriting a new security issue an investment banker buys itand then resells it to investors.

5. How does a preemptive right protect the interests of existing stockholders?

167

A preemptive right protects the interests of existing stockholders by giving them the opportunity to preempt other investors in the purchase of new shares. If these rights are exercised, existing shareholders would maintain their same percentage of ownership after the new stock issue as before.

6. Explain why warrants are rarely exercised unless the time to maturity is small?

Warrants are rarely exercised until the time to expiration is smallbecause the market price of the warrant is greater than the exercise value. The holder of the warrant would therefore sell it in the secondary market instead of exercising it if he or she wanted to cash in.

7. Under what circumstances is a warrant’s value high? Explain.

A warrant’s value would be high when the stock price, time to expiration, and/or expected stock price volatility are high.

Answers to End-of-Chapter Problems

15-1.Original Ownership = 20,000/1,000,000 = 2%Diluted Ownership = 20,000 /1,500,000 = 1.33%

15-2. 1. Book Value Approach: (200mil. - 150mil.)/5mil. = $10 per share

2. Liquidation Value Approach: (250 - 150)/5 = $20 per share3. Replacement Value Approach: (400 - 150)/5 = $50 per share4. Dividend Growth Model: $2/(0.13 - 0.08) = $40 per share

Ms. Phinlay should buy the stock as the share is selling at a price ($20)which is lower than what she is prepared to pay ($40) to get her requiredrate of return.

168

15-3.a) .45(2,500,000) = 1,125,000 [(1,125,000 - 1) * (5 + 1)] / 2,500,000 = 2.669 2 directors

b) .55(2,500,000) = 1,375,000 [(1,375,000 – 1) * (5 + 1)] / 2,500,000 = 3.299 3 directors

15-4. a) [(1 * 2,500,000) / (5 + 1)] + 1 = 416,667 b) [(3 * 2,500,000) / (5 + 1)] + 1 = 1,250,001

c) [(5 * 2,500,000) / (5 + 1)] + 1 = 2,083,334

15-5.a) .35 (2,500,000) = 875,000 [(875,000 – 1) (5 + 1)] / 2,500,000 = 2.1 2 directors

b) [(2 * 2,500,000)/ (5 + 1)] + 1 = 833,334

15-6. Length of term = (9/3) X 3 = 9 years for each board members

15-7. NUM DIR = [(0.35 X 1,000,000 - 1) X (4 + 1)]/1,000,000 = 1.75 rounded down to 1

The minority group can elect 1 of their people to the board out of the 4 to be elected.

15-8. NUM VOTING SHARES NEEDED = [(1 X 200,000)/(7 + 1)] + 1 = 25,001. Since Ms. O'Niel holds more shares than required, she can elect herself to the board.

15-9. a) How many directors can the young stockholders elect under(i) cumulative voting procedure

NUM DIR = (600,000 X 0.30-1) X (13 + 1)/600,000 = 4.2 rounded down to 4

169

(ii) majority rule: NONE

b) What percentage of voting shares and/or proxies the dissident group must have to be able to elect 7 out of the 13 board members?

NUM VOTING SHARES NEEDED=[(7 X 600,000)/(13 + 1)] + 1=300,001

Percentage of voting shares and/or proxies = 300,001/600,000 50% (slightly greater than 50%)

15-10. Number of rights required to buy a share = 500,000/50,000 = 10

15-11. N = 2,000,000/500,000 = 4Approx. Market Value of a Right = R = (65 - 55)/(4 + 1) = $2

15-12. Market Price of stocks selling ex-rights = 65 - 2 = $63Approx. Market Value of a Right = R = (63 - 55)/(4) = $2

15-13. N = 7Approx. Market Value of a Right = R = (77 - 65)/(7 + 1) = $1.50Market Price of stocks selling ex-rights = 77 - 1.5 = $75.50

15-14. a) Approx. Market Value of a Right = R = (72 - 60)/(4 + 1) = $2.40

b) Maximum number of new shares that Johnny can buy = 700/4 = 175

c) Amount Johnny would spend = 175 X 60 = $10,500

d) Selling price of all of Johnny's rights = 700 X 2.40= $1,680

15-17. a) Approx. Market Value of a Right = R = (62 - 50)/(4 + 1) = $2.40

b) Ex-rights price = $62 - $2.40 = $59.60 per shareDiluted price after issue of new stock = $59.6 X 4/5 = $47.68

170

Option I: Sell rights and hold stocks at diluted value:

Amount obtained by selling rights = $2.40 X 60 = $144.00

Value of stocks held at diluted price = 60 X $47.68 = $2,860.80

Unused Cash = $750.00

Net worth from Option I = $3,784.80

Option II: Buy new shares:

Number of shares Selena can buy = 60/4 = 15Amount to be spent to buy 15 shares = $50 X 15 = $750. So, Selena can buy all the 15 new shares with her available cash.

Diluted price of stocks = $47.68

Net worth from Option II = $47.68 X (60 + 15) = $3,576.00

So, Selena should go for Option I, that is, Sell rights and hold her stocks at diluted value.

15-18. XV = (100 - 85) X 5 = $75

What happens to the exercise value of the warrant if the stock price changes to

a) $110 : XV = (110 - 85) X 5 = $125b) $80 : XV = $0

15-19. Issue of new common stock by Wilkerson Corporation:

Current Market Price per share of Common Stock $40 Number of Common Shares outstanding 600,000 Amount of additional funds needed $2,000,0

00 Net Income for the year $1,000,0

00 Number of shares owned by Guy Hamilton 10,000

171

Possible Subscription Prices $36 $33 $29 $26

a. Number of shares to beissued

55,556 60,606 68,966 76,923

Number of Rights required to buyone share

10.8 9.9 8.7 7.8

b. EPS before the rightsissue

$1.67 $1.67 $1.67 $1.67

EPS after the rightsissue

$1.53 $1.51 $1.49 $1.48

c. Max number of new shares Guy canbuy

926 1,010 1,149 1,282

Guy's claim to earning before therights issue

$16,667 $16,667 $16,66

7 $16,66

7 Guy's claim to earning after the

rights issue$16,667 $16,66

7 $16,66

7 $16,66

7

172

Chapter 16 Solutions

Answers to Review Questions

1. Explain the role of cash and of earnings when a corporation is deciding how much, if any, cash dividends to pay to common stockholders.

In the long-run earnings are necessary to maintain dividend payments, but at the time an actual dividend payment is made, adequate cash is necessary.

2. Are there any legal factors that could restrict a corporation in its attempt to pay cash dividends to common stockholders? Explain.

A firm may be legally restricted as to the dividends it can pay by existing bond indentures or loan agreements. It may also be restricted as to the payment of common stock dividends is scheduledpreferred stock dividends have not been paid.

3. What are some of the factors that common stockholders consider whendeciding how much, if any, cash dividends they desire from the corporation in which they have invested?

Common stockholders would consider the company’s investment opportunity, their need for income, and their tax bracket when deciding on their desire for dividends.

4. What is the Modigliani and Miller theory of dividends? Explain.

The Modigliani-Miller theory of dividends says that dividend theoryis irrelevant. They claim that it is the income produced by assetsthat is important, not how funds are distributed.

173

5. Do you believe an increased common stock cash dividend can send a signal to the common stockholders? If so, what signal might it send?

An increase in cash dividends is often seen as a positive signal. A company would be unlikely to increase its dividend if it did not believe its future prospects were good enough to sustain the higherlevel of dividends. This is because the market usually frowns upona cut in dividends.

6. Explain the bird in the hand theory of cash dividends.

The bird in the hand dividends theory says that dividends received now are better than a promise of future dividends. Uncertainty is resolved when a dividend is paid.

7. What is the effect of stock (not cash) dividends and stock splits on the market price of common stock? Why do corporations declare stock splits and stock dividends?

Stock splits and stock dividends decrease the price per share of the common stock but should not increase the total market value of all common stock outstanding unless other positive things are perceived to occur. Many companies believe that a stock split or stock dividend makes their stock more affordable and therefore moreattractive to a wider range of potential investors.

Answers to End-of-Chapter Problems

16-1. Retention Ratio = $600/$1,000 = 0.6 or 60%∴ Payout Ratio = 1-.6 = .4 = 40%

16-2. Dividend Paid = 0.4 X $50 million = $20 millionAddition to Retained Earnings = $50 mil. - $20 mil. = $30 million

174

16-3.Retained Earnings Maintain $1,000,000Retained Earnings end of 2005 750,000Additional needed for maintenance $250,000

Earnings Avail. to Common Stockholders $800,000Needed for maintenance 250,000Dividend Payout $550,000

$550,000 / $800,000 = .6875 = 68.75%

16-4.Net income $4,000,000Dividend Payout (35%) 1,400,000Addition to retained earnings $2,600,000

Retained earnings end of 2005 $1,200,000Addition to retained earnings 2,600,000Retained earnings end of 2006 $3,800,000

16-5. (Figures in $ millions)

Year1

Year2

Year3

Net Income 30 20 25Dividend Payout ratio

0.3 0.3 0.3

Dividend Paid 9 6 7.5Addition to RE 21 14 17.5Total Addition to RE = 21 + 14 + 17.5 = 52.5

16-6. (Figures in $ millions)

Year1

Year2

Year3

Net Income 30 20 25Dividend Paid 10 10 10Dividend Payout ratio

0.33 0.5 0.4

Addition to RE 20 10 15Total Addition to RE = 20 + 10 + 15 = 45

175

16-7.Equity Investment = $14mil. X 0.6 = $8.4 millionDividend to be Paid = $10mil. - $8.4 mil. = $1.6 million

16-8.a) Equity Investment = $14mil. X 0.6 = $8.4 million Dividend to be Paid = $16 mil. - $8.4 mil. = $7.6 million

b) Equity Investment = $14 mil. X 0.6 = $8.4 millionDividend to be Paid = $0

16-9.Equity funds needed $12,000,000 * .80 = $9,600,000Amount avail. to stockholders $24,000,000 – 9,600,000 =

$14,400,000Dividend per share $14,400,000 / 20,000,000 = $0.72

16-10. 20% Stock Dividend ($000s)

Increase in number of shares = 0.2 X $2 million = 400 thousandIncrease in common stock account = 400 X $1 = $400Increase in capital in excess of par account = 400 X $30 =

$12,000Total increase = $12,000 + $400 = $12,400

This increase is greater than the Retained Earnings of $10,000. Hence it is not possible to pay a 20% stock dividend.

10% Stock Dividend($000s)Increase in number of shares = 0.1 X $2 million = 200 thousandIncrease in common stock account = 200 X $1 = $200Increase in capital in excess of par account = 200 X $30 = $6,000Total increase = $6,000 + $200 = $6,200

This increase can be covered by a matching decrease in the retainedearning account keeping the total equity capital unchanged. The newretained earning will be 10,000 - 6,200 = $3,800. Hence it is possible to pay a 10% stock dividend.

176

16-11. ($000s)Common Stock ( 2 million shares, $1 par) 2,000Capital in excess of par 8,000Retained Earnings 10,000Total Common Equity 20,000

After Payment of Dividend:($000s)

Common Stock ( 2.2 million shares, $1 par) 2,200Capital in excess of par = 8,000 + 6000 = 14,000Retained Earnings = 10,000 - 6,000 - 200 = 3,800Total Common Equity 20,000

New Market Price of the Stock = $31 X 2,000/2,200 = $28.18 per share.

16-12. a) 800,000 * (1 + 0.30) = 1,040,000 new total shares 1,040,000 – 800,000 = 240,000 new shares

b) CIEP = Capital in Excess of Par CIEPbefore = $13,600,000 CIEPafter = $13,600,000 + (240,000 new shares * ($40 - $3)) CIEPafter = $22,480,000

c) CS = Common Stock CSbefore = 800,000 * $3 = $2,400,000 CSafter = 1,040,000 * $3 = $3,120,000

RE = Retained Earnings REbefore = $60,000,000 REafter = $60,000,000 – ($3,120,000 - $2,400,000) – ($22,480,000- 13,600,000) REafter = $50,400,000

16-13. 800,000 * $40 = $32,000,000

x = new stock price1,040,000 * x = $32,000,000

177

New stock price = $30.77

16-14. Before the stock split

Common Stock ( 3 million shares, $1.00 par) 3,000Capital in excess of par 7,000Retained Earnings 10,000Total Common Equity 20,000

After the stock split

Common Stock ( 9 million shares, $0.33 1/3 par) 3,000Capital in excess of par 7,000Retained Earnings 10,000Total Common Equity 20,000

New Market Price of the stock = 33 X 1/3 = $11.00 per share.

16-15. EPS (Before the stock split) = $800,000/3,000,000 = $0.27EPS (After the stock split) = $800,000/9,000,000 = $0.09

The P/E ratio will remain the same (123.75) before and after the split unless other factors influence the market’s perception of this stock’s value.

16-16. a) Dividend per share last year = $1.33 X 5/1 = $6.65b) Dividend per share last year = $1.33/1.1 X 5/1 = $6.05

16-17. Dividend per share = EPS X Payout Ratio = ($10/1) X 0.4 = $4Price of stock (ex-dividend) = $30 - $4 = $26 per share.

16-18.Spring Field Manufacturing Company's

NUMBER OF SHARES OF COMMON STOCK OUTSTANDING = 500,000NUMBER OF SHARES OF COMMON STOCK OWNED BY YOU = 500

178

NET CAPITAL EQUITY DIVIDEND DIVIDEND DIVIDENDYEAR INCOME INVESTMENTS FINANCING PAYMENT PER SHARE RECEIVED

2007 $1,000,000 $800,000 $480,000 $520,000 $1.04 $520 2008 $1,100,000 $1,000,000 $600,000 $500,000 $1.00 $500 2009 $1,200,000 $2,000,000 $1,200,00

0 $0 $0.00 $0

2010 $1,300,000 $800,000 $480,000 $820,000 $1.64 $820 2011 $1,400,000 $1,000,000 $600,000 $800,000 $1.60 $800

16-19.Spring Field Manufacturing Company'sDividend Payments:

NUMBER OF SHARES OF COMMON STOCK OWNED BY YOU = 500

AMOUNT NO. OFNET CAPITAL EQUITY FROM SHARES DIVIDEND DIVIDEND DIVIDEND

YEAR INCOME INVESTMENTS FINANCING NEW SHARES OUTSTANDING PAYMENT PER SHARE RECEIVED

2007 $1,000,000

$800,000 $480,000 500,000 $520,000 $1.04 $520

2008 $1,100,000

$1,000,000 $600,000 500,000 $500,000 $1.00 $500

2009 $1,200,000

$2,000,000 $1,200,000 $600,000 600,000 $600,000 $1.00 $500

2010 $1,300,000

$800,000 $480,000 600,000 $820,000 $1.37 $683

2011 $1,400,000

$1,000,000 $600,000 600,000 $800,000 $1.33 $667

16-20. Comprehensive Problem:

a) Expected Dividend per share = $3,000,000 X 0.5/500,000 = $3Repurchase Price = $47 + $3 = $50 per share

179

b) Number of shares that could be repurchased = $3,000,000/$50 = 60,000

c) Before Repurchase of Stock($ 000s)

Common Stock (500,000 shares, $3 par) 1,500Capital in excess of par ($7/share) 3,500Retained Earnings 5,000Total Common Equity 10,000

After Repurchase of Stock($ 000s)

Common Stock (440,000 shares, $3 par) 1,320Capital in excess of par = 7 X 440 = 3,080Retained Earnings = 5,000 + (1,500 - 1,320)

+ (3,500 - 3,080) = 5,600Total Common Equity 10,000

d) If net income next year is expected to be $4 million, what would be the EPS next year with and without the repurchase?

EPS (without repurchase) = $4,000,000/500,000 = $8EPS (with repurchase) = $4,000,000/440,000 = $9.09

e) If you own 50 shares of common stock of the company, would you like the company's decision of buying back the stocks instead of paying a dividend?

Without Repurchase:Dividend Earning Last Year = $3/Share X 50 shares = $ 150Value of stock = $47/share X 50 shares = $2,350

Total = $2,500

With Repurchase:Price of stock = $50/share X 500/440 = $56.82 per shareValue of stock = $56.82 X 50 = $2,841.00

The decision to buy back instead of paying a dividend would be preferred if the stock price were to increase to $56.82 per share with the repurchase. The taxes that may be owed on the $150 in dividends under the no repurchase scenario would decrease further the attractiveness of this alternative.

180

16-21. Before the split

# of shares 300,000Common Stock $1,200,000Par Value $4Capital in Excess of Par

$1,500,000

Retained Earnings $10,000,000Total Common Stock Equity

$12,700,000

After the split

# of shares 1,200,000Common Stock $1,200,000Par Value $1Capital in Excess of Par

$1,500,000

Retained Earnings $10,000,000Total Common Stock Equity

$12,700,000

181

Chapter 17 Solutions

Answers to Review Questions

1. What is working capital?

Working capital consists of the current assets of the firm.

2. What is the primary advantage to a corporation of investing some ofits funds in working capital?

By investing in working capital a firm gets the liquidity it needs helping it to pay its bills. The risk of the firm is therefore reduced.

3. Can a corporation have too much working capital? Explain.

A firm can have too much working capital if it is losing the opportunity to invest in high returning fixed assets and if it goesbeyond the amount of working capital needed for reasonable liquidity needs.

4. Explain how a firm determines the optimal level of current assets.

The optimal level of working capital is determined by finding the amount that balances the need for liquidity and for profitability.

5. What are the risks associated with using a large amount of short-term financing for working capital?

Using a large amount of short-term financing generally allows fundsto be raised at a lower cost but increases the firm’s risk.

182

6. What is the matching principle of working capital financing? What are the benefits of following this principle?

The matching principle is when short-term financing is used for temporary current assets while long-term financing is used for permanent current assets and fixed assets. The main benefit of this approach is that as temporary current assets are sold off the proceeds can be used to pay off the short-term debt.

7. What are the advantages and disadvantages of the aggressive workingcapital financing approach?

An aggressive working capital financing approach usually results ina lower cost of funds for a firm but a higher level of risk.

8. What is the most conservative type of working capital financing plan a company could implement? Explain.

An all equity capital structure would be the most conservative typeof working capital financing plan approach. The more long-term financing used the more conservative the financing plan, and equityis permanent financing.

Answers to End-of-Chapter Problems

17-1.a) ($150,000 + $120,000 + $80,000) = $350,000b) $350,000 – ($100,000 + $90,000) = $160,000c) ($150,000 + $120,000 + $80,000) * 0.25 = $87,500d) ($150,000 + $120,000 + $80,000) * 0.75 = $262,500

17-2.Firm 1: $10,000 + $3,000 + $2,500 = $15,500 (working capital)$15,500 - $7,500 - $4,000 = $4,000 (net working capital)Current ratio = $15,500 / ($7,500 + $4,000) = 1.35Quick ratio = ($15,500 - $3,000) / $11,500 = 1.09

Firm 2:$8,000 + $6,000 + $3,500 = $17,500 (working capital)

183

$17,500 - $3,500 - $11,000 = $3,000 (net working capital)Current ratio = $17,500 / ($3,500 + $11,000) = 1.21Quick ratio = ($17,500 - $6,000) / $14,500 = 0.79

Firm 1 is more liquid due to its higher liquidity ratios.

17-3. Company A: NWC = ($1,000 + $400) - $900 = $500Company B: NWC = ($80 + $880) - $600 = $360

Company A has the higher net working capital and would therefore generally be considered the more liquid company. Although Company A has a slightly smaller current ratio value (1.56 for A and 1.6 for B) Company A has a much higher percentage of cash in its current assets, so would likely be considered by most analysts the more liquid firm.

17-4. a) CA = $30,000 + $15,000 + $130,000 = $175,000

b) CL = $100,000 + $60,000 = $160,000

c) NWC = $175,000 - $160,000 = $15,000

d) 160/(175 X .5) or 183.86% of TCA is financed by CL. This is an aggressive approach since all TCA and most of PCA are beingfinanced with riskier short-term funds.

17-5. a) CA = $30,000 + $15,000 + $130,000 = $175,000

b) CL = $30,000 + $20,000 = $50,000

c) NWC = $175,000 - $50,000 = $125,000

d) 50/(175 X .5) or 57.14% of TCA is financed by CL. This is a relatively conservative approach. Long-term financing of $625,000 exceeds the total of fixed assets and permanent current assets, $587,500, by $37,500. Only $50,000 of the $87,500 in temporary current assets is being financed with short-term funds.

17-6. a) CA = $50 + $0 + $40 + $70 = $160

184

b) CL = $80 + $90 = $170NWC = $160 - $170 = ($10)

c) All of LuLu Belle’s current assets, and some of the fixed assets, are financed with short-term funds (current liabilities). This is an aggressive approach.

d) Reduce short-term debt, increase long-term debt and equity andinvest in marketable securities. This will increase net working capital and the current ratio.

17-7.Cash $100,00

0Inventory $200,00

0AccountsReceivable

$150,000

Net FixedAssets

$550,000

Total Assets $1,000,000

Permanent assets, net fixed assets anda small portion of temporary assets arefinanced with long-term debt andequity. This is using a very conservative approach. Your exact numbersare likely to be different, but the point is that long-term debt andequity financing are emphasized.

17-8.PCA = $225,000 * 0.60 = $135,000PCA + FA = $135,000 + $475,000 = $610,000LTD + CSEQ = $410,000 + $200,000 = $610,000

a) Cash $50,000 Accounts

Payable__$40,000_ Accounts Receivable 25,000 Notes Payable___50,000_

185

Accounts Payable $35,000Notes Payable $60,000Long-term Debt $505,00

0Common Equity $

400,000Total Liabilitiesand Equity

$1,000,000

Inventory 150,000 Long-termDebt __410,000_ Fixed Assets 475,000 Common Equity__ 200,000_ $700,000

b)Cash $50,000 Accounts

Payable__$30,000_ Accounts Receivable 25,000 Notes Payable___60,000_ Inventory 150,000 Long-termDebt __185,000_ Fixed Assets 475,000 Common Equity__425,000_ $700,000

17-9.a) Accounts payable = $180,000 Notes payable = $320,000 Long-term debt = $0 Common Equity = $200,000

Your exact numbers are likely to be different, but the point isthat short-term debt is emphasized.

17-10. CA = $30,000 + $15,000 + $130,000 = $175,000Perm. CA = $15,000 + $5,000 + $80,000 = $100,000Temp. CA = $175,000 - $100,000 = $75,000CL = Short-Term Debt + $20,000 = $75,000 = Temp.CA (By

Matching Principle)Short-Term Debt = $75,000 - $20,000 = $55,000Long-Term Debt = $675,000 - $450,000 - $55,000 - $20,000 =

$150,000

17-11. NWC = CA - CL

186

$25,000 = ($30,000 + $15,000 + $130,000) - ($20,000 + Short-Term Debt)

Short-Term Debt = $175,000 - $25,000 - $20,000 = $130,000Long-Term Debt = $675,000 - $450,000 - $130,000 - $20,000 =

$75,000

17-12. AGG.(A)(HIGH RISK)

MOD.(M)(MOD. RISK)

CON.(C)(LOWRISK)

COMMENTS

Temporary CA 75 75 75Permanent CA 100 100 100Fixed Assets 500 500 500Total Assets 675 675 675Current Liabilities

160 75 50

Long Term Debt 90 150 150Stockholders' Equity

425 450 475

Net Income 70 70 70NWC 15 100 125 LOWEST FOR A, HIGHEST

FOR CCurrent Ratio 1.09 2.33 3.50 LOWEST FOR A, HIGHEST

FOR CDebt to Asset 0.37 0.33 0.30 HIGHEST FOR A, LOWEST

FOR CROE 16.47% 15.56% 14.74% HIGHEST FOR A, LOWEST

FOR C

17-13. Assumption (i)(a) Interest Expense: 0.13 X 5 X $500,000 = $325,000(b) Interest Expense: .11 X 5 X $500,000 = $275,000Alternative (b) will save $50,000

Assumption (ii)(a) Interest Expense: 0.13 X 5 X $500,000 = $325,000(b) Interest Expense: (0.11 X 2 X $500,000) + (.14 X 2 X $500,000)

+ (.16 X $500,000)= $330,000

Alternative (a) will save $5,000

17-14. Data for graph:

187

Permanent

Temporary

Current Liabilities

Total assets

Fixed assets

Current Current if Matching Principle

Date (given) (given) Assets Assets is Followed

31-Jan

$45 $14 $14 $17 $17

28-Feb

$46 $14 $14 $18 $18

31-Mar

$34 $14 $14 $6 $6

30-Apr

$48 $14 $14 $20 $20

31-May

$40 $14 $14 $12 $12

30-Jun

$30 $14 $14 $2 $2

31-Jul

$28 $14 $14 $0 $0

31-Aug

$39 $14 $14 $11 $11

30-Sep

$45 $14 $14 $17 $17

31-Oct

$39 $14 $14 $11 $11

30-Nov

$52 $14 $14 $24 $24

31-Dec

$50 $14 $14 $22 $22

17-15.

188

W orking Capital Trends

$30

$35

$40

$45

$50

$55

$60

Jan Apr Jul OctJan Apr Jul OctJan Apr Jul OctJan Apr Jul OctJan Apr Jul OctM ONTH/ YEAR

Tem porary Current Assets

Perm anent Current Assets

Fixed Assets

W orking Capital Trends

$0

$10

$20

$30

$40

$50

$60

31-Jan

28-Feb

31-M ar

30-Apr

31-M ay

30-Jun

31-Jul

31-Aug

30-Sep

31-Oct

30-Nov

31-Dec

Fixed Assets

Perm enant Current Assets

Tem porary Current AssetsTem porary Current Assets

b) Sep. of year 4: Aug. of year 5:TA= $52.04 TA= $54.80TCA= $4.00 TCA= $5.00PCA= $9.04 PCA= $10.80FA= $39.00 FA= $39.00

c) Sep. of year 4:

(i) aggressive approachCL = over $4.00LT Financing = the remainder of $52.04

(ii) moderate approachCL = $4.00LT Financing = $48.04

(iii) conservative approach CL = less than $4.00LT Financing = the remainder of $52.04

Aug. of year 5:

(i) aggressive approachCL = over $5.00LT Financing = the remainder of $54.80

(ii) moderate approachCL = $5.00LT Financing = $49.80

(iii) conservative approach CL = less than $5.00LT Financing = the remainder of $54.80

17-16.

189

W orking Capital Trends

$30

$35

$40

$45

$50

$55

$60

Jan Apr Jul OctJan Apr Jul OctJan Apr Jul OctJan Apr Jul OctJan Apr Jul OctM ONTH/ YEAR

Tem porary Current Assets

Perm anent Current Assets

Fixed Assets

17-16. b) Sep. of year 2: Oct. of year 4:TA = $89.00 TA = $101.00TCA = $18.00 TCA = $22.50PCA = $16.00 PCA = $23.50FA = $55.00 FA = $55.00

Year 5 minimum total assets occur in January in the amount of $79.40.

Year 5 maximum total assets occur in December in the amount of$115.70.

c) Sep. of year 2: (i) aggressive approach

CL = over $18.00LT Financing = the remainder of $89.00

(ii) moderate approachCL = $18.00LT Financing = $71.00

(iii) conservative approach CL = less than $18.00LT Financing = the remainder of $89.00

Oct. of year 4: (i) aggressive approach

CL = over $22.50LT Financing = the remainder of $101.00

(ii) moderate approachCL = $22.50

190

W orking Capital Trends

$40$50$60$70$80$90$100$110$120

Jan M ay Sep Jan M ay Sep Jan M ay Sep Jan M ay Sep Jan M ay SepM O NTH/ YEAR

Tem porary Current Assets

Perm enant Current Assets

Fixed Assets

LT Financing = $78.50

(iii) conservative approach CL = less than $22.50LT Financing = the remainder of $101.00

191

Chapter 18 Solutions

Answers to Review Questions

1. What are the primary reasons that companies hold cash?

Companies hold cash to make necessary payments, to take advantage of opportunities as they arise, and to cover unforeseen emergencies.

2. Explain the factors affecting the choice of a minimum cash balance amount.

The minimum cash balance amount is determined by how easy it is to raise funds when needed, how predictable the cash flows are, and how risk averse managers are.

3. What are the negative consequences of a company holding too much cash?

A company holding too much cash would be giving up the opportunity to invest more in income producing assets

4. Explain the factors affecting the choice of a maximum cash balance amount.

The maximum cash balance amount is determined by available investment opportunities, the expected return on investments, and the transaction cost of making investments.

5. What is the difference between pro forma financial statements and a cash budget? Explain why pro forma financial statements are not usedto forecast cash needs.

192

Pro forma income statements deal with revenues and expenses that are not always cash flows while cash budgets deal only with projected cash inflows and outflows.

6. What are the benefits of “collecting early” and how do companies attempt to do this?

Money has time value. The sooner cash is collected, the better. Companies use regional collection centers and lock boxes to facilitate this.

7. What are the benefits of “paying late” (but not too late) and how do companies attempt to do this?

Because money has time value, the later cash is paid, but not too late, the better. Companies use remote disbursement banks to facilitate holding onto funds longer.

8. Refer to the Bulldog battery company’s cash budget in Table 18-7. Explain why the company would probably not issue $1 million worth of new common stock in January to avoid all short-term borrowing during the year.

Common stock financing is long-term financing so it would probably not be used to meet this short-term financing need.

Answers to End-of-Chapter Problems

18-1. Miller-Orr Model:H = 3Z - 2LTarget Cash Balance = Z = (H + 2L)/3 = ($9,000 + 2 X $3,000)/

3 = $5,000

18-2. Miller-Orr Model:

a)

193

OR

Target Cash Balance = Z = $2,424 + $2,200 = $4,624

b) Upper Limit of cash balance = H = 3Z -2L = 3 X $4,624 - 2 X $2,200= $9,472

18-3. Miller-Orr Model:a)

OR

Target Cash Balance = Z = $2,667 + $3,900 = $6,567

b) Upper Limit of cash balance = H = 3Z -2L = 3 X $6,567 - 2 X $3,900 = $11,901

3

18-4.Z = √ [(3 * $25 * $65,580) / (4 * (.05/365))] + $15,000 3

Z = √ (4,918,500 / .000547945) + $15,000 3

Z = √ 8,976,265,866 + $15,000 Z = $2,078.25 + $15,000Z = $17,078.25

194

Z=3√3 X $40 X $39,0004 X .03/365+$2,200

Z=3√3 X $40 X $52,0004 X .03/365+$3,900

18-5.H = (3 * $17,078.25) – (2 * $15,000)H = $51,234.75 – $30,000H = $21,234.75

195

18-6.Lifelong Appliances Cash

Collections

Given:20% of customers pay off their accounts in

month of sale70% of customers pay off their accounts in first

month following sale10% of customers pay off their accounts in second

month following sale

2006 ---> 2007 --->

2008 --->

Nov Dec Jan Feb Mar Apr May Jun Jul Aug Sep Oct Nov Dec Jan Feb

Sales ($000s) $131 $129 $126 $133 $139 $143 $191

$226

$242

$224

$184

$173

$166

$143

$136 $139

Monthly Cash Collections Worksheet:

(in $000s)

2006 ---> 2007 --->

2008--->

Nov Dec Jan Feb Mar Apr May Jun Jul Aug Sep Oct Nov Dec Jan FebCash collections: in month of sale

$25 $27 $28 $29 $38 $45 $48 $45 $37 $35 $33 $29

first month after sale 90 88 93 97 100 134 158 169 157 129 121 116 second month after sale 13 13 13 13 14 14 19 23 24 22 18 17 Total monthly cash collections

$129 $128 $134 $139 $152

$193

$226

$237

$218

$186

$173

$162

18-7.Lifelong Appliances Cash Collections with Stricter

Credit Terms

196

Given:40% of customers pay off their accounts in

month of sale55% of customers pay off their accounts in first

month following sale5% of customers pay off their accounts in second

month following sale

2006 ---> 2007 ---> 2008 --->

Nov Dec Jan Feb Mar Apr May Jun Jul Aug Sep Oct Nov Dec Jan Feb

Sales ($000s) $131 $129 $126 $133 $139 $143 $191

$226

$242

$224

$184

$173

$166

$143

$136 $139

Monthly Cash Collections Worksheet:

(in $000s)2006 ---> 2007 ---> 2008

--->Nov Dec Jan Feb Mar Apr May Jun Jul Aug Sep Oct Nov Dec Jan Feb

Cash collections: in month of sale

$50 $53 $56 $57 $76 $90 $97 $90 $74 $69 $66 $57

first month after sale 71 69 73 76 79 105 124 133 123 101 95 91 second month after sale

7 6 6 7 7 7 10 11 12 11 9 9

Total monthly cash collections

$128 $129 $135 $140 $162

$203

$231

$234

$209

$182

$171

$157

18-8.a) ($18,366 * .45) + ($16,523 * .40) + ($17,956 * .15) = $8,264.70 + $6,609.20 + $2693.40 =$17,567.30

b) ($22,980 * .45) + ($22,890 * .40) + ($19,500 * .15) =$10,341 + $9,156 + 2,925 =

197

$22,422

c) ($21,650 * .45) + ($23,000 * .40) + ($23,157 * .15) =$9,742.50 + $9,200 + $3,473.55 = $22,416.05

18-9.a) $2,000 + ($17,956 * .05) + $62.50 = $2,960.30

b) $2,000 + ($22,890 * .05) + $62.50 +$1,125 = $4,332.00

c) $2,000 + ($19,250 * .05) + $62.50 = $3,025.00

18-10.Lifelong Appliances Cash Expenditures

Given:2006 ---> 2007 ---> 2008

--->Nov Dec Jan Feb Mar Apr May Jun Jul Aug Sep Oct Nov Dec Jan Feb

Sales ($000s) $131 $129 $126 $133 $139

$143

$191

$226

$242

$224

$184

$173

$166

$143

$136

$139

Dec Jan Feb MarMaterials purchasing

Schedule:Order materials Manufacture

AppliancesSell appliances

Repeat each month

Cost of materials=

30% of sales

Payment for materials one month after purchase

Production costs other than purchases = 80% of purchasesSelling and marketing Expenses = 19% of sales

198

General and Administrative Expenses = $11 thousand each monthInterest

Payments =$31 thousand, paid in December

Tax payments = $100 thousand, paid in 4 installments in April, June, September, and December

Dividendpayments =

$50 thousand each, paid in June and December

Monthly Cash Expenditures Worksheet:(in $000s)

2006 ---> 2007 ---> 2008--->

Nov Dec Jan Feb Mar Apr May Jun Jul Aug Sep Oct Nov Dec Jan Feb

Materials Purchases $40 $42 $43 $57 $68 $73 $67 $55 $52 $50 $43 $41 $42 (reference only; not a cash flow)Payments for materials purchases: $40 $42 $43 $57 $68 $73 $67 $55 $52 $50 $43 $41 Other cash payments: Production costs other than purchases $33 $34 $46 $54 $58 $54 $44 $42 $40 $34 $33 $33 Selling and marketing Expenses $24 $25 $26 $27 $36 $43 $46 $43 $35 $33 $32 $27 General and Administrative Expenses $11 $11 $11 $11 $11 $11 $11 $11 $11 $11 $11 $11 Interest Payments $31 Tax payments $25 $25 $25 $25 Dividend payments $50 $50 Total Cash Outflows $108 $112 $12

6 $17

5 $173

$255

$168

$150

$163

$128

$118

$218

18-11.Lifelong Appliances Cash Expenditures, Revised

Given:2006 ---> 2007 ---> 2008

--->Nov Dec Jan Feb Mar Apr May Jun Jul Aug Sep Oct Nov Dec Jan Feb

Sales ($000s) $131 $129 $126 $133 $139

$143

$191

$226

$242

$224

$184

$173

$166

$143

$136

$139

Dec Jan Feb Mar

199

Materials purchasingSchedule:

Order materials Manufacture Appliances

Sell appliances

Repeat each month

Cost of materials=

30% of sales

Payment schedule for materials:30% paid in cash in month of purchase70% paid in cash in month following month of purchase

Production costs other than purchases = 80% of purchasesSelling and marketing Expenses = 19% of sales

General and Administrative Expenses = $11 thousand each monthInterest

Payments =$31 thousand, paid in December

Tax payments = $100 thousand, paid in 4 installments in April, June, September, and December

Dividendpayments =

$50 thousand each, paid in June and December

Monthly Cash Expenditures Worksheet:(in $000s)

2006 ---> 2007 ---> 2008--->

Nov Dec Jan Feb Mar Apr May Jun Jul Aug Sep Oct Nov Dec Jan Feb

Materials Purchases $40 $42 $43 $57 $68 $73 $67 $55 $52 $50 $43 $41 $42 (reference only; not a cash flow)Payments for materials purchases: in month of purchase $13 $13 $17 $20 $22 $20 $17 $16 $15 $13 $12 $13 in month following month of purchase $28 $29 $30 $40 $47 $51 $47 $39 $36 $35 $30 $29 Other cash payments: Production costs other than purchases $33 $34 $46 $54 $58 $54 $44 $42 $40 $34 $33 $33 Selling and marketing Expenses $24 $25 $26 $27 $36 $43 $46 $43 $35 $33 $32 $27 General and Administrative Expenses $11 $11 $11 $11 $11 $11 $11 $11 $11 $11 $11 $11 Interest Payments $31

200

Tax payments $25 $25 $25 $25 Dividend payments $50 $50 Total Cash Outflows $109 $113 $13

0 $17

8 $175

$254

$165

$149

$162

$126

$117

$219

18-12.Fit-and-Forget Fittings Cash Budget

Given:

Sales: 2006 ---> 2007 ---> 2008 --->Nov Dec Jan Feb Mar Apr May Jun Jul Aug Sep Oct Nov Dec Jan Feb

Sales ($000s) $2,266

$2,230 $2,116 $2,300 $2,402 $2,420 $3,390 $3,90

9 $4,16

4 $3,93

3 $3,16

3 $2,91

2 $2,88

6 $2,42

4 $2,35

3 $2,44

2

Collections:30% of customers pay off their accounts in month

of sale65% of customers pay off their accounts in first month

following sale5% of customers pay off their accounts in second month

following sale

Purchases & Expenses:Dec Jan Feb Mar

MaterialspurchasingSchedule:

Order materials

Manufacture Products

Sell Products

Repeat each month

Cost of materials =

20% of sales

Payment schedule formaterials:

20% paid in cash in month of purchase

80% paid in cash in month following month

201

of purchaseProduction costs other than

purchases =14% of purchases

Selling and marketingExpenses =

16% of sales

General and AdministrativeExpenses =

$180 thousand each month

Interest Payments = $500 thousand, paid in December

Tax payments = $1,600 thousand, paid in 4 installments in April, June, September, and December

Dividend payments = $855 thousand each, paid in June and December

Cash Inflows: (in $000s)2006 --->

2007 ---> 2008--->

Nov Dec Jan Feb Mar Apr May Jun Jul Aug Sep Oct Nov Dec Jan FebCash collections: in month of sale $635 $690 $721 $726 $1,01

7 $1,17

3 $1,24

9 $1,180 $949 $874 $866 $727

first month after sale 1,450 1,375 1,495 1,561 1,573 2,204 2,541 2,707 2,556 2,056 1,893 1,876 second month after sale 113 112 106 115 120 121 170 195 208 197 158 146 Total monthly cash collections $2,19

8 $2,17

7 $2,32

1 $2,40

2 $2,71

0 $3,49

7 $3,96

0 $4,082 $3,714 $3,126 $2,917 $2,749

Cash Outflows: (in $000s)2006 --->

2007 ---> 2008 --->

Nov Dec Jan Feb Mar Apr May Jun Jul Aug Sep Oct Nov Dec Jan Feb

Materials Purchases $460 $480 $484 $678 $782 $833 $787 $633 $582 $577 $485 $471 $488 (reference only; not a cash flow)Payments for materials purchases: in month of purchase $96 $97 $136 $156 $167 $157 $127 $116 $115 $97 $94 $98 in month following month of purchase

$368 $384 $387 $542 $625 $666 $629 $506 $466 $462 $388 $376

Other cash payments: Production costs other than purchases

$67 $68 $95 $109 $117 $110 $89 $82 $81 $68 $66 $68

202

Selling and marketing Expenses $339 $368 $384 $387 $542 $625 $666 $629 $506 $466 $462 $388 General and Administrative Expenses

$180 $180 $180 $180 $180 $180 $180 $180 $180 $180 $180 $180

Interest Payments $500 Tax payments $400 $400 $400 $400 Dividend payments $855 $855 Total Cash Outflows $1,05

0 $1,09

7 $1,18

2 $1,77

5 $1,63

1 $2,99

4 $1,69

1 $1,513 $1,748 $1,273 $1,190 $2,865

Net Cash Gain(Loss) $1,148 $1,08

0 $1,13

9 $627 $1,07

9 $503 $2,26

9 $2,569 $1,965 $1,854 $1,727 ($117)

Cash Flow Summary: (in $000s)

Jan Feb Mar Apr May Jun Jul Aug Sep Oct Nov Dec

1. Cash Balance at start of month $1,133 $2,28

1 $3,36

1 $4,50

0 $5,12

7 $6,20

6 $6,70

9 $8,978 $11,54

7 $13,51

2 $15,36

6 $17,09

3 2. Net Cash Gain(Loss) during month 1,148 1,080 1,139 627 1,079 503 2,269 2,569 1,965 1,854 1,727 (117)3. Cash balance at end of month before financing

2,281 3,361 4,500 5,127 6,206 6,709 8,978 11,547 13,512 15,366 17,093 16,976

(line 1 plus line 2)4. Minimum Cash Balance Desired 1,110 1,110 1,110 1,110 1,110 1,110 1,110 1,110 1,110 1,110 1,110 1,110 5. Surplus cash(deficit) (Line 3 minus line 4)

$1,171 $2,25

1 $3,39

0 $4,01

7 $5,09

6 $5,59

9 $7,86

8 $10,43

7 $12,40

2 $14,25

6 $15,98

3 $15,86

6

External Financing Summary: (in $000s)

6. External financing balance at start of month

$0 $0 $0 $0 $0 $0 $0 $0 $0 $0 $0 $0

7. New financing required 0 0 0 0 0 0 0 0 0 0 0 0 (negative amount from line 5)8. Financing repayments 0 0 0 0 0 0 0 0 0 0 0 0 (positive amount from line 5)9. External financing balance at endof month

0 0 0 0 0 0 0 0 0 0 0 0

203

10. Cash balance at end of month after financing

$2,281 $3,36

1 $4,50

0 $5,12

7 $6,20

6 $6,70

9 $8,97

8 $11,54

7 $13,51

2 $15,36

6 $17,09

3 $16,97

6

204

Chapter 19 Solutions

Answers to Review Questions

1. Accounts receivable are sometimes not collected. Why do companies extend trade credit when they could insist on cash for all sales?

Extending trade credit almost always leads to more sales. If the incremental cashflows, including the investment in accounts receivable give a positive NPV, the decision to extend trade creditwould increase the value of the firm.

2. Inventory is sometimes thought of as a necessary evil. Explain.

Inventory ties up funds and these funds are not earning an explicitreturn. Some inventory is often necessary, however, as companies try to hold the lowest acceptable amount.

3. What are the primary variables being balanced in the EOQ inventory model? Explain

The primary variables being balanced in the EOQ model are carrying costs and ordering costs. The more frequent orders are placed the lower the firm’s carrying costs and the higher its ordering costs.

4. What are the benefits of the JIT inventory control system?

The just-in-time (JIT) inventory control system lowers inventory carrying costs and tends to increase quality.

5. What are the primary requirements for a successful JIT inventory control system?

205

For a JIT system to be successful the supplier must be willing and able to deliver materials immediately and the quality of delivered materials must be high.

6. Can a company have a default rate on its accounts receivable that is too low? Explain.

A company could have a default rate on AR that would be considered too low if by liberalizing credit terms a significant increase in sales revenue and cash inflows were to result. If the increase in the default rate is more than offset by the increase in sales revenue, after all incremental cash flows are considered a positiveNPV could result.

7. How does accounts receivable factoring work? What are the benefitsto the two parties involved? What are the risks?

Factoring is when one firm sells accounts receivable (AR) to another. The purchasing firm is called a factor. The factor makesa profit by purchasing the AR at a discount. Its risk is that someof the AR may default. The selling firm gets the cash it needs.

Answers to End-of-Chapter Problems

19-1. Accounts Receivable, ACP:

Accounts Receivable = ACP X Sales/365 = 22 X $8,030,000/365 = $484,000

19-2. Accounts Receivable, ACP:

Accounts Receivable = ACP X Sales/365 = 26 X $7,600,000/365 = $541,369.86

The company appears to have relaxed its credit policy since accounts receivable increased as did the average collection period.

206

19-3. Accounts Receivable, ACP, Credit Policy:(a) ACP = 0.4 X 15 + 0.57 X 60 + 0.03 X 100 = 43.2 days(b) AR = 43.2 X $730,000/365 = $86,400

19-4. Accounts Receivable, ACP, Credit Policy:

ACP = 0.4 X 10 + 0.58 X 30 + 0.02 X 100 = 23.4 daysAR = 23.4 X $657,000/365 = $42,120

19-5.(0.25 * 10) + (0.60 * 20) + (0.15 * 30) = 2.5 + 12 + 4.5 = 19 days

19-6.(0.32 * 10) + (0.67 * 30) + (0.01 * 45) =3.2 + 20.1 + 0.45 = 23.75 days

No, the new policy should not be implemented because the ACP would increase.

207

19-7 Effect of Change of Credit Policy:Given:

All sales on credit

Old credit terms 2/15, n40

New credit terms 2/15, n60

Sales expected under old credit policy:

$350,000

Sales change expected with new credit policy:

20% increase

Under old credit policy:

40% of customers take discount, pay in

15 days

58% of customers pay at the end of

40 days

2% of customers payin

100

days

Under new credit policy:

40% of customers take discount, pay in

15 days

57% of customers pay at the end of

60 days

3% of customers payin

100

days

Bad debt expenses under old credit policy:

2% of sales

Bad debt expenses under new credit policy:

3% of sales

Short-term interest rate

7%

Long term interest rate 10%Income tax rate 40%Cost of capital 11%Cost of goods sold 80% of

salesOther operating expenses

$10,000 under old creditpolicy

Question a:

Average collection period underold policy

31.2 days (weighted average of customers paying)

Average collection period undernew policy

43.2 days (weighted average of customers paying)

Accounts Receivable under old $29,91 AR = ACP * Credit sales per day

208

policy 8 Accounts Receivable under new policy

$49,710 AR = ACP * Credit sales per day

Question b:

East-West Trading Company FinancialStatements

INCOME STATEMENT

Withold

With new

credit creditterms: terms:2/15,

n402/15,

n60(given

)(pro

forma)

Sales (all on credit) $350,000

$420,000 20% increase

Cost of Goods Sold 280,000

$336,000 increase in proportion with sales

Gross Profit 70,000 84,000 Bad debt expenses 7,000 12,600 from assumptionsOther operating expenses 10,000 $12,000 increase in proportion with salesOperating Income 53,000 59,400 Interest Expense 5,450 5,940 (ST Debt * ST Cost of Debt) + (LT Debt *

LT Cost of Debt)Before-Tax Income 47,550 53,460 Income Taxes 19,020 21,384 Net Income $28,53

0 $32,076

BALANCE SHEET, as of Dec 31

AssetsCurrent Assets: Cash & Securities $15,00

0 $18,000 increase in proportion with sales

Accounts Receivable 29,918 49,710 from Tab a Inventory 50,000 60,000 increase in proportion with salesTotal Current Assets 94,918 127,710 Property, Plant & Equipment, Net

120,000

120,000 same

Total Assets $214,918

$247,710

Liabilities & EquityCurrent Liabilities: Accounts Payable $14,91 $17,902 increase in proportion with sales

209

8 Notes Payable 35,000 $42,000 increase in proportion with salesTotal Current Liabilities

49,918 59,902

Long-Term Debt 30,000 30,000 sameTotal Liabilities 79,918 89,902 Common Stock 25,000 25,000 sameCapital in Excess of Par 60,000 60,000 sameRetained Earnings 50,000 50,000 sameTotal Stockholders' Equity

135,000

135,000

Total Liabilities & Equity

$214,918

$224,902

AFN to balance: $22,808 obtain from ST sources

Question c:

Incremental cash flows associated with the credit policy change

Initial investment at T-0 ($22,808) AFN from Tab B

Future incremental cash flows, T-1 onward:

Inflows:Increase in Sales $70,000 Outflows:Increase in Cost of Goods Sold

$56,000

Increase in Bad Debt Expense $5,600 Increase in Other Operating Exps

$2,000

Increase in Interest Expense $490 Increase in Taxes $2,364 Total Outflows $66,454

Net future incremental cash flows

$3,546 each year from T-1 onward

Question d, Investment Decision:

NPV of the Credit Policy Change:

InitialInvestment

Future Cash Flows

($22,808) $3,546 per year

NPV = $9,428 at a cost of capital 11%

210

of

19-8.

Given:

All sales on credit

Old credit terms 2/15, n40

New credit terms 2/15, n60

Sales expected under old credit policy:

$350,000

Sales change expected with new credit policy:

20% increase

Under old credit policy: 40% of customers take discount, pay in

15 days

58% of customers pay at the end of

40 days

2% of customers payin

100

days

Under new credit policy: 30% of customers take discount, pay in

15 days

60% of customers pay at the end of

60 days

10% of customers payin

100

days

Bad debt expenses under old credit policy:

2% of sales

Bad debt expenses under new credit policy:

4% of sales

Short-term interest rate 7%Long term interest rate 10%Income tax rate 40%Cost of capital 11%Cost of goods sold 80% of

salesOther operating expenses $10,000 under old credit

policy

Question a:

211

Average collection period underold policy

31.2 days (weighted average of customers paying)

Average collection period undernew policy

50.5 days (weighted average of customers paying)

Accounts Receivable under old policy

$29,918 AR = ACP X Credit sales per day

Accounts Receivable under new policy

$58,110 AR = ACP X Credit sales per day

Question b:

East-West Company Financial Statements

INCOME STATEMENT

Withold

With new

credit creditterms: terms:2/15,n40

2/15,n60

(given)

(proforma)

Sales (all on credit) $350,000

$420,000 20% increase

Cost of Goods Sold 280,000

$336,000 increase in proportion with sales

Gross Profit 70,000 84,000 Bad debt expenses 7,000 16,800 from assumptionsOther operating expenses 10,000 $12,000 increase in proportion

with salesOperating Income 53,000 55,200 Interest Expense 5,450 5,940 (ST Debt X ST Cost of

Debt) +(LT Debt X LT Cost of Debt)

Before-Tax Income 47,550 49,260 Income Taxes 19,020 19,704 Net Income $28,53

0 $29,556

BALANCE SHEET, as of Dec 31

AssetsCurrent Assets: Cash & Securities $15,00

0 $18,000 increase in proportion

with sales Accounts Receivable 29,918 58,110

212

Inventory 50,000 60,000 increase in proportion with sales

Total Current Assets 94,918 136,110 Property, Plant & Equipment, Net

120,000

120,000 same

Total Assets $214,918

$256,110

Liabilities & EquityCurrent Liabilities: Accounts Payable $14,91

8 $17,902 increase in proportion

with sales Notes Payable 35,000 $42,000 increase in proportion

with salesTotal Current Liabilities

49,918 59,902

Long-Term Debt 30,000 30,000 sameTotal Liabilities 79,918 89,902 Common Stock 25,000 25,000 sameCapital in Excess of Par 60,000 60,000 sameRetained Earnings 50,000 50,000 sameTotal Stockholders' Equity

135,000

135,000

Total Liabilities & Equity

$214,918

$224,902

AFN to balance: $31,208 obtain from ST sources

Question c:

Incremental cash flows associated with the credit policy change

Initial investment at T-0 ($31,208) AFN

Future incremental cash flows, T-1 onward:

Inflows:Increase in Sales $70,000 Outflows:Increase in Cost of Goods Sold $56,000 Increase in Bad Debt Expense $9,800 Increase in Other Operating Exps $2,000 Increase in Interest Expense $490 Increase in Taxes $684 Total Outflows $68,974

Net future incremental cash flows $1,026 each year from T-1 onward

Question d, Investment

213

Decision:

NPV of the Credit Policy Change:

InitialInvestment

Future CashFlows

($31,208) $1,026 per year

NPV = ($21,881) at a cost of capital of

11%

19-9. Effect of Change of Credit Policy:

Given:

All sales on credit

Old credit terms 3/10, n40

New credit terms 3/15, n30

Sales expected under old credit policy:

$2,000,000

Sales change expected with new credit policy:

-10% decrease

Under old credit policy: 30% of customers take discount, pay in

10 days

60% of customers pay at the end of

40 days

10% of customers pay in 100

days

Under new credit policy: 42% of customers take discount, pay in

15 days

57% of customers pay at the end of

30 days

1% of customers pay in 100

days

Bad debt expenses under old creditpolicy:

3% of sales

Bad debt expenses under new creditpolicy:

1% of sales

Short-term interest rate 8%Long term interest rate 11%

214

Income tax rate 40%Cost of capital 13%Cost of goods sold 80% of salesOther operating expenses $60,000 under old credit

policy

Question a:

Average collection period underold policy

37 days (weighted average of customers paying)

Average collection period undernew policy

24.4 days (weighted average of customers paying)

Accounts Receivable under old policy

$202,740

AR = ACP X Credit sales per day

Accounts Receivable under new policy

$120,329

AR = ACP X Credit sales per day

Question b:

A-Z Trading Company Financial Statements

INCOME STATEMENT

With old With newcredit creditterms: terms:3/10,

n403/15,n30

(given) (proforma)

Sales (all on credit) $2,000,000

$1,800,000

10% decrease

Cost of Goods Sold 1,600,000

$1,440,000

decrease in proportion with sales

Gross Profit 400,000 360,000 Bad debt expenses 60,000 18,000 from assumptionsOther operating expenses 60,000 $54,000 decrease in proportion with salesOperating Income 280,000 288,000 Interest Expense 34,810 33,210 (ST Debt X ST Cost of Debt) + (LT Debt X LT

Cost of Debt)Before-Tax Income 245,190 254,790 Income Taxes 98,076 101,916 Net Income $147,114 $152,874

BALANCE SHEET, as of Dec 31

AssetsCurrent Assets: Cash & Securities $86,000 $77,400 decrease in proportion with sales

215

Accounts Receivable 202,740 120,329 Inventory 285,000 256,500 decrease in proportion with salesTotal Current Assets 573,740 454,229 Property, Plant & Equipment, Net

652,000 652,000 same

Total Assets $1,225,740

$1,106,229

Liabilities & EquityCurrent Liabilities: Accounts Payable $85,000 $76,500 decrease in proportion with sales Notes Payable 200,000 $180,000 decrease in proportion with salesTotal Current Liabilities

285,000 256,500

Long-Term Debt 171,000 171,000 sameTotal Liabilities 456,000 427,500 Common Stock 143,000 143,000 sameCapital in Excess of Par 342,000 342,000 sameRetained Earnings 285,000 285,000 sameTotal Stockholders' Equity

770,000 770,000

Total Liabilities & Equity

$1,225,740

$1,197,500

AFN to balance: ($91,271)excess financing

Question c:

Incremental cash flows associated with the credit policy change

Initial investment at T-0 $91,271

Future incremental cash flows, T-1 onward:

Inflows:Increase in Sales ($200,000)Outflows:Increase in Cost of Goods Sold ($160,000)Increase in Bad Debt Expense ($42,000)Increase in Other Operating Exps ($6,000)Increase in Interest Expense ($1,600)Increase in Taxes $3,840 Total Outflows ($205,760)

Net future incremental cash flows $5,760 each year from T-1 onward

Question d, Investment Decision:

216

NPV of the Credit Policy Change:

InitialInvestment

Future CashFlows

$91,271 $5,760 per year

NPV = $135,579 at a cost of capital of

13%

19-10. Economic Order Quantity

EOQ = [2 X 500 X $250/$300] = 28.87 units 29 unitsNumber of orders per year = 500/29 = 17.24 orders 17 ordersOrdering Cost = 17 X $250 = $4,250

19-11. Economic Order Quantity:

EOQ = [2 X 500 X $250/$330] = 27.52 units 28 unitsNumber of orders per year = 500/28 = 17.86orders 18 ordersOrdering cost = 18 X $250 = $4,500

19-12. a) √ (2 * 1,200 * $250) / $100 =

√ 6,000 = 77.46 units per order

b) 1200 / 78 = 15.38 orders per year

19-13. Sales (2007) = 200 * (1 + 0.25) = 250 unitsCarrying Costs = $150 * (1 + 0.10) = $165Ordering Costs = $50 * (1 + 0.10) = $55

√ (2 * 250 * $55) / $165 = √ 166.66 = 12.91 units

19-14. Credit Scoring

Total Score = 3 + 4 + 3 + 4 + 3 + 2 = 19 > 12. Yes, Danny should approve the credit.

217

19-15. CriteriaPoints Score

Length of time since lastdelinquent payment:

Greater 2.5 years 2-2.5 years 1.5-2 years 1-1.5 years Less than 1 year

43210

___4________________________________

Length of Time in Business Greater 5 years 4-5 years 3-4 years 2-3 years Less than 2 years

43210

___4________________________________

Net Income Greater $100,000 $75,000-$100,000 $50,000-$75,000 $25,000-$50,000 Less than $25,000

43210

_________________2__________________

Total Score: 10$1,200,000 * 0.30 = $360,000

Yes, they will be approved. TWI will be approved for $360,000.

19-16.

Sunrise Corporation Inventory Policy

Given:

Present inventory level

60

Proposed inventory level

100

Sales expected under old inventory policy: 350 units per

218

yearSales expected with new inventory policy: 450 units per

year

Ordering cost $200 per orderCarrying cost $600 per unit per year

Unit sales price $10,000

Unit purchase price $8,000

Short-term interestrate

7%

Long term interest rate

10%

Income tax rate 40%Cost of capital 11%Cost of goods sold 80% of salesOther operating expenses

$100,000

under current inventory policy

Question a:Under old inventory

policyUnder new inventory

policy60 units 100 units

E.O.Q 15 17 Number of orders per year

23 26

Ordering cost $4,583 $5,196 Carrying cost $36,000 $60,000 Total inventory cost

$40,583 $65,196

Question b:

Sunrise Company Financial Statements

INCOME STATEMENT

Under oldinventory policy

Under new inventory policy

60 units 100

219

units(given) (pro

forma)

Sales (all on credit) $3,500,000 $4,500,000

unit sales x price, from assumptions

Cost of Goods Sold 2,800,000 3,600,000 unit sales x purchase price, from assumptions

Gross Profit 700,000 900,000 Inventory costs 40,583 65,196 from Tab aOther operating expenses

100,000 128,571 increase in proportion with sales

Operating Income 559,417 706,232 Interest Expense 13,150 15,050 (ST Debt * ST Cost of Debt) + (LT Debt *

LT Cost of Debt)Before-Tax Income 546,267 691,182 Income Taxes 218,507 276,473 Net Income $327,760 $414,709

BALANCE SHEET, as of Dec 31

AssetsCurrent Assets: Cash & Securities $55,000 70,714 increase in proportion with sales Accounts Receivable 105,000 135,000 increase in proportion with sales Inventory 480,000 800,000 assumed inventory level x unit purchase

price, from assumptionsTotal Current Assets 640,000 1,005,71

4 Property, Plant & Equipment, Net

100,000 100,000 same

Total Assets $740,000 $1,105,714

Liabilities & EquityCurrent Liabilities: Accounts Payable $100,000 128,571 increase in proportion with sales Notes Payable 95,000 122,143 increase in proportion with salesTotal Current Liabilities

195,000 250,714

Long-Term Debt 65,000 65,000 sameTotal Liabilities 260,000 315,714 Common Stock 60,000 60,000 sameCapital in Excess of Par

220,000 220,000 same

Retained Earnings 200,000 200,000 sameTotal Stockholders' Equity

480,000 480,000

Total Liabilities & Equity

$740,000 $795,714

AFN to balance: $310,000

220

Question c:

Incremental cash flows associated with the credit policy change

Initial investment at T-0 ($310,000) AFN

Future incremental cash flows, T-1 onward:

Inflows:Increase in Sales $1,000,000 Outflows:Increase in Cost of Goods Sold $800,000 Increase in Inventory Expense $24,614 Increase in Other Operating Exps $28,571 Increase in Interest Expense $1,900 Increase in Taxes $57,966 Total Outflows $913,051

Net future incremental cash flows $86,949 each year from T-1 onward

Question d, Investment Decision:

NPV of the Credit Policy Change:

InitialInvestment

Future Cash Flows

($310,000) $86,949 per year

NPV = $480,445 at a cost of capital of

11%

19-17.Given:

Present inventory level

60

Proposed inventory level

90

Sales expected under old inventory policy:

350 units per year

Sales expected with new inventory policy:

390 units per year

221

Ordering cost $200 per orderCarrying cost $600 per unit per year

Unit sales price $10,000 Unit purchase price $8,000

Short-term interest rate

7%

Long term interest rate

10%

Income tax rate 40%Cost of capital 11%Cost of goods sold 80% of salesOther operating expenses

$100,000 under current inventory policy

Question a:Under old inventory

policyUnder new inventory

policy60 units 90 units

E.O.Q 15 16 Number of orders per year

23 24

Ordering cost $4,583 $4,837 Carrying cost $36,000 $54,000 Total inventory cost

$40,583 $58,837

Question b:

Sunrise Company Financial Statements

INCOME STATEMENT

Under old inventorypolicy

Under new inventory policy

60 units 90 units(given) (pro forma)

Sales (all on credit)

$3,500,000 $3,900,000 unit sales x price, from assumptions

Cost of Goods Sold 2,800,000 3,120,000 unit sales x purchase price, fromassumptions

Gross Profit 700,000 780,000 Inventory costs 40,583 58,837

222

Other operating expenses

100,000 111,429 increase in proportion with sales

Operating Income 559,417 609,734 Interest Expense 13,150 13,910 (ST Debt X ST Cost of Debt) +

(LT Debt X LT Cost of Debt)Before-Tax Income 546,267 595,824 Income Taxes 218,507 238,330 Net Income $327,760 $357,494

BALANCE SHEET, as of Dec 31

AssetsCurrent Assets: Cash & Securities $55,00

0 61,286 increase in proportion with

sales Accounts Receivable 105,00

0 117,00

0 increase in proportion with sales

Inventory 480,000

720,000 assumed inventory level x unitpurchase price, from assumptions

Total Current Assets 640,000

898,286

Property, Plant & Equipment, Net

100,000

100,000 same

Total Assets $740,000

$998,286

Liabilities & EquityCurrent Liabilities: Accounts Payable $100,0

00 111,42

9 increase in proportion with sales

Notes Payable 95,000 105,857 increase in proportion with sales

Total Current Liabilities

195,000

217,286

Long-Term Debt 65,000 65,000 sameTotal Liabilities 260,00

0 282,28

6 Common Stock 60,000 60,000 sameCapital in Excess of Par 220,00

0 220,00

0 same

Retained Earnings 200,000

200,000 same

Total Stockholders' Equity

480,000

480,000

Total Liabilities & Equity

$740,000

$762,286

AFN to balance: $236,000

223

Question c:

Incremental cash flows associated with the credit policy change

Initial investment at T-0 ($236,000) AFN

Future incremental cash flows, T-1 onward:Inflows:Increase in Sales $400,000 Outflows:Increase in Cost of Goods Sold $320,000 Increase in Inventory Expense $18,255 Increase in Other Operating Exps $11,429 Increase in Interest Expense $760 Increase in Taxes $19,823 Total Outflows $370,266

Net future incremental cash flows $29,734 each year from T-1 onward

Question d, Investment Decision:

NPV of the Credit Policy Change:

InitialInvestment

Future CashFlows

($236,000) $29,734 per year

NPV = $34,309 at a cost of capital of

11%

19-18.Given:

Inventory Levelin Units

Expected Sales

Present inventory level

70 340 units per year

Proposed inventory level (1)

80 375 units per year

Proposed inventory level (2)

90 390 units per year

Proposed inventory level (3)

100 400 units per year

Ordering cost $160 per order

Carrying cost $400 per unit per year

224

Unit sales price $16,000 Unit purchase price $12,800

Short-term interest rate

7%

Long term interest rate

11%

Income tax rate 40%Cost of capital 13%Cost of goods sold 80% of salesOther operating expenses

$130,000 under current inventory policy

Question a:Under old

inventory policyUnder new inventory

policy (1)Under new inventory

policy (2)Under new inventory

policy (3)70 units 80 units 90 units 100 units

E.O.Q 16 17 18 18 Number of orders per year

21 22 22 22

Ordering cost $3,298 $3,464 $3,533 $3,578 Carrying cost $28,000 $32,000 $36,000 $40,000 Total inventory cost

$31,298 $35,464 $39,533 $43,578

Question b:

Windermere Corporation

INCOME STATEMENT

Under oldinventory policy

Under new inventorypolicy (1)

Under new inventorypolicy (2)

Under new inventorypolicy (3)

70 units 80 units 90 units 100 units(given) (pro forma) (pro forma) (pro forma)

Sales (all on credit)

$5,440,000 $6,000,000 $6,240,000 $6,400,000

Cost of Goods Sold

4,352,000 4,800,000 4,992,000 5,120,000

Gross Profit 1,088,000 1,200,000 1,248,000 1,280,000 Inventory costs 31,298 35,464 39,533 43,578

225

Other operating expenses

130,000 143,382 149,118 152,941

Operating Income 926,702 1,021,154 1,059,350 1,083,481 Interest Expense 13,800 14,485 14,778 14,974 Before-Tax Income

912,902 1,006,669 1,044,572 1,068,508

Income Taxes 365,161 402,668 417,829 427,403 Net Income $547,741 $604,001 $626,743 $641,105

BALANCE SHEET, as of Dec 31

AssetsCurrent Assets: Cash & Securities $65,000 71,691 74,559 76,471 Accounts Receivable 114,000 125,735 130,765 134,118 Inventory 896,000 1,024,00

0 1,152,00

0 1,280,00

0 Total Current Assets 1,075,00

0 1,221,42

6 1,357,32

4 1,490,58

8 Property, Plant & Equipment, Net

113,000 113,000 113,000 113,000

Total Assets $1,188,000

$1,334,426

$1,470,324

$1,603,588

Liabilities & EquityCurrent Liabilities: Accounts Payable $110,000 121,324 126,176 129,412 Notes Payable 95,000 104,779 108,971 111,765 Total Current Liabilities

205,000 226,103 235,147 241,176

Long-Term Debt 65,000 65,000 65,000 65,000 Total Liabilities 270,000 291,103 300,147 306,176 Common Stock 80,000 80,000 80,000 80,000 Capital in Excess of Par 320,000 320,000 320,000 320,000 Retained Earnings 518,000 518,000 518,000 518,000 Total Stockholders' Equity

918,000 918,000 918,000 918,000

Total Liabilities & Equity

$1,188,000

$1,209,103

$1,218,147

$1,224,176

AFN to balance: $125,324 $252,176 $379,412

Question c:

Incremental cash flows associated with the credit policy changes

Under new inventorypolicy (1)

Under new inventorypolicy (2)

Under new inventory policy(3)

80 units 90 units 100units

226

Initial investment at T-0

($125,324) ($252,176) ($379,412)

Excess financing(AFN)

Future incremental cash flows, T-1 onward:

Inflows:Increase in Sales $560,000 $800,000 $960,000 Outflows:Increase in Cost of Goods Sold

$448,000 $640,000 $768,000

Increase in Inventory Expense

$4,166 $8,234 $12,279

Increase in Other Operating Exps

$13,382 $19,118 $22,941

Increase in Interest Expense

$685 $978 $1,174

Increase in Taxes $37,507 $52,668 $62,242 Total Outflows $503,740 $720,998 $866,636

Net future incrementalcash flows

$56,260 $79,002 $93,364 each year from T-1onward

Question d, Investment Decision:

NPV of the Credit Policy Change:

InitialInvestment

Future Cash Flowsper Year

NPV at a cost ofcapital of

13%

Under new inventory policy(1)

($125,324) $56,260 $307,449

Under new inventory policy(2)

($252,176) $79,002 $355,532

Under new inventory policy(3)

($379,412) $93,364 $338,770

Comments: All three proposed inventory policy changes have positive NPVs, and would thereforebe acceptable at the firm's cost of capital of 13%. Policy #2, inventorylevel of 90 units, has the highest NPV, so it should be the alternative selected.

227

Chapter 20 Solutions

Answers to Review Questions

1. Companies with rapidly growing levels of sales do not need to worryabout raising funds from outside the firm. Do you agree or disagree with this statement? Explain.

Disagree. Rapidly growing firms need more assets to accommodate the increasing sales. Such firms are more likely, not less, to seek outside financing. Internal funds are often insufficient.

2. Banks like to make short-term, self-liquidating loans to businesses. Why?

Banks like to be able to see where the funds are likely to come from such that the borrower is able to use to make the required loan payments. Short term, self-liquidating loans do this since the borrowed funds are used to purchase assets that generate the needed funds.

3. What are compensating balances and why do banks require them from some customers? Under what circumstances would banks be most likely to impose compensating balances?

Compensating balances are funds that a bank requires a customer to maintain in a non-interest bearing account until the loan is retired. Banks sometimes impose compensating balance requirements so as to increase the bank’s return on a loan. Compensating balances are most likely to be used when the stated interest rate on a loan is below the bank’s required rate of return.

4. What happens when a bank charges discount interest on a loan?

When a bank charges discount interest on a loan the required interest payment is subtracted from the loan proceeds at the time

228

the loan is made. This makes the effective interest rate greater than the stated rate.

5. What is trustworthy collateral from the lenders’ perspective? Explain whether accounts receivable and inventory are trustworthy collateral.

Assets that are readily marketable, of stable value, and not likelyto “disappear” make for trustworthy collateral. Accounts receivable and inventory could meet this test depending upon their particular characteristics.

6. Trade credit is free credit. Do you agree or disagree with this statement? Explain.

Trade credit is not free. It has a cost. Who bears that cost depends on the terms of the transaction between the grantor and therecipient of the trade credit.

7. What are the pros and cons of commercial paper relative to bank loans for a company seeking short-term financing?

Commercial paper is usually a cheaper source of short-term financing for a firm, compared to bank loans. Also, a larger amount of funds can often be raised by issuing commercial paper. Bank loans are usually a more flexible source of short-term financing and establishing an on-going business relationship with abank may prove beneficial when money is tight.

Answers to End-of-Chapter Problems

20-1. a) at the end of the year $1,600/$20,000 = 8%

b) at the beginning of the year (discount loan) $1,600/($20,000- $1,600) = 8.696%

229

20-2. Effective annual interest = $2,400/($40,000 - $2,400 - $40,000X 0.10)

= $2,400/$33,600 = 7.143%

20-3. a) Effective annual interest = 1,800/20,000 = 9%b) Interest = $20,000 X 0.08 = $1,600 Effective annual interest = 1,600/(20,000-1,600) =

1,600/18,400 = 8.70%c) Interest = $20,000 X 0.075 = $1500 Compensating balance = $20,000 X 0.10 = $2,000 Effective annual interest = $1,500/($20,000 - $2,000) =

1,500/18,000 = 8.33%

(i) Which alternative is best for Ralph from minimum effective interest rate point of view?

Alternative c

(ii) Let B be the amount Ralph should borrowSo, X - 0.1 X B = $20,000Solving, B = $22,222.22So, Ralph should borrow $22,222.22 andInterest payment = 22,222.22 X .075 = $1,666.67

20-4. Interest = 14,000 X (0.16/4) = $560Compensating Balance = 14,000 X 0.10 = $1400Effective annual interest = {1 + [560/(14,000 - 1,400 - 560)]}4 - 1

= 19.94%

20-5. Duration of loan = 2 weeks = 1/26 yearInterest = 10,000 X (0.07/26) = $26.92Compensating Balance = 10,000 X 0.10 = $1,000Effective annual interest = [1 + 26.92/(10,000 - 1,000 - 26.92)]26 -

1 = 8.10%

20-6. a) Discount = 0.06 X $1,000,000 X 60/360 = $10,000b) Price = $1,000,000 - $10,000 = $990,000

230

c) Effective annual interest rate = [$1,000,000/$990,000]365/60 - 1 = 6.305%

20-7. Discount = 0.04 X $2,000,000 X 60/360 = $13,333.33Price = $2,000,000 - $13,333.33 = $1,986,666.67Effective annual interest rate = [$2,000,000/$1,986,666.67]365/60 - 1

= 4.15%

20-8. [1 + 2/98]365/(45 - 15) - 1 = 27.86%

20-9. a) 3/10, n 60 [1 + 3/97]365/(60 - 10) - 1 = 24.90%

b) 2/15, n 30 [1 + 2/98]365/(30 - 15) - 1 = 63.49%

Re-calculate the costs assuming payments were made on the 40th day in each of the above cases. Compare your results.

a) 3/10, n 60 [1 + 3/97]365/(40 - 10) - 1 = 44.86% (Higher)

b) 2/15, n 30 [1 + 2/98]365/(40 - 15) - 1 = 34.31% (Lower)

20-10. k = [1 + (3 / (100 – 3)) (365 / (45 –15))] – 1k = .4486 = 44.86%

20.11. a) (.038 * $2,000,000 * 90) / 360 = $19,000b) $2,000,000 - $19,000 = $1,981,000c) ($2,000,000 / $1,981,000)(365/90) – 1 = .03947 = 3.95%

20-12. $20,000 * 0.065 = $1,300 (interest)$1,300 / ($20,000 - $1,300) = .0695 = 6.95%

20-13. $30,000 * 0.10 = $3,000$30,000 * 0.13 = $3,900$3,000 / ($30,000 - $3,900) = .1149 = 11.5%

231

20-14. a) 1.00512 – 1 = 0.0617 = 6.17%b) 1.00612 – 1 = 0.0744 = 7.44%c) 1.0075 12 – 1 = 0.0938 = 9.38%d) 1.008 12 – 1 = 0.1003 = 10.03%

20-15. a) [1 + 1/99]365/(45 - 10) - 1 =11.05% (Lower)

b) Interest = $100,000 X 0.10 = $10,000 Effective annual interest = $10,000/($100,000 - $10,000) =

11.11%

a is the better source since 11.11% is higher than 11.05%.

20-16. a) [1 + 1/99]365/(60 - 15) - 1 = 8.49%

b) Interest = $100,000 X 0.10 = $12,000 Compensating Balance = $100,000 X 0.12 = $12,000 Effective annual interest = $10,000/($100,000 - $10,000 -

$12,000) = 12.82%

a is still the better source.

20-17. $1,500,000 / (1 - 0.09 – 0.12) = $1,898,734.18

232

Chapter 21 Solutions

Answers to Review Questions

1. What does it mean when the U.S. dollar weakens in the foreign exchange market?

When the U.S. dollar weakens in the foreign exchange market one U.S. dollar buys fewer units of another country’s currency. It costs more U.S. dollars to buy a given quantity of another country’s currency.

2. What kinds of U.S. companies would benefit most from a stronger dollar in the foreign exchange market? Explain.

U.S. companies that import goods from other countries would benefitfrom a stronger dollar. More units of a foreign currency could be purchased for a given number of dollars. Other things equal, this would lower the cost of foreign goods for the U.S. importer.

3. Under what circumstance would the U.S. dollar and the Canadian dollar be said to have achieved purchasing power parity?

The U.S. dollar and the Canadian dollar would be considered to haveachieved purchasing power parity when the exchange rate reflects the relative prices of a market basket of traded goods and servicesat the current exchange rate. There would be no incentive to convert U.S. dollars to Canadian dollars nor to convert Canadian dollars to U.S. dollars and purchase goods or services in the othercountry.

4. What are some of the primary advantages when a corporation has operations in countries other than its home country? What are someof the risks?

233

Foreign operations may reduce a company’s labor or material costs, and may increase its sales. Risks include possible seizure of company assets by a foreign government, possible cultural blunders that lead to lost sales, and exchange rate risks.

5. What is GATT, and what is its goal?

GATT is the General Agreement on Tariffs and Trade. It is a treatythat seeks to reduce trade barriers among participant nations.

Answers to End-of-Chapter Problems

21-1. a) British pound 1,000,000/1.8508 = £540,307b) Indian rupee 1/.02122 = Rs47.125 millionc) Japanese yen 1/.008800 = ¥113.636 milliond) Australian dollar 1/.7514 = A$1.331 millione) Mexican peso 1/.0900 = Peso 11.111 millionf) Israeli shekel 1/.2315 = Shekel

4.320 million

21-2. a) Chilean pesos 1/.001855 = Pesos 539.084 million

b) HK dollars 1/.1287 = HK$7.770 million

c) Singaporean dollars 1/.6321 = S$1.582 million

d) euros 1,000,000/1.2810 = 780,640.12e) Indian rupees 1/.02122 = Rs47.125 millionf) Mexican pesos 1/.0900 = Peso 11.111

milliong)Thai bahts 1/.0314 = Baht 31.847

million

234

21-3. a) 2 million Australian dollars 2 X 0.7514 =$1,502,800

b) 1.6 million Singaporean dollars 1.6 X 0.6321 =$1,011,360

c) 5 million euros 5 X 1.2810 = $6,405,000d) 2.6 million Mexican pesos 2.6 X 0.0900 =

$234,000e) 2 million Japanese yen 2 X 0.00880 = $16,000f) 25 million Thai bahts 25 X 0.02186 = $546,500

21-4.a) 1.2810b) (1) $100,000 * 113.6363636 = 11,363,636 (2) $100,000 * 0.540306894 = 54,031 (3) $100,000 * 1.113957892 = 111,396 (4) $100,000 * 11.1111111 = 1,111,111

21-5.a) 113.6363636 * 0.0900 = 10.2273 ¥ / pesob) 11.1111111 * 1.8508 = 20.5644 pesos / ₤c) .780640125 * 0.8977 = 0.7008 € / Canadian Dollard) 113.6363636 * 0.8977 = 102.0114 ¥ / Canadian Dollar

21-6. 0.6471 X 1/0.2003 = 3.23

21-7. 1 euro = 58 rupees = 9.67 HK dollars1 HK dollar = 58/9.67 = 6.00 rupees

21-8. 1 British pound = 16.9 Mexican pesos = 2.8 Singapore dollars10 million Mexican pesos = 10 X 2.8/16.9 = 1.657 million Singapore

dollars

21-9.If one British pound is equivalent to 1.5 euros, and one euro can purchase 60 baht, how many baht can one purchase with 1 millionBritish pounds?

1 British pound = 1.5 euros = 60 X 1.5 = 90 baht1 million British pounds = 90 million baht

235

21-10. British pound = 1.5 euros; .8 X 1.5 euros = 1.2 dinars; 1.2dinars X 160 yen

= 192 yen; 1 million British pounds = 192 million yen

21-11. a) 16.5 * 1/.9188 = $17.96 per share $17.96 * 100 = $1,796

b) 16.5 * 1/.70 = $23.57 per share $23.57 * 100 = $2,357

c) 16.5 * 1/1.212 = $13.61 per share $13.61 * 100 = $1,361

21-12. 55,150 * 1.020408163 = $56,275.51

21-13. 230,000 ¥ / $2,000 = 115 ¥ / $

21-14. Initial Investment = $100,000Current Value = $100,000 X 119/100 = $119,000Return on Investment = ($119,000 - $100,000)/$100,000 = 19%

21-15. Initial Investment= 1,000 shares X $37/shareXRs42/$ = Rs1,554,000

Current Value = 1,000 shares X $37/shareXRs44/$ = Rs1,628,000

Return on Investment = (Rs1,628,000 - Rs1,554,000)/Rs1,554,000 =4.76%

236